SlideShare une entreprise Scribd logo
1  sur  847
Télécharger pour lire hors ligne
2009 PREP SA on CD-ROM 
Question: 1 
You are evaluating a 6-month-old child who has a ventricular septal defect and is scheduled for cardiac 
surgery. The child’s weight is 6 kg (3rd percentile), length is at the 30th percentile, and head 
circumference is at the 50th percentile. His mother states she prepares the formula by adding 1 scoop 
of powder to 2 oz of water. She estimates that he drinks 24 oz of formula per day. You estimate the 
baby’s intake is approximately 500 kcal per day of cow milk formula, which is the recommended dietary 
allowance (RDA) for his age. According to his mother, he spits up three times a day and passes two 
soft stools daily. On physical examination, you hear a 3/6 holosystolic murmur and palpate the liver 1 
cm below the right costal margin. 
Of the following, the BEST explanation for the child’s malnutrition is 
page 1 
A. caloric requirements exceeding the RDA 
B. cow milk protein intolerance 
C. incorrect preparation of the formula 
D. pathologic gastroesophageal reflux 
E. undiagnosed pancreatic insufficiency
2009 PREP SA on CD-ROM 
Critique: 1 Preferred Response: A 
Children who have large ventricular septal defects, such as described for the child in the vignette, have 
increased pulmonary blood flow and may have ventricular hypertrophy and heart failure. Because their 
hearts have to work harder, their caloric needs often are increased above the recommended dietary 
allowance (RDA) for healthy children. The absence of significant vomiting, diarrhea, or rectal bleeding 
suggests that the patient does not have significant gastroesophageal reflux, cow milk protein 
intolerance, or pancreatic insufficiency. Incorrect preparation of the formula always should be 
considered, but the mother's reported preparation is correct. 
The RDA is defined as "a nutrient intake level that is ... sufficient to meet the nutrient requirements of 
97% of healthy individuals" in a given group, categorized by sex and age. RDAs are developed by the 
Food and Nutrition Board of the Institute of Medicine in collaboration with Health Canada. A full list of 
RDAs may be found in many reference textbooks and at the Food and Nutrition Information Center on 
the United States Department of Agriculture web site (http://fnic.nal.usda.gov). RDAs estimate energy 
requirements of healthy children and do not adjust for chronic illness states that increase caloric needs, 
such as cystic fibrosis or congenital heart disease. Children who have chronic illnesses often require 
more calories than the RDA to grow and develop. 
The estimated daily caloric requirement of a patient is the aggregate of the patient's basal metabolic 
rate and physical activity. Various mathematic equations can be used to estimate the daily caloric 
intake of children and adults of different ages. Such equations usually take into account the individual's 
age, sex, physical activity level, and either the weight and height or, preferably, the body surface area. 
For children who have chronic illnesses, more accurate estimates of daily caloric requirements can be 
obtained by using a laboratory-based technique, such as indirect calorimetry. 
References: 
McDaniel NL. Ventricular and atrial septal defects. Pediatr Rev. 2001;22:265-270. Available at: 
http://pedsinreview.aappublications.org/cgi/content/full/22/8/265 
Sonneville K. Nutritional requirements: dietary reference intakes. In: Hendricks KM, Duggan C. Manual 
of Pediatric Nutrition. 4th ed. Hamilton, Ontario, Canada: BC Decker; 2005:83-100 
page 2
2009 PREP SA on CD-ROM 
Question: 2 
You are admitting a 750-g female infant to the neonatal intensive care unit (NICU) for treatment of 
respiratory distress and presumed sepsis. The pregnancy was complicated by chorioamnionitis and 
preterm labor. The infant’s trachea was intubated, a single dose of exogenous surfactant administered, 
and both an umbilical venous catheter and umbilical arterial catheter were placed successfully in the 
delivery room. In the NICU, the infant is placed on a radiant warmer. The nurse caring for the infant 
asks if the infant will need to be transferred to an isolette incubator. 
Of the following, the MOST likely reason for this infant to be relocated into an isolette incubator is 
page 3 
A. avoidance of light damage to the eyes 
B. bronzing of the skin under the radiant warmer 
C. inability to maintain core temperature on a radiant warmer 
D. increased risk for infection under the radiant warmer 
E. increased transcutaneous water loss under the radiant warmer
2009 PREP SA on CD-ROM 
Critique: 2 Preferred Response: E 
The extremely low gestational-age newborn (ELGAN) who has a birthweight of less than 1,000 g is at 
great risk for transcutaneous evaporative water loss in an open air, nonhumidified environment such as 
a radiant warmer. This is most concerning in the first 24 to 72 hours of postnatal life, diminishing 
thereafter as the infant's skin becomes cornified. The use of a contained, convectively heated, and 
humidified environment can reduce transcutaneous evaporative water loss best in the first few days of 
the ELGAN's postnatal life. 
Ambient light is not a contributing factor to retinopathy of prematurity, for which the ELGAN is also at 
risk. Although excessive thermal warming can occur under the radiant warmer, bronzing of the skin is 
related to the use of phototherapy lights in newborns who have conjugated hyperbilirubinemia. The 
ELGAN's core temperature can be maintained using a radiant warmer, but evaporative and convective 
heat losses remain a concern over lengthy periods of time, and differences in extremity, head, and core 
temperatures may affect perfusion and acid-base status. There is no increased risk for infection on the 
radiant warmer compared with the isolette incubator. 
References: 
Dollberg S, Hoath SB. Temperature regulation in preterm infants: role of the skin-environment interface. 
NeoReviews. 2001;2:e282-e291. Available for subscription at: 
http://neoreviews.aappublications.org/cgi/content/full/2/12/e282 
Korones SB. An encapsulated history of thermoregulation in the neonate. NeoReviews. 
2004;5:e78-e85. Available for subscription at: 
http://neoreviews.aappublications.org/cgi/content/full/5/3/e78 
Sedin G. The thermal environment of the newborn infant. In: Martin RJ, Fanaroff AA, Walsh MC, eds. 
Fanaroff and Martin's Neonatal-Perinatal Medicine. 8th ed. Philadelphia, Pa: Mosby Elsevier; 
2006:585-596 
page 4
2009 PREP SA on CD-ROM 
Question: 3 
You are seeing a 1-month-old girl for follow-up after a hospitalization for acute gastroenteritis caused by 
rotavirus. Her diarrhea had decreased in the hospital while taking oral rehydration solution, but when 
her mother resumed her usual cow milk formula, the girl began to have an increased number of very 
watery stools. She appears well hydrated, and findings on her abdominal examination are normal. 
Of the following, the MOST appropriate approach to managing this infant’s diarrhea is to 
page 5 
A. change to a lactose-free formula for the next few days 
B. dilute the cow milk formula with oral rehydration solution for the next few days 
C. give her only oral rehydration solution until the diarrhea resolves 
D. readmit her to the hospital for administration of intravenous fluids 
E. repeat her stool studies to confirm the diagnosis of rotavirus infection
2009 PREP SA on CD-ROM 
Critique: 3 Preferred Response: A 
The infant described in the vignette most likely has lactase deficiency due to rotavirus infection. Lactase 
is an enzyme found in the most superficial villous portion of the intestinal brush border, which 
hydrolyzes lactose to glucose and galactose. Lactase deficiency may have several causes in children 
and adults. Primary lactase deficiency, the most common type, is a genetically determined condition 
that affects children and adults at different ages but is unusual before 5 years. Symptoms include 
abdominal distention, bloating, flatulence, or nausea after the ingestion of lactose, with the amount of 
lactose needed to cause such symptoms varying from person to person. The diagnosis is made by 
breath hydrogen testing, and management consists of removing some or all lactose from the diet. 
Congenital lactase deficiency is extremely rare. 
Secondary lactase deficiency may develop after an infectious gastroenteritis, such as rotavirus, 
giardiasis, or cryptosporidiosis. Other causes include celiac disease and enteropathy related to 
immunodeficiency. Secondary lactase deficiency is suggested when a child who has a recent diarrheal 
illness experiences worsening diarrhea or bloating after the reintroduction of lactose into the diet, as 
described for the girl in the vignette. Most children who have gastroenteritis do not develop lactase 
deficiency. For this reason, most infants can tolerate and should continue taking human milk or 
standard lactose-containing formula throughout a diarrheal illness. For very young infants (eg, <3 
months old), such as the one described in the vignette, or those who have significant fluid losses, a 
lactose-free formula may be attempted until the diarrhea resolves. Infants who are breastfed should be 
encouraged to continue breastfeeding, even if secondary lactase deficiency is suspected. 
Giving full-strength formula or human milk is recommended to supply the child with sufficient calories 
during the recovery phase of a diarrheal illness; therefore, diluting the formula or providing only oral 
rehydration solution is inappropriate. If the child is not vomiting, oral hydration is optimal, and 
intravenous hydration is not necessary. There is no need to confirm the diagnosis of rotavirus infection; 
doing so would not alter management plans. 
References: 
Dalby-Payne J, Elliott E. Gastroenteritis in children. BMJ Clinical Evidence. 2007. Available for 
subscription at: http://clinicalevidence.bmj.com/ceweb/conditions/chd/0314/0314.jsp 
Heyman MB; Committee on Nutrition. Lactose intolerance in infants, children, and adolescents. 
Pediatrics. 2006;118:1279-1286. Available at: 
http://pediatrics.aappublications.org/cgi/content/full/118/3/1279 
King CK, Glass R, Bresee, Duggan C. Managing acute gastroenteritis among children: oral rehydration, 
maintenance, and nutritional therapy. MMWR Morbid Mortal Wkly Rep Recomm Rep. 
2003;52(RR-16):1-16. Available at: http://www.cdc.gov/mmwr/preview/mmwrhtml/rr5216a1.htm 
page 6
2009 PREP SA on CD-ROM 
Question: 4 
During a prenatal visit with expectant parents, they report that they are strict vegans. They ask you to 
advise them on a healthy diet and any required supplements. The mother plans to breastfeed the 
newborn exclusively for the first 6 months. 
Of the following, you are MOST likely to tell them that their newborn may require supplemental 
A. calcium 
B. folate 
C. iron 
D. vitamin B6 
E. vitamin B12 
page 7
2009 PREP SA on CD-ROM 
Critique: 4 Preferred Response: E 
A vegan diet, by definition, excludes all foods derived from animal products. A lacto-ovo-vegetarian diet 
may include milk and eggs. Although a vegan diet may be healthy, there is a risk for vitamin B12 
deficiency because vitamin B12 is only found in foods of animal origin. Breastfeeding vegan mothers 
may produce milk that is deficient in this vitamin and require supplementation that generally is achieved 
by continuing the consumption of prenatal vitamins containing vitamin B12. 
The recommended supplementation for breastfed vegan infants to prevent vitamin B12 deficiency is 
0.4 mcg/day during the first 6 postnatal months and 0.5 mcg/day from 6 months to 1 year of age. Vegan 
infants who are not breastfed should receive iron-fortified soy infant formula until 1 year of age to avoid 
deficiencies in iron. Vegan infants require no other mineral or vitamin supplementation. 
Vegan diets in older children and adolescents may be low in calcium (similar to the typical American 
"teenage diet" that contains less than the recommended intake of dairy products), and the zinc 
consumption may be relatively low due to the absence of phytate, which renders zinc more bioavailable. 
Children who follow vegan diets may have relatively diminished overall energy intake because such 
diets commonly are low in fat and high in fiber. 
Review of nutrient intake and energy intake in conjunction with growth curves of children eating 
vegan diets in both the United States and the United Kingdom demonstrate no significant health issues. 
Height and weight measured in vegan populations may be slightly lower than average but not in the 
range of failure to thrive or short stature. Adolescents eating vegan diets are more likely than 
adolescents eating a typical American diet to meet nutritional goals, including recommended intake of 
fruits and vegetables. Vegan adolescents are less likely to be obese because they consume fewer 
foods high in fat. However, they remain at risk for vitamin B12 deficiency and should consume at least a 
daily multivitamin. They are less likely to have anemia but just as likely to have low calcium intake as 
their non-vegan peers. Adolescents who follow a lacto-ovo-vegetarian diet are less likely to have 
deficiencies in vitamin B12, calcium, and iron. 
Folate and vitamin B6 are not likely to be deficient in persons who consume vegan diets because 
those nutrients are found in many legumes, fruits, and vegetables that are the mainstays of the diet. 
References: 
Kleinman RE. Nutritional aspects of vegetarian diets. In: Pediatric Nutrition Handbook. 5th ed. Elk 
Grove Village, Ill: American Academy of Pediatrics; 2003:191-208 
Mangels AR, Messina V. Considerations in planning vegan diets: infants. J Am Diet Assoc. 
2001:101:670-677. Abstract available at: http://www.ncbi.nlm.nih.gov/pubmed/11424546 
Messina V, Mangels AR. Considerations in planning vegan diets: children. J Am Diet Assoc. 
2001:101:661-669. Abstract available at: http://www.ncbi.nlm.nih.gov/pubmed/11424545 
Moilanen BC. Vegan diets in infants, children and adolescents. Pediatr Rev. 2004:25:174-176. 
Available at: http://pedsinreview.aappublications.org/cgi/content/full/25/5/174 
Perry CL, McGuire MT, Neumark-Sztainer D, Story M. Adolescent vegetarians: how well do their dietary 
patterns meet the Healthy People 2010 objectives? Arch Pediatr Adolesc Med. 2002; 156:431-437. 
Available at: http://archpedi.ama-assn.org/cgi/content/full/156/5/431 
page 8
2009 PREP SA on CD-ROM 
Question: 5 
You are treating a child who has suffered a splenic injury and is being transfused with large volumes of 
packed red blood cells for severe anemia. He weighs 10 kg and has received 4 units thus far. 
Of the following, the finding on electrocardiography that is MOST likely to represent a serious 
complication of his therapy is 
page 9 
A. atrial flutter 
B. delta waves 
C. prominent U waves 
D. supraventricular tachycardia 
E. tall-peaked T waves
2009 PREP SA on CD-ROM 
Critique: 5 Preferred Response: E 
Administration of fluids and blood products can be essential for resuscitation of the trauma patient but 
may lead to potentially dangerous electrolyte imbalances. Recognition of these abnormalities, which 
may be subtle, can be important and even lifesaving. When large volumes of blood are transfused, as 
reported for the child in the vignette, hyperkalemia may occur, which is believed to be the result of 
extravasation of potassium from the red blood cells that have been irradiated and stored over time. 
Hyperkalemia can be associated with paresthesias, weakness, and tingling, although cardiac toxicity 
typically precedes such symptoms. Severe cardiac rhythym changes may begin abruptly. The classic 
electrocardiographic sign of hyperkalemia is tall, peaked T waves (Item C5A), particularly as the serum 
potassium concentration approaches or exceeds 5.0 to 6.0 mEq/L (5.0 to 6.0 mmol/L). As hyperkalemia 
progresses, other changes in the ECG, such as widening of the QRS complex, may be noted. The 
rhythm changes that occur as a result of hyperkalemia, including ventricular arrhythmias, may develop 
abruptly, leading to sudden changes in the patient's clinical condition. 
Another potential electrolyte complication of fluid and blood administration is hypocalcemia, which 
results from the citrate-containing anticoagulants that bind free calcium. Hypocalcemia may manifest on 
ECG as a prolonged QT interval, which tends to widen as the ionized calcium concentrations decrease. 
Atrial flutter, a primary disease of the atrial tissue, would not be expected in the patient described in 
the vignette. The delta wave refers to the ECG finding of pre-excitation, seen in conditions such as the 
Wolff-Parkinson-White syndrome that have an associated "bypass" tract allowing for excitement of the 
His-Purkinje system without passage through the atrioventricular node (Item C5B). 
The U wave may be seen in hypokalemia (Item C5C), particularly as concentrations decrease below 
2.7 mEq/L (2.7 mmol/L), or hypercalcemia (calcium concentrations exceeding 12.0 mg/dL [3.0 mmol/L]), 
which would not be expected in the patient described in the vignette. Hypercalcemia also may lead to a 
diminished QT interval, and with more severe hypercalcemia, second- or third-degree heart block (Item 
C5D) may develop. Supraventricular tachycardia (Item C5E) would not be expected to occur as a result 
of a large-volume transfusion process. 
References: 
Galel SA, Naiman JL. Use of blood and blood products. In: Rudolph CD, Rudolph AM, eds. Rudolph's 
Pediatrics. 21st ed. New York, NY: McGraw-Hill Medical Publishing Division; 2003:1576-1581 
Vetter V. Arrhythmias. In: Moller JH, Hoffman JIE, eds. Pediatric Cardiovascular Medicine. Philadelphia, 
Pa: Churchill Livingstone; 2000:833-884 
page 10
2009 PREP SA on CD-ROM 
page 11 
Critique: 5 
The electrocardiogram in hyperkalemia shows peaked T waves. (Courtesy of 
A. Friedman)
2009 PREP SA on CD-ROM 
page 12 
Critique: 5 
In Wolff-Parkinson-White syndrome, delta waves (arrows) are present that 
represent pre-excitation depolarization of the QRS complex. (Courtesy of A. 
Friedman)
2009 PREP SA on CD-ROM 
page 13 
Critique: 5 
The U wave (arrow) may be observed in hypokalemia (shown here in which 
there is also ST depression and flattening of the T wave) or hypercalcemia. 
(Courtesy of A. Friedman)
2009 PREP SA on CD-ROM 
page 14 
Critique: 5 
In third-degree heart block that may result from severe hypercalcemia, P 
waves are completely dissociated from QRS complexes. (Courtesy of A. 
Friedman)
2009 PREP SA on CD-ROM 
page 15 
Critique: 5 
Narrow complex tachycardia that is consistent with supraventricular 
tachycardia or paroxysmal atrial tachycardia. (Courtesy of A. Friedman)
2009 PREP SA on CD-ROM 
Question: 6 
A 15-year-old girl presents to the emergency department with a 4-week history of nasal drainage and 
face pain and a 2-week history of frontal headaches and fatigue. Her mother complains that her 
daughter has an "attitude" and has not been respectful or seemed to care about anything for the past 2 
weeks. The daughter awoke this morning with a headache and vomited. On physical examination, the 
adolescent is afebrile and has normal vital signs. She responds slowly to questions and is not oriented 
to the date. She complains of pain to palpation of her cheeks and forehead. She has no nuchal rigidity 
and no focal weakness. The remainder of the physical examination findings are normal. 
Of the following, the BEST initial diagnostic procedure is 
page 16 
A. computed tomography scan of the head with intravenous contrast 
B. emergent electroencephalography to rule out nonconvulsive status epilepticus 
C. lumbar puncture to rule out meningitis 
D. nasal swab for bacterial culture 
E. urine drug screen for barbiturates, amphetamines, and cocaine
2009 PREP SA on CD-ROM 
Critique: 6 Preferred Response: A 
The subacute onset of mental status changes described for the adolescent in the vignette warrants an 
emergency evaluation. In most cases, neuroimaging is indicated, along with appropriate laboratory 
testing. 
The relatively nonspecific pain and what her mother perceives as common emotional problems 
(apathy in a teenager) probably represent early frontal lobe symptoms. The headache on awakening 
and vomiting are concerning for increased intracranial pressure (ICP). Confusion and psychomotor 
retardation on the mental status examination indicate involvement of the central nervous system. A 
focal, ischemic, ictal, infectious/inflammatory, or toxic/metabolic process must be identified urgently. A 
brain abscess is suggested by the prominent facial pain in this setting; the sinuses are a common 
source of brain abscesses (Item C6). Brain abscesses often present only with nonspecific pain and not 
with fever. 
Head computed tomography (CT) scan is preferred for this patient because the constellation of pain, 
confusion, and morning vomiting makes a focal intracranial mass a possibility. Increased ICP is 
associated with morning vomiting because ICP is highest in the morning. Contrast is recommended 
because of the insidious onset, which could indicate either a neoplasm or infectious process. 
Intravenous contrast is not needed for all neuroimaging procedures. However, it increases the 
diagnostic yield of imaging studies where either neoplasm or infection is suspected because both 
typically involve some degradation of the blood-brain barrier or hypervascularity, resulting in contrast 
enhancement at the site of the lesion. Magnetic resonance imaging (MRI) with contrast also is a good 
choice. The advantage of MRI is higher spatial and soft-tissue resolution. Disadvantages of MRI 
compared with CT include: 1) less availability for emergency department studies; 2) need for 
pharmacologic sedation in agitated patients because sedation affects mental status, thereby masking 
disease-related mental status; 3) longer time in the scanner, which could delay treatment decisions; 
and 4) cost. Thus, in most cases, a head CT scan with contrast is preferred as the initial study in the 
emergency department. 
page 17 
Electroencephalography (EEG) is an important test for assessment of a patient who has 
encephalopathy of unclear cause to rule out nonconvulsive status epilepticus (NCSE), particularly if the 
patient is known to have epilepsy. If an EEG cannot be obtained rapidly, intravenous administration of 
0.1 mg/kg lorazepam can treat NCSE immediately, although this would not clear confusion about other 
causes. For this adolescent, the facial pain makes the diagnosis of NCSE less likely than a brain 
abscess. 
A lumbar puncture may be needed to rule out meningitis or encephalitis, but the pain and morning 
vomiting more strongly suggest the possibility of an intracranial mass. Lumbar puncture prior to head 
CT is not advised in this case because it could reduce pressure below the foramen magnum and result 
in herniation from the supratentorial mass. Toxicology screening and nasal swabs are reasonable but 
not the preferred initial diagnostic tests because they will not affect emergency management of the 
increased intracranial pressure. 
References: 
Goodkin HP, Harper MB, Pomeroy SL. Intracerebral abscess in children: historical trends at Children's 
Hospital Boston. Pediatrics. 2004;113:1765-1770. Available at: 
http://pediatrics.aappublications.org/cgi/content/full/113/6/1765 
Haslam RHA. Brain abscess. In: Kliegman RM, Behrman RE, Jenson HB, Stanton BF, eds. Nelson 
Textbook of Pediatrics. 18th ed. Philadelphia, Pa: Saunders Elsevier; 2007:2524-2525
2009 PREP SA on CD-ROM 
Kan L, Nagelberg J, Maytal J. Headaches in a pediatric emergency department: etiology, imaging, and 
treatment. Headache. 2000;40:25-29. Abstract available at: 
http://www.ncbi.nlm.nih.gov/pubmed/10759899 
page 18
2009 PREP SA on CD-ROM 
page 19 
Critique: 6 
Brain abscess: Contrast-enhanced computed tomography scan of the head 
reveals a rim-enhancing lesion extending to an area of bony destruction in an 
opacified left frontal sinus. Vasogenic edema surrounding the abscess 
creates a mass effect and shift of the midline to the right. (Courtesy of D. 
Krowchuk)
2009 PREP SA on CD-ROM 
Question: 7 
A 5-year-old girl who is new to your practice presents to the clinic for a prekindergarten physical 
examination. Her primary caretaker, the maternal grandmother, reports that the child’s mother used 
multiple street drugs throughout her pregnancy as well as medications prescribed for seizure and 
bipolar disorders. The grandmother is concerned that this child’s speech development is delayed. On 
physical examination, you note that the girl has wide-spaced eyes, a short nose, and midface 
hypoplasia. 
Of the following, the substance that is MOST likely to be associated with this child’s dysmorphic 
features is 
page 20 
A. lithium 
B. lysergic acid diethylamide (LSD) 
C. marijuana 
D. methamphetamine 
E. phenobarbital
2009 PREP SA on CD-ROM 
Critique: 7 Preferred Response: E 
The features described for the child in the vignette are most consistent with fetal anticonvulsant 
syndrome, which can occur following exposure to numerous medications, including phenytoin, 
carbamazepine, valproate, and phenobarbital. Multiple authors have observed a 10% to 20% incidence 
of birth defects in infants exposed to phenobarbital in utero. Anomalies include midface hypoplasia, 
ocular hypertelorism (Item C7A), nail hypoplasia (Item C7B), cleft lip+/-cleft palate, and heart defects as 
well as developmental delay and pre- and postnatal growth failure. 
page 21 
Phenobarbital-exposed newborns may exhibit a withdrawal syndrome that is evidenced by 
tremulousness and increased activity. Of interest, such infants are likely to have lower serum bilirubin 
concentrations than nonexposed neonates. 
Prenatal exposure to lithium is associated with an increased risk of cardiac malformations (eg, 
Ebstein anomaly) in the fetus. If the mother takes lithium near term, the exposed neonate may have 
cyanosis, hypotonia, abnormalities of cardiac rhythm, goiter, hypothyroidism, and nephrogenic diabetes 
insipidus. Lithium exposure is not associated with dysmorphic features or developmental delays. 
Despite popular belief, lysergic acid diethylamide (LSD) generally is not associated with birth defects 
or withdrawal symptoms in prenatally exposed infants. Although there are isolated case reports of birth 
defects in exposed neonates, an increased risk for anomalies is not borne out by epidemiologic studies. 
It is important to note, however, that LSD users often abuse other substances, underscoring the 
importance of taking an in-depth drug/substance abuse history in pregnant women. 
Marijuana use during pregnancy is not known to be associated with an increased incidence of birth 
defects, dysmorphic features, or developmental delay in exposed offspring, although further study is 
needed in this regard. Some investigations have shown reduced fetal growth in exposed pregnancies, 
but this is not confirmed. Prenatally exposed newborns may have tremulousness, increased irritability, 
and abnormal visual response to light stimulus. 
Although there does not appear to be an increase in congenital anomalies associated with 
methamphetamine use during pregnancy, further study is necessary. There are reports of decreased 
birthweight in exposed neonates. A neonatal withdrawal syndrome consisting of abnormal sleep 
patterns, tremulousness, poor feeding, and increased tone has been observed commonly. Once again, 
it is important to consider polydrug abuse in these instances. 
As with all teratogens, the timing of exposure is critical, with the most vulnerable period of embryonic 
development occurring between 18 and 60 days after conception, during organogenesis. Dosage of the 
offending agent also is important, as are route of administration, modifying environmental factors, and 
genetic background of the mother and fetus. 
References: 
Gallagher RC, Kingham K, Hoyme HE. Fetal anticonvulsant syndrome. In: Cassidy SB, Allanson JE, 
eds. Management of Genetic Syndromes. 2nd ed. Hoboken, NJ: Wiley-Liss; 2005:239-250 
Phenobarbital, lithium, LSD, marijuana, methamphetamine. Reprotox. Available for subscription at: 
http://www.reprotox.org 
Phenobarbital, lithium, LSD, marijuana, methamphetamine. Teris. Available for subscription at: 
http://depts.washington.edu/terisweb/teris
2009 PREP SA on CD-ROM 
page 22 
Critique: 7 
Wide-spaced eyes and a short upturned nose are facial features of the fetal 
anticonvulsant syndrome. This child was exposed to phenytoin. (Courtesy of 
M. Rimsza)
2009 PREP SA on CD-ROM 
page 23 
Critique: 7 
Nail hypoplasia is observed in a child who has fetal anticonvulsant syndrome. 
(Courtesy of the Media Lab at Doernbecher)
2009 PREP SA on CD-ROM 
Question: 8 
A 15-year-old girl comes to the urgent care clinic complaining of lower abdominal pain for 48 hours. She 
is nauseated but has had no fever, vomiting, or diarrhea. She is afebrile and denies abdominal trauma. 
She localizes the pain to the left lower quadrant and describes it as intermittent, stabbing pain episodes 
separated by intervals of more continuous dull pain. She has never been sexually active. Her last 
menstrual period was 1 week ago. She has had no vaginal discharge or itching. On physical 
examination, she has left lower quadrant guarding and rebound tenderness. Her pelvic examination 
shows no vaginal discharge or uterine tenderness, although there is an exquisitely tender mass in the 
left adnexal area. 
Of the following, the MOST likely diagnosis is 
page 24 
A. appendicitis 
B. endometritis 
C. ovarian torsion 
D. sacroiliitis 
E. splenic rupture
2009 PREP SA on CD-ROM 
Critique: 8 Preferred Response: C 
Torsion of any adnexal structure, including the ovary, can result in an acute pelvic mass (Item C8). The 
most common causes of adnexal torsion in young women are cysts and neoplasms, with about 6% of 
torsions in one series occurring in the setting of normal-appearing ovaries. Histopathology was benign 
in more than 90% of cases in this series. Torsion occurs when masses cause the ovary to swing on its 
vascular pedicle, and larger masses generally are associated with a greater potential for torsion until 
the size of the mass impedes movement. The onset of pain associated with torsion can be abrupt, 
sharp, and very severe. However, with intermittent or partial torsion, intense periods of intermittent pain 
may be separated by generalized aching during detorsion, as described for the teenager in the vignette. 
Pain can occur for several days or weeks prior to a complete torsion. Nausea or vomiting can 
accompany severe pain. If adnexal torsion is diagnosed, prompt intervention (untwisting the adnexa 
usually via laparoscopy) is indicated to preserve ovarian function. 
Appendicitis is in the differential diagnosis for lower abdominal pain, but is less likely when pain 
occurs in the left lower quadrant. Splenic rupture is a surgical emergency, like ovarian torsion, but the 
finding of an adnexal mass on examination is unlikely. Endometritis is not very likely in a young woman 
who is not sexually active and more commonly presents with midline uterine pain. An orthopedic cause 
for this patient's pain (eg, sacroiliitis) is unlikely, but it remains in the differential diagnosis of pelvic pain. 
Sacroiliitis usually presents with low back or hip pain. 
References: 
Adams Hillard PJ. Pelvic masses. In: Neinstein LS, eds. Adolescent Health Care: A Practical Guide. 5th 
ed. Philadelphia, Pa: Lippincott Williams & Wilkins; 2008:706-713 
Growdon WB, Laufer MR. Ovarian torsion. UpToDate Online 15.3. 2008. Available for subscription at: 
http://www.utdol.com/utd/content/topic.do?topicKey=gyn_surg/5273 
Laufer MR, Goldstein DP. Gynecologic pain: dysmenorrhea, acute and chronic pelvic pain, 
endometriosis, and premenstrual syndrome. In: Emans SJH, Laufer MR, Goldstein DP, eds. Pediatric 
and Adolescent Gynecology. 5th ed. Philadelphia, Pa: Lippincott, Williams & Wilkins; 2005:417-476 
Varras M, Tsikini A, Polyzos D, Samara Ch, Hadjopoulos G, Akrivis Ch. Uterine adnexal torsion: 
pathologic and gray-scale ultrasonographic findings. Clin Exp Obstet Gynecol. 2004;31:34-38. Abstract 
available at: http://www.ncbi.nlm.nih.gov/pubmed/14998184 
page 25
2009 PREP SA on CD-ROM 
page 26 
Critique: 8 
Ovarian torsion: Color flow Doppler ultrasonography of the right ovary shows 
abundant flow in the adjacent pelvic tissue (blue, red, and orange color seen 
inferiorly) but none in the ovary (the area within the dashed line).
2009 PREP SA on CD-ROM 
Question: 9 
A 2-year-old boy who has chronic renal failure is brought to the emergency department for evaluation of 
nausea, fatigue, and muscle weakness. On physical examination, the boy has a heart rate of 140 
beats/min, decreased perfusion, and palpable pulses. You obtain electrocardiography (Item Q9). 
Electrolyte measurements include a potassium concentration of 7.5 mEq/L (7.5 mmol/L) and a glucose 
value of 72.0 mg/dL (4.0 mmol/L). 
Of the following, the MOST appropriate initial treatment is administration of 
page 27 
A. calcium chloride 
B. insulin 
C. normal saline 20 mL/kg 
D. sodium polystyrene sulfonate 
E. verapamil
2009 PREP SA on CD-ROM 
page 28 
Question: 9 
In hyperkalemia, electrocardiography demonstrates peaked T waves. 
(Courtesy of A. Friedman)
2009 PREP SA on CD-ROM 
Critique: 9 Preferred Response: A 
Hyperkalemia can be asymptomatic or produce symptoms such as nausea, fatigue, and muscle 
weakness, as reported for the boy in the vignette. The most concerning effect of hyperkalemia is on 
cardiac membrane polarization. Electrocardiographic changes typically consist of peaked T waves and 
progress to widening of the QRS complexes with dampening of P waves as the hyperkalemia 
increases. Without treatment, the patient eventually develops ventricular fibrillation or asystole. 
The electrocardiographic tracing of the boy described in the vignette demonstrates peaked T waves 
(Item C9), and he has decreased perfusion. The immediate priority is to stabilize his cardiac membrane 
potential and decrease his risk of arrhythmias, which is accomplished best by administration of 
intravenous calcium chloride, an agent that has a rapid onset of action. Insulin causes potassium to 
shift to the intracellular spaces, thereby decreasing serum concentrations, but it always should be 
administered in conjunction with glucose to avoid hypoglycemia. Sodium bicarbonate administered 
intravenously also produces intracellular shifts of potassium and may be useful in patients who have 
metabolic acidosis. 
page 29 
Treatments to enhance the elimination of potassium often are indicated in the treatment of 
hyperkalemia, but they have a slower onset of action and, therefore, should not be the first line of 
treatment in life-threatening hyperkalemia. Sodium polystyrene sulfonate, which can be administered 
orally or rectally, exchanges sodium for potassium, which subsequently is eliminated from the body. 
Loop diuretics also can enhance potassium elimination. Dialysis may be indicated when ongoing 
elevated potassium concentrations are expected, such as in renal failure, or with very high acute 
concentrations, as seen with tumor lysis syndrome or rhabdomyolysis. 
Administration of verapamil is not indicated in the treatment of hyperkalemia; it has been reported to 
be associated with the development of complete heart block in this setting. Normal saline has a minimal 
effect on hyperkalemia. 
References: 
Greenbaum LA. Electrolyte and acid-base disorders: potassium. In: Kliegman RM, Behrman RE, 
Jenson HB, Stanton BF, eds. Nelson Textbook of Pediatrics. 18th ed. Philadelphia, Pa: Saunders 
Elsevier; 2007:279-284 
Hauser GJ, Kulick AF. Electrolyte disorders in the pediatric intensive care unit. In: Wheeler DS, Wong 
HR, Shanley TP, eds. Pediatric Critical Care Medicine: Basic Science and Clinical Evidence. New York, 
NY: Springer-Verlag; 2007:1156-1175
2009 PREP SA on CD-ROM 
page 30 
Critique: 9 
In hyperkalemia, electrocardiography demonstrates peaked T waves. 
(Courtesy of A. Friedman)
2009 PREP SA on CD-ROM 
Question: 10 
A 13-year-old girl who has just moved to the United States from Brazil comes to your office because her 
mother is worried that she is not "developing yet." On physical examination, her height is 50 inches, and 
she has a triangular face, a low hairline, high-arched palate, and a shield-shaped chest (Item Q10). 
Breast tissue is not visible or palpable, but there is Sexual Maturity Rating 3 pubic hair. You obtain bone 
age radiography and a karyotype and measure serum luteinizing hormone and follicle-stimulating 
hormone. 
Of the following, the MOST appropriate additional laboratory measurement is 
page 31 
A. adrenocorticotropic hormone 
B. prolactin 
C. 17-hydroxyprogesterone 
D. testosterone 
E. thyroid-stimulating hormone
2009 PREP SA on CD-ROM 
page 32 
Question: 10 
Shield-shaped chest and lack of breast development, as described for the girl 
in the vignette. (Courtesy of M. Rimsza)
2009 PREP SA on CD-ROM 
Critique: 10 Preferred Response: E 
The clinical findings described for the girl in the vignette are characteristic of Turner syndrome (gonadal 
dysgenesis) associated with an abnormality of one X chromosome. Girls who have this disorder usually 
are short (mean adult height, approximately 55 inches without growth hormone treatment); have poorly 
developed ovaries; and often have dysmorphisms, including a triangular facies, low hairline, 
high-arched palate, hypoplastic nipples, and an increased carrying angle. They may have left heart 
disorders such as coarctation of the aorta as well as horseshoe kidney or other renal malformations. 
Initial screening studies to diagnose Turner syndrome include a karyotype and measurement of 
luteinizing hormone (LH) and follicle-stimulating hormone (FSH). Most girls who have Turner syndrome 
do not initiate normal puberty. Concentrations of LH and FSH rise as they reach pubertal age range 
because they have ovarian failure. Although concentrations of estradiol and other estrogens are low, 
clinical estradiol assays are not designed to provide accurate values in the low-normal range expected 
in early puberty. Therefore, physical findings such as breast development are a better marker of 
estrogen effect than measurements of estrogen. 
page 33 
Adolescents who have Turner syndrome are at higher risk of developing chronic lymphocytic 
thyroiditis and hypothyroidism than the general population. Approximately 20% of affected adolescent 
girls have antibody-positive autoimmune chronic lymphocytic thyroiditis, and 5% to 10% develop overt 
hypothyroidism. Accordingly, measurement of thyroid-stimulating hormone is an appropriate laboratory 
test for patients such as the girl described in the vignette. An elevated value indicates primary 
hypothyroidism and the need for confirmatory assessment of free thyroxine and antithyroid antibodies 
(thyroperoxidase, antimicrosomal, or antithyroglobulin). 
Abnormalities of the hypothalamic-pituitary-adrenal axis are unusual in patients who have Turner 
syndrome. Therefore, measurement of adrenocorticotropic hormone is not useful. Measurement of 
prolactin would be useful if the girl had a pituitary or hypothalamic problem, but her clinical findings 
strongly point to Turner syndrome. A 17-hydroxyprogesterone value would be elevated in the presence 
of an adrenal biosynthetic defect leading to the development of the most common form of congenital 
adrenal hyperplasia (cyp21 or 21-hydroxylase deficiency) as well as some of the less common 
disorders of adrenal biosynthesis. Measuring testosterone would be reasonable if there were evidence 
of inappropriate masculinization, such as clitoromegaly and a growth spurt. Some girls who have Turner 
syndrome have functioning Y chromosomal DNA and could have androgenization, but this is unusual. 
The presence of Y chromosomal DNA does increase the risk of gonadal malignancy, and girls who 
have significant Y chromosomal DNA on testing often require prophylactic gonadectomy. 
References: 
Chiovato L, Larizza D, Bendinelli G, et al. Autoimmune hypothyroidism and hyperthyroidism in patients 
with Turner's syndrome. Eur J Endocrinol. 1996;134:568-575. Abstract available at: 
http://www.ncbi.nlm.nih.gov/pubmed/8664977 
Doswell BH, Visootsak J, Brady AN, Graham JM Jr. Turner syndrome: an update and review for the 
primary pediatrician. Clin Pediatr. 2006;45:301-313. Abstract available at: 
http://www.ncbi.nlm.nih.gov/pubmed/16703153 
Frias JL, Davenport ML, Committee on Genetics and Section on Endocrinology. Health supervision for 
children with Turner syndrome. Pediatrics. 2003;111:692-702. Available at: 
http://pediatrics.aappublications.org/cgi/content/full/111/3/692 
Matura LA, Ho VB, Rosing DR, Bondy CA. Aortic dilatation and dissection in Turner syndrome.
2009 PREP SA on CD-ROM 
Circulation. 2007;116:1663-1670. Abstract available at: http://www.ncbi.nlm.nih.gov/pubmed/17875973 
Sybert VP, McCauley E. Turner's syndrome. N Engl J Med. 2004;351:1227-1238. Extract available at: 
http://content.nejm.org/cgi/content/extract/351/12/1227 
Turner Syndrome Society Website. Available at: http://www.turnersyndrome.org 
page 34
2009 PREP SA on CD-ROM 
page 35 
Question: 11 
A 2-month-old infant has lost the vision in both of his eyes due to bilateral retinoblastoma. His 
distressed parents ask how the infant’s blindness will affect his behavior and development. 
Of the following, the child MOST likely will 
A. begin saying single words at 16 to 20 months 
B. begin walking between 18 and 22 months 
C. display behaviors of an autism spectrum disorder 
D. have a language-based learning disorder 
E. have significant cognitive impairments
2009 PREP SA on CD-ROM 
Critique: 11 Preferred Response: B 
Legal blindness is defined as central visual acuity with corrective lenses of 20/200 or less in the 
strongest eye or a limited visual field that extends to an angle of 20 degrees. Congenital blindness 
occurs in 30 per 100,000 births. More than 50% of children who have visual impairment also have 
developmental disabilities, such as cognitive-adaptive disability, seizures, hearing impairments, and 
learning disorders. In many of these cases, the disabilities result from central nervous system 
pathology. Postnatal blindness, which accounts for approximately 8% to 11% of all childhood blindness, 
can be caused by infections, trauma, or tumors. Retinoblastoma is the most common primary malignant 
intraocular tumor of childhood. The initial finding in most cases is a white pupillary reflex (leukokoria) 
(Item C11). Advanced tumors may be treated with enucleation. 
Children who have congenital or acquired (eg, due to retinoblastoma) blindness without associated 
neurologic abnormalities should not be at increased risk for motor or cognitive impairment. They are not 
at increased risk for language-based learning disabilities or autism spectrum disorders. However, 
children who have significant visual impairment may begin to walk at an older age (18 to 22 months) 
than sighted children due to different exposure to motor exploration. They typically develop language 
skills at the same time (12 months) as sighted children. Children who have visual impairments should 
be provided with much physical contact that includes hugging and comforting. They should be 
encouraged to partake in self-help skills and exploration of their environment. 
References: 
Davidson PW, Burns CM. Visual impairment and blindness. In: Levine MD, Carey WB, Crocker AC, 
eds. Developmental- Behavioral Pediatrics. 3rd ed. Philadelphia, Pa: WB Saunders Company; 
1999:571-578 
Msall ME. Visual impairment. In: Parker S, Zukerman B, Augustyn M. Developmental and Behavioral 
Pediatrics: A Handbook for Primary Care. 2nd ed. Philadelphia, Pa: Lippincott Williams & Wilkins; 
2005:366-369 
Olitsky SE, Hug D, Smith LP. Disorders of vision. In: Kleigman RM, Behrman RE, Jenson HB, Stanton 
BF, eds. Textbook of Pediatrics. 18th ed. Philadelphia, Pa: Saunders Elsevier; 2007:2573-2576 
page 36
2009 PREP SA on CD-ROM 
page 37 
Critique: 11 
Leukokoria may be observed in patients who have retinoblastoma. (Courtesy 
of R.G. Weaver, Jr)
2009 PREP SA on CD-ROM 
Question: 12 
A 15-year-old boy presents to the clinic because of a persistent cough. According to his mother, his 
cough has been present for approximately 2 weeks, but it seems to be getting worse. He does not 
cough all the time, but the coughing episodes tend to come in bursts. This morning she became very 
worried because he passed out during a coughing spell. Physical examination reveals a 
healthy-appearing male in no apparent distress. He is afebrile, and his vital signs are normal. He has 
petechiae on his face but no other skin lesions. His lungs are clear. 
Of the following, the MOST appropriate antimicrobial agent to prescribe for this patient is 
page 38 
A. azithromycin 
B. clarithromycin 
C. doxycycline 
D. erythromycin 
E. trimethoprim-sulfamethoxazole
2009 PREP SA on CD-ROM 
Critique: 12 Preferred Response: A 
Any patient who has episodic coughing episodes that conclude in syncope or vomiting, such as the boy 
described in the vignette, should be considered to have pertussis. Other complications from pertussis 
described in adolescents include urinary incontinence, sleep interruption, rib fractures, and pneumonia. 
Despite universal immunization of children against pertussis, a marked increase in disease incidence 
has been demonstrated among adolescents of 11 to 18 years of age. In an attempt to address this 
problem, the American Academy of Pediatrics recommends that adolescents in this age group receive 
a single dose of tetanus toxoid, reduced diphtheria toxoid, and acellular pertussis (Tdap) for booster 
immunization. For those who received only a tetanus toxoid and reduced diphtheria toxoid (Td) booster, 
a booster with Tdap is suggested if the interval since the Td administration is at least 2 years or if the 
adolescent is living in a setting of increased disease, has a risk of a complicated course if he or she 
acquires the disease, or possibly can transmit infection to a vulnerable contact. 
Treatment of pertussis with antimicrobial agents once the cough has started does not affect the 
course of the illness but is recommended to limit the spread of disease to others. In the past, the 
treatment of choice was erythromycin, but recent data have demonstrated that the effectiveness of 
other macrolides such as azithromycin or clarithromycin is similar to that of erythromycin, and they have 
fewer adverse effects. The most common complaints of patients who take erythromycin preparations 
are gastrointestinal irritation, including epigastric distress, abdominal cramping, nausea, vomiting, and 
diarrhea. The occurrence of these adverse effects can result in poor adherence to treatment regimens. 
In addition, erythromycin cannot be used in children younger than 1 month of age due to its association 
with the development of hypertrophic pyloric stenosis. Azithromycin and clarithromycin attain higher 
tissue concentrations than erythromycin, have longer half-lives, and can be administered less often and 
for a shorter total duration. With the advent of these newer macrolides, erythromycin rarely is indicated 
as a first-line agent for any illness. 
Azithromycin has become the agent of choice for treatment of pertussis because of the ease of 
administration (once daily for 5 days) and the fact that it does not inhibit cytochrome P-450, as 
erythromycin and clarithromycin do. Therefore, it does not interact with other medications that are 
metabolized by this system (eg, digoxin, carbamazepine). Trimethoprim-sulfamethoxazole also is 
effective against pertussis and is considered an alternative for patients who cannot tolerate a macrolide 
or have a macrolide-resistant isolate if they are older than 2 months of age. Doxycycline is not 
recommended for the treatment of pertussis. 
References: 
American Academy of Pediatrics. Pertussis (whooping cough). In: Pickering LK, Baker CJ, Long SS, 
McMillan JA, eds. Red Book: 2006 Report of the Committee on Infectious Diseases. 27th ed. Elk Grove 
Village, Ill: American Academy of Pediatrics; 2006:498-520 
Committee on Infectious Diseases. Prevention of pertussis among adolescents: recommendations for 
use of tetanus toxoid, reduced diphtheria toxoid, and acellular pertussis (Tdap) vaccine. Pediatrics. 
2006;117:965-978. Available at: http://pediatrics.aappublications.org/cgi/content/full/117/3/965 
Tiwari T, Murphy TV, Moran J. Recommended antimicrobial agents for the treatment and postexposure 
prophylaxis of pertussis: 2005 CDC guidelines. MMWR Recomm Rep. 2005;54(RR14):1-16. Available 
at: http://www.cdc.gov/mmwr/preview/mmwrhtml/rr5414a1.htm 
page 39
2009 PREP SA on CD-ROM 
Question: 13 
A 15-year-old male presents for evaluation of a progressively enlarging lesion on his left forearm that 
began 5 days ago. He explains that the lesion initially looked like a "spider bite" with a blister, but over 
the last several days, a black scab has developed in the center of the lesion, and there is a large area 
of redness around the scab. The lesion has been pruritic but not painful. Except for low-grade fevers for 
the last 2 days, he has had no other systemic symptoms. He returned 1 week ago from a school trip to 
Morocco, where he visited a leather tannery, went shopping in the large outdoor marketplace, visited 
some historic sites, and took a camel ride in the desert. He states that the students stayed in a hostel in 
Morocco, but there were no screens on the windows, and spiders, ants, and other insects were visible 
in the rooms. On physical examination, the boy is afebrile, and his left forearm is edematous, with a 
3x3-cm black eschar surrounded by a 5-cm area of erythema and induration (Item Q13). The lesion is 
not tender to palpation, and there is no drainage. There are several 1.5-cm tender lymph nodes in his 
left axilla. Findings on the remainder of his examination are within normal limits. 
Of the following, the MOST likely cause of this patient’s lesion is 
page 40 
A. Bacillus anthracis 
B. Francisella tularensis 
C. Loxosceles laeta 
D. methicillin-resistant Staphylococcus aureus 
E. Yersinia pestis
2009 PREP SA on CD-ROM 
page 41 
Question: 13 
Eschar, as desribed for the patient in the vignette. (Courtesy of the Centers for 
Disease Control and Prevention, Public Health Image Library, James H. Steele)
2009 PREP SA on CD-ROM 
Critique: 13 Preferred Response: A 
Anthrax is a zoonotic disease caused by Bacillus anthracis, which is a gram-positive, encapsulated, 
spore-forming rod that occurs in many areas of the world. B anthracis spores can remain viable in the 
soil for decades (Item C13A), representing a major reservoir of infection for herbivorous livestock 
through ingestion. Human infection occurs through contact with infected animals or contaminated 
animal products, including carcasses, hides, hair, wool, meat, bone meal, and other contaminated 
foodstuffs. 
page 42 
Depending on the route of infection, anthrax disease may manifest in three different forms: 
cutaneous, inhalational, and gastrointestinal. Approximately 95% of all human anthrax cases are 
cutaneous anthrax. The incubation period of cutaneous anthrax is 1 to 12 days. The initial skin lesion is 
a pruritic papule that resembles an insect or spider bite, as described for the boy in the vignette. The 
papule progresses to the development of a central vesicular or bullous lesion that becomes necrotic 
and hemorrhagic and forms a central black painless eschar, which is the classic lesion of anthrax (Item 
C13B). There is marked surrounding edema, swelling, induration, and erythema of the involved area but 
no associated tenderness. Regional lymphadenopathy, fever, malaise, and headache also may be 
present. In most cases, the eschar falls off in 1 to 2 weeks, and total resolution occurs in 6 weeks. 
Infections caused by methicillin-resistant Staphylococcus aureus (MRSA), the bite of Loxosceles 
laeta, Francisella tularensis, and Yersinia pestis are associated with pain, rapid progression, and in 
most cases, systemic symptoms. Skin and soft-tissue infections due to MRSA have been described as 
resembling a "spider bite" that is very tender, with progressive worsening of the erythema, warmth, and 
tenderness (Item C13C). It is not associated with the development of a bullous, necrotic lesion with 
eschar. The bite of Loxosceles laeta (the brown recluse spider) may resemble the cutaneous lesion of 
anthrax, but it is very painful (Item C13D). Also, symptoms develop shortly after the bite and progress 
rapidly within 24 hours. Francisella tularensis is the agent that causes tularemia. Signs and symptoms 
develop within 3 to 5 days of exposure and include the abrupt onset of fever, chills, headache, malaise, 
and fatigue as well as the development of a progressively enlarging, tender, localized lymphadenopathy 
and a red, painful papule (Item C13E) in a region draining into the involved lymph nodes. The signs and 
symptoms of Yersinia pestis (plague) develop abruptly and include fever; chills; weakness; headache; 
and extremely tender, rapidly enlarging swelling of the lymph nodes of the groin, axilla, or neck. 
References: 
American Academy of Pediatrics. Anthrax. In: Pickering LK, Baker CJ, Long SS, McMillan JA, eds. Red 
Book: 2006 Report of the Committee on Infectious Diseases. 27th ed. Elk Grove Village, Ill: American 
Academy of Pediatrics; 2006:208-211 
Butler T, Dennis DT. Yersinia species, including plague. In: Mandell GL, Bennett JE, Dolin R, eds. 
Mandell, Douglas and Bennett's Principles and Practice of Infectious Diseases. 6th ed. New York, NY: 
Elsevier Churchill Livingstone; 2005:2691-2700 
Inglesby TV, Henderson DA, Bartlett JG, et al. Anthrax as a biological weapon: medical and public 
health management. JAMA. 1999;281:1735-1745. Available at: 
http://jama.ama-assn.org/cgi/content/full/281/18/1735 
King MD, Humphrey BJ, Wang YF, Kourbatova EV, Ray SM, Blumberg HM. Emergence of 
community-acquired methicillin-resistant Staphylococcus aureus USA 300 clone as the predominant 
cause of skin and soft-tissue infections. Ann Intern Med. 2006;144:309-317. Abstract available at: 
http://www.ncbi.nlm.nih.gov/pubmed/16520471
2009 PREP SA on CD-ROM 
Penn RL. Francisella tularensis (tularemia). In: Mandell GL, Bennett JE, Dolin R, eds. Mandell, Douglas 
and Bennett's Principles and Practice of Infectious Diseases. 6th ed. New York, NY: Elsevier Churchill 
Livingstone; 2005:2674-2685 
Swanson DL, Vetter RS. Bites of brown recluse spiders and suspected necrotic arachnidism. N Engl J 
Med. 2005;352:700-707. Extract available at: http://content.nejm.org/cgi/content/extract/352/7/700 
page 43
2009 PREP SA on CD-ROM 
page 44 
Critique: 13 
Scanning electron micrograph at magnification 6408X demonstrates spores of 
Bacillus anthracis. The spores may surive in soil for decades. (Courtesy of 
the Centers for Disease Control and Prevention, Public Health Image Library, 
Laura Rose)
2009 PREP SA on CD-ROM 
page 45 
Critique: 13 
The classic lesion of cutaneous anthrax is a black eschar with surrounding 
swelling, erythema, and induration. The affected area is not painful. (Courtesy 
of the Centers for Disease Control and Prevention, Public Health Image 
Library)
2009 PREP SA on CD-ROM 
page 46 
Critique: 13 
Initial lesions of staphylococcal skin infection may be small tender papules 
that often are attributed to spider bites (yellow arrows). Frequently, lesions 
enlarge and develop an overlying pustule (black arrow). (Courtesy of Bernard 
Cohen, MD, DermAtlas; www.dermatlas.org)
2009 PREP SA on CD-ROM 
page 47 
Critique: 13 
An eschar with surrounding painful erythema and swelling is observed 
following the bite of a brown recluse spider. (Courtesy of M. Smith)
2009 PREP SA on CD-ROM 
page 48 
Critique: 13 
An ulcerated papule may appear at the site of inoculation of Francisella 
tularensis. (Courtesy of the Centers for Disease Control and Prevention, 
Public Health Image Library, Dr. Thomas F. Sellers)
2009 PREP SA on CD-ROM 
Question: 14 
A 3-month-old infant who has a history of renal dysplasia associated with obstructive uropathy has 
marked polyuria. He is breastfeeding and receiving supplemental cow milk-based formula. In an effort 
to reduce the high urine output, you consider reducing the renal solute load by changing feedings from 
the milk-based formula currently being used. 
Of the following, the MOST appropriate change is to 
page 49 
A. a hydrolyzed formula containing medium-chain triglycerides 
B. a more concentrated (24-kcal) milk-based formula 
C. human milk exclusively 
D. soy milk-based formula 
E. whole cow milk
2009 PREP SA on CD-ROM 
Critique: 14 Preferred Response: C 
The infant described in the vignette has polyuria caused by a urinary concentrating defect. The 
concentrating defect is the result of tubular damage due to the obstructive uropathy. The inability to 
concentrate the urine causes the kidneys to create an "excessive" volume of urine to excrete the solute 
load presented to them. 
One strategy to reduce polyuria is to reduce the solute burden placed on the kidneys. Potential renal 
solute load is affected by intake of protein, sodium, potassium, chloride, and phosphorus. The protein 
and phosphorus content are the most important variables when comparing infant feeding regimens. 
page 50 
Human milk possesses a lower potential renal solute load than cow milk or cow milk-based 
formulas. Accordingly, the most appropriate change in feeding for the infant in the vignette is to 
recommend that the mother stop cow milk formula supplementation and exclusively breastfeed. If 
human milk is not available, a "low-solute" cow milk-based formula can be used. A low 
calcium-phosphorus formula has the next lowest potential renal solute load compared with human milk. 
Cow milk, soy milk-based formula, hydrolyzed formula with medium-chain triglycerides, and 24-kcal 
milk-based formula all have greater renal solute loads than human milk. 
Renal solute load should also be considered in nephrogenic diabetes insipidus. 
References: 
Fiorino KN, Cox J. Nutrition and growth. In: Robertson J, Shilkofski N, eds. Harriet Lane Handbook: A 
Manual for Pediatric House Officers. 17th ed. Philadelphia, Pa: Elsevier Mosby; 2005:525-608 
Hall RT, Carroll RE. Infant feeding. Pediatr Rev. 2000;21:191-200. Available at: 
http://pedsinreview.aappublications.org/cgi/content/full/21/6/191 
Linshaw MA. Congenital nephrogenic diabetes insipidus. Pediatr Rev. 2007;28:372-380. Available at: 
http://pedsinreview.aappublications.org/cgi/content/full/28/10/372 
Ziegler EE, Fomon SJ. Potential renal solute load of infant formulas. J Nutr. 1989;119 (12 
suppl):1785-1788. Available at: http://jn.nutrition.org/cgi/reprint/119/12_Suppl/1785
2009 PREP SA on CD-ROM 
Question: 15 
A mother brings in her 13-month-old daughter for evaluation because her girl developed a perioral rash 
and "hives" on two occasions last week. One episode occurred while eating yogurt and another 
happened immediately after eating a bagel with cream cheese. She states that her daughter has eaten 
other foods such as eggs and bread without problems but is breastfeeding and never has been given 
milk-based formulas or cow milk. The infant has been given rice milk, but she became fussy and seems 
to prefer breastfeeding. The mother is concerned that her daughter may be allergic to milk but would 
like to stop breastfeeding. 
Of the following, the BEST advice is to recommend 
page 51 
A. a cow milk food challenge in the clinic 
B. avoidance of milk, egg, soy, and wheat products 
C. breastfeeding until the child is 3 years old 
D. switching to an amino acid-based formula 
E. switching to soy milk
2009 PREP SA on CD-ROM 
Critique: 15 Preferred Response: E 
Milk protein allergy is an immunoglobulin (Ig) E-mediated food reaction that affects 2% to 3% of infants 
within the first postnatal year. Typical symptoms include urticaria, angioedema, atopic dermatitis, and 
anaphylaxis. With IgE-mediated reactions, the quantity of milk required to result in a reaction often is 
minimal (eg, milk touching the face, a taste of ice cream). Taking a detailed history about the specific 
food(s) involved, timing of the onset of symptoms, and type of symptoms is important to distinguish 
IgE-mediated reactions, as described for the child in the vignette, from other adverse milk reactions, 
such as milk protein enterocolitis and lactose intolerance. Once an IgE-mediated food allergy is 
suspected, the clinician should consider allergy skin testing or serum IgE testing for the suspected food. 
page 52 
While awaiting results from either blood testing or allergy consultation for skin testing, the first 
reasonable action is to switch to a soy-based formula. Approximately 10% to 15% of infants and 
children who have IgE-mediated milk protein allergies may not tolerate soy formula, but this risk applies 
to infants younger than 6 months of age. For infants older than 6 months, the risk is closer to 5%. 
Nonetheless, the initial soy formula trial should be performed in the clinic. Other acceptable formula 
options in this scenario include an extensively hydrolyzed or an amino acid-based formula, although the 
unpleasant taste and significantly higher cost can be limiting for many families. Also, because almost all 
affected infants can be fed successfully with a soy or extensively hydrolyzed formula, switching initially 
to an amino acid-based formula is not required. 
Food challenges often are used to assess adverse food reactions, but they generally are reserved 
for foods that are unlikely allergens or if the clinical history is inconsistent or vague (eg, a patient who 
eats a particular food and does not always have a reaction). Food challenges may result in anaphylaxis 
and generally are avoided when the history and testing results support an IgE-mediated reaction. 
Infants who have a specific food allergy sometimes are placed incorrectly on restricted diets that 
avoid multiple foods. Without a specific history of other adverse food reactions, avoidance of other 
foods such as egg or wheat is not recommended. However, parents should be counseled that children 
can develop other food allergies and should monitor their children during ingestion of other common 
food allergens. 
Breastfeeding until age 3 years old is an option that is not preferred by the mother in the vignette. 
Although most IgE-mediated cow milk allergies resolve by 3 years of age, milk protein allergy can 
persist past 5 years of age in up to 20% of affected children. 
References: 
American Academy of Pediatrics Committee on Nutrition. Soy protein-based formulas: 
recommendations for use in infant feeding. Pediatrics. 1998;101:148-153. Available at: 
http://pediatrics.aappublications.org/cgi/content/full/101/1/148 
Bhatia J, Greer F, and the Committee on Nutrition. The use of soy protein-based formulas in infant 
feeding. Pediatrics. 2008;121:1062-1068. Available at: 
http://pediatrics.aappublications.org/cgi/content/full/121/5/1062 
Klemola T, Vanto T, Juntunen-Backman K, Kalimo K, Korpela R, Varjonen E. Allergy to soy formula and 
to extensively hydrolyzed whey formula in infants with cow's milk allergy: a prospective, randomized 
study with a follow-up to the age of 2 years. J Pediatr. 2002;140:219-224. Abstract available at: 
http://www.ncbi.nlm.nih.gov/ pubmed/11865274 
Saarinen KM, Pelkonen AS, Mäkelä MJ, Savilahti E. Clinical course and prognosis of cow's milk allergy 
are dependent on milk-specific IgE status. J Allergy Clin Immunol. 2005;116:869-875. Abstract
2009 PREP SA on CD-ROM 
available at: http://www.ncbi.nlm.nih.gov/pubmed/16210063 
Sampson HA, Leung DYM. Adverse reactions to foods. In: Kleigman RM, Behrman RE, Jenson HB, 
Stanton BF, eds. Nelson Textbook of Pediatrics. 18th ed. Philadelphia, Pa: Saunders Elsevier; 
2007:986-989 
page 53
2009 PREP SA on CD-ROM 
Question: 16 
A 16-year-old girl is brought to the emergency department after being found unresponsive in her 
bedroom. Her parents report finding a note in which she wrote of "wanting to end the pain." In addition, 
they found several empty, unlabeled pill vials on her dresser. On physical examination, the girl is 
responsive only to painful stimuli. Her heart rate is 60 beats/min, respiratory rate is 16 breaths/min, 
blood pressure is 90/60 mm Hg, and oxygen saturation is 92%. Her pupils are 3 mm, equal in size, and 
sluggishly reactive. The remainder of findings on her physical examination are normal. 
Of the following, the MOST important diagnostic test to obtain when evaluating this patient is a 
page 54 
A. carboxyhemoglobin concentration 
B. complete blood count 
C. serum acetaminophen concentration 
D. serum ammonia concentration 
E. serum osmolality
2009 PREP SA on CD-ROM 
Critique: 16 Preferred Response: C 
Treatment of a patient who has ingested an unknown substance or substances, such as the one 
described in the vignette, should focus initially on stabilization of vital functions; treatment of the 
patient's symptoms; and identification of agents that are potentially fatal, have delayed clinical toxicity, 
or for which antidotal therapy is indicated. The evaluation should include a screening history to identify 
circumstances surrounding the event, potential environmental exposures, available medications or 
toxins, and previous medical or psychiatric history. The physical examination should focus on vital sign 
abnormalities, pupillary size and reactivity, skin findings, and mental status. These components are 
likely to yield the most useful clues to toxins that have identifiable symptom complexes (Item C16A). 
Laboratory testing should assess the patient's acid/base status, oxygenation and ventilation, glucose 
concentration, and anion gap. Qualitative urine drug testing may be obtained but has significant 
limitations, including the small number of drugs tested (typically drugs of abuse) and screening 
thresholds that may produce false-negative results. Therefore, the results of urine toxicologic screening 
rarely affect treatment plans. Quantitative drug concentrations should be measured based on the 
information gleaned from the initial history, physical examination, and screening laboratory testing, 
although acetaminophen and salicylate concentrations should be measured for every patient who has 
an unknown, mixed, or intentional ingestion. Acetaminophen is of particular concern because it causes 
few initial symptoms, is a common agent in adolescent and adult intentional ingestions, and may 
require antidotal therapy to prevent potential fatal liver damage. Abdominal radiographs may be 
considered to examine the patient for radio-opaque substances (Item C16B). 
A complete blood count is not likely to provide information leading to the identification of a toxin. 
Measurement of serum ammonia may be indicated if acetaminophen is identified as the ingested agent 
because toxicity can lead to hepatic damage. Carboxyhemoglobin should be measured in the patient 
whose history is suggestive of carbon monoxide exposure. Increased serum osmolality may provide 
indirect evidence of alcohol ingestion, although a quantitative serum test for alcohols is more useful in 
this setting. 
References: 
Erikson TB, Thompson TM, Lu JJ. The approach to the patient with an unknown overdose. Emerg Med 
Clin North Am. 2007;25:249-281. Abstract available at: http://www.ncbi.nlm.nih.gov/pubmed/17482020 
Lavallee M, Olsson J Jr, Cheng TL. In brief: unknown poison. Pediatr Rev. 2004;25:370-371. Available 
at: http://pedsinreview.aappublications.org/cgi/content/full/25/10/370 
McKay CA Jr. Can the laboratory help me? Toxicology laboratory testing in the possibly poisoned 
pediatric patient. Clin Pediatr Emerg Med. 2005;6:116-122 
Valez LI, Shepherd JG, Goto CS. Approach to the child with occult toxic exposure. UpToDate Online 
15.3. 2008. Available for subscription at: 
http://www.utdol.com/utd/content/topic.do?topicKey=ped_tox/3023&selectedTitle=4~150&source=searc 
h_result 
page 55
2009 PREP SA on CD-ROM 
page 56 
Critique: 16
2009 PREP SA on CD-ROM 
page 57 
Critique: 16
2009 PREP SA on CD-ROM 
Question: 17 
You are following a 3-month-old infant who was born at 30 weeks’ gestation, underwent a distal ileal 
resection for necrotizing enterocolitis at 2 weeks of age, and subsequently was placed on parenteral 
nutrition for 2 months. The baby has residual cholestasis from the parenteral nutrition (total bilirubin, 5.0 
mg/dL [85.5 mcmol/L]; direct bilirubin, 3.0 mg/dL [51.3 mcmol/L]). Currently, she is receiving a cow milk 
protein hydrolysate formula concentrated to 24 kcal/oz (0.8 kcal/mL). You are considering adding a 
dietary supplement to increase the caloric density of the formula. 
Of the following, the supplement that is the MOST likely to be tolerated and cause less diarrhea in this 
infant is 
page 58 
A. flaxseed oil 
B. medium-chain triglyceride oil 
C. olive oil 
D. omega-3 polyunsaturated fatty acid (fish oil) 
E. soybean oil
2009 PREP SA on CD-ROM 
Critique: 17 Preferred Response: B 
Infants who have chronic illnesses may have specialized nutritional requirements and often do not 
tolerate the standard 20-kcal/oz formula given to healthy term infants. For example, children who have 
some forms of congenital heart disease or renal disease may require a more concentrated formula 
because the standard formula may lead to volume overload. Children who have intestinal disease or 
malabsorption, such as the child described in the vignette, also may require a more concentrated 
formula to decrease the likelihood of feeding intolerance or diarrhea. 
page 59 
Formula may be concentrated by increasing the concentration of protein, carbohydrate, or fat. 
Perhaps the easiest method of increasing the caloric density of a formula is to mix more powder with 
the same amount of water. For example, four scoops of most commercially available formulas mixed 
with 8 oz of water yields standard 0.67-kcal/mL (20-kcal/oz) formula, but mixing five scoops in 8 oz 
results in 0.83-kcal/mL (25-kcal/oz) formula. However, exceeding 25-kcal/oz formula by increasing the 
amount of powder may yield too high a concentration of protein, which could result in an excessive 
renal solute load. For this reason, carbohydrate and fat supplements are available to concentrate infant 
and toddler feedings further. The most common adverse effect of carbohydrate supplements is 
diarrhea, and lipid supplements may cause either diarrhea or delayed gastric emptying. For these 
reasons, caution is recommended when increasing the caloric density of a formula, especially when the 
caloric density is increased to greater than 1 kcal/mL (30 kcal/oz). 
Because the patient in the vignette has had an ileal resection and cholestasis, the best fat 
supplement for him is oil composed of medium-chain triglycerides (MCT oil). MCT oil can be absorbed 
directly across the enterocyte and does not require intraluminal digestion by bile acids. In contrast, soy, 
olive, flaxseed, and fish oils are long-chain fatty acids that require bile acids for digestion and might 
cause diarrhea in a child who has cholestasis and ileal resection. 
References: 
Courtney E, Grunko A, McCarthy T. Enteral nutrition. In: Hendricks KM, Duggan C. Manual of Pediatric 
Nutrition. 4th ed. Hamilton, Ontario, Canada: BC Decker; 2005:252-316 
Suchy FJ. Neonatal cholestasis. Pediatr Rev. 2004;25:388-396. Available at: 
http://pedsinreview.aappublications.org/cgi/content/full/25/11/388
2009 PREP SA on CD-ROM 
Question: 18 
You have admitted a 750-g male infant to the neonatal intensive care unit (NICU) for treatment of 
respiratory distress and presumed sepsis. The Apgar scores were 1, 5, and 7 at 1, 5, and 10 minutes, 
respectively. The infant received one dose of exogenous surfactant in the delivery room. In the NICU, 
the infant is being cared for on a radiant warmer. At 4 hours after birth, physical examination reveals a 
temperature of 97.0°F (36.1°C), heart rate of 180 beats/min, respiratory rate of 40 breaths/min (assisted 
breaths on the ventilator), blood pressure of 45/27 mm Hg, mean arterial blood pressure of 30 mm Hg, 
and pulse oximetry of 92%. The infant is receiving synchronized intermittent mechanical ventilation with 
a peak inflation pressure of 18 cm H2O over a positive end-expiratory pressure of 4 cm H2O at a rate of 
40 breaths/min and an FiO2 of 0.40. Umbilical catheters are present in the umbilical artery and vein. On 
physical examination, you note a soft, flat anterior fontanelle. You auscultate equal mechanical breath 
sounds bilaterally over the chest and note minimal subcostal retractions. The skin is thin and somewhat 
moist, and many veins are visible through it. The ears are flattened against the cranium and lack any 
cartilage or recoil. There is a small phallus and an empty scrotum. The infant is laying on the warmer 
with legs and arms extended. The neuromotor tone is decreased, and the infant does not appear to be 
very active, but he does respond to tactile stimuli with movement of the arms and legs in seemingly 
random and purposeless activity. 
Of the following, the MOST important next step is to 
page 60 
A. administer another dose of exogenous surfactant 
B. consult the urologist for cryptorchidism 
C. increase the inspired oxygen concentration 
D. move the infant into an isolette incubator 
E. obtain head ultrasonography to evaluate for any hemorrhage
2009 PREP SA on CD-ROM 
Critique: 18 Preferred Response: D 
The extremely low gestational-age newborn (ELGAN) whose birthweight is less than 1,000 g may 
present with physical examination findings indicative of marked physical and neurodevelopmental 
immaturity. These findings have been studied and correlated with gestational age in scoring systems 
such as those published by Dubowitz in 1980 and Ballard in 1979 and (revised) 1991. Even the best 
efforts to estimate the gestational age of very low-birthweight and immature infants at dates less than 
28 completed weeks of gestation by maternal last menstrual period and obstetric ultrasonography may 
be imprecise and tend to overestimate maturity (gestational age) by 2 weeks. Nevertheless, the thin, 
moist, transparent skin; flattened thin ears without cartilage or recoil; and small phallus and empty 
scrotum described for the infant in the vignette indicate a degree of physical immaturity consistent with 
28 weeks' gestation or less. Similarly, the low neuromotor tone, absence of flexed posture, relative 
inactivity, and random purposeless flailing of extremities with tactile stimulation indicate significant 
neurologic immaturity. 
Moving the infant into an isolette incubator is an important next step in the care of this ELGAN. The 
absolute benefit of using a radiant warmer bed (access to the infant to conduct early examinations, 
admission nursing care, and procedures such as placement of umbilical catheters) is eclipsed by its 
associated increased ambient convection, evaporative heat loss, and transcutaneous water loss 
compared with the contained, convectively heated, and humidified environment of the isolette. The 
isolette environment still allows access, visibility, and treatment with phototherapy, but with less 
insensible water loss or variability in patient temperature. 
This ELGAN is being treated for respiratory distress syndrome and has had a good response to 
exogenous surfactant administration, oxygen, and assisted ventilation. At 4 hours of postnatal age, it is 
too early to administer a second dose of surfactant. Developmentally cryptorchid testes may yet 
descend into the scrotal sac in coming weeks. An oxygen saturation of 92% is good for this infant, and 
increasing the inspired oxygen concentration may be harmful, contributing to oxidative injury of 
pulmonary tissues or retinopathy of prematurity. Cranial ultrasonography is indicated in this sick and 
preterm infant, but without clinical indications of hemorrhage, shock, and seizure or encephalopathy, 
such a study is performed best at 3 to 7 days of postnatal life. 
References: 
Donovan EF, Tyson JE, Ehrenkranz RA, et al. Inaccuracy of Ballard scores before 28 weeks' gestation. 
National Institute of Child Health and Human Development Neonatal Research Network. J Pediatr. 
1999;135:147-152. Abstract available at: http://www.ncbi.nlm.nih.gov/pubmed/10431107 
Marín GMA, Martín Moreiras J, Llitera Fleixas G, et al. Assessment of the new Ballard score to estimate 
gestational age [in Spanish]. An Pediatr (Barc). 2006;64:140-145. English abstract available at: 
http://www.ncbi.nlm.nih.gov/pubmed/16527066 
Sedin G. The thermal environment of the newborn infant. In: Martin RJ, Fanaroff AA, Walsh MC, eds. 
Fanaroff and Martin's Neonatal-Perinatal Medicine. 8th ed. Philadelphia, Pa: Mosby Elsevier; 
2006:585-596 
Thilo EH, Rosenberg AA. The newborn infant. In: Hay WW Jr, Levin MJ, Sondheimer JM, Deterding 
RR, eds. Current Pediatric Diagnosis & Treatment. 18th ed. New York, NY: The McGraw-Hill 
Companies, Inc; 2007:chap 1 
page 61
2009 PREP SA on CD-ROM 
Question: 19 
You are called to the newborn nursery to evaluate a 1-day-old girl whose hands and feet are blue. She 
was born at term via a cesarean section, and there were no complications. Apgar scores were 9 at both 
1 and 5 minutes. Her respiratory rate is 40 breaths/min, heart rate is 140 beats/min, and blood pressure 
is normal. Pulse oximetry is 98% on room air. Her lungs are clear, and there is no murmur. Her lips are 
pink, but her hands and feet are cyanotic (Item Q19), and capillary refill is less than 2 seconds. 
Of the following, the MOST likely cause of her peripheral cyanosis is 
page 62 
A. cold environment 
B. polycythemia 
C. retained fetal lung fluid 
D. sepsis 
E. transposition of the great arteries
2009 PREP SA on CD-ROM 
page 63 
Question: 19 
Cyanosis of the feet, as exhibited by the infant in the vignette. (Courtesy of the 
Media Lab at Doernbecher)
2009 PREP SA on CD-ROM 
Critique: 19 Preferred Response: A 
Cyanosis is a common finding in newborns. The first step in the evaluation of an infant who exhibits 
cyanosis is to determine whether it is central or peripheral (acrocyanosis). The infant described in the 
vignette has clinical features suggestive of acrocyanosis, a bluish discoloration of the hands and feet 
(Item C19A) in response to vasomotor instability or cold environment. In this condition, the lips and 
mucous membranes are spared, although the perioral area may be affected. The extremities may be 
cool to touch. Acrocyanosis is believed to be due to vasoconstriction of small arterioles and resolves in 
the first few postnatal months. Infants who have acrocyanosis require no further evaluation; parental 
reassurance is all that is needed. 
Central cyanosis is the bluish discoloration of the tissues best observed in the lips, tongue, mucous 
membranes, and occasionally nail beds (Item C19B). It occurs when the concentration of deoxygenated 
(reduced) hemoglobin exceeds 4.0 to 6.0 g/dL (40.0 to 60.0 g/L) within the capillary bed. In an individual 
who has normal hemoglobin, cyanosis usually becomes apparent when the oxygen saturation drops 
below 75%-80%. Causes of central cyanosis in the neonatal period typically are related to cyanotic 
heart disease, such as transposition of the great arteries, or respiratory disease, such as pneumonia 
and retained fetal lung fluid (transient tachypnea of the newborn). The absence of central cyanosis, a 
cardiac murmur, and respiratory distress for the infant in the vignette makes these diagnoses less likely 
than acrocyanosis of infancy. 
Polycythemia, defined as a venous hematocrit of at least 65% (0.65), causes a deep reddish-purple 
discoloration of the skin and mucous membranes. Infants may be otherwise asymptomatic or may 
exhibit lethargy, anorexia, or respiratory distress. Polycythemia does not cause either peripheral or 
central cyanosis per se, but it does affect the percent saturation at which cyanosis becomes clinically 
apparent. Cyanosis in infants who have polycythemia occurs at higher oxygen saturations than in those 
who have anemia. For example, if an infant has a hemoglobin of 24.0 g/dL (240.0 g/L), he is likely to 
appear cyanotic when his oxygen saturation reaches 87%. In an infant who has a hemoglobin of 12.0 
g/dL (120.0 g/L), cyanosis may not be seen until the oxygen saturation falls to 75%. Newborns who 
have sepsis may develop cyanosis as a component of multisystem involvement, but other signs, such 
as respiratory distress, poor peripheral perfusion, lethargy, or poor feeding, also are evident. 
References: 
Bernstein D. Evaluation of the cardiovascular system: history and physical examination. In: Kliegman 
RM, Behrman RE, Jenson HB, Stanton BF, eds. Nelson Textbook of Pediatrics. 18th ed. Philadelphia, 
Pa: Saunders Elsevier; 2007:1857-1863 
Sasidharan P. An approach to diagnosis and management of cyanosis and tachypnea in term infants. 
Pediatr Clin North Am. 2004;51:999-1021. Abstract available at: 
http://www.ncbi.nlm.nih.gov/pubmed/15275985 
page 64
2009 PREP SA on CD-ROM 
page 65 
Critique: 19 
Acrocyanosis is a blue discoloration of the hands and feet. (Courtesy of the 
Media Lab at Doernbecher)
2009 PREP SA on CD-ROM 
page 66 
Critique: 19 
Central cyanosis, as exhibited by this infant who has tetralogy of Fallot, is 
observed best in highly vascularized areas, such as the lips, tongue, mucous 
membranes, and nail beds. (Courtesy of M. Rimsza)
2009 PREP SA on CD-ROM 
Question: 20 
A frustrated mother requests referral of her 15-month-old child to an allergy and asthma specialist 
because the boy never seems to have stopped coughing and wheezing over the 6 months of the past 
respiratory virus season. During the history taking, the mother states that the baby only occasionally is 
exposed to wood smoke at the family’s barbecue restaurant and to cosmetic chemicals used at the 
grandmother’s hair salon. Both the father and grandfather smoke cigarettes in the home. 
Of the following, the environmental exposure that is MOST likely to be causing the child’s respiratory 
symptoms is exposure to 
page 67 
A. cigarette smoke 
B. cleaning fluids 
C. dust mites 
D. hairspray 
E. wood smoke
2009 PREP SA on CD-ROM 
Critique: 20 Preferred Response: A 
The child described in the vignette may have multiple triggers for his wheezing exacerbations, although 
tobacco smoke probably is the one to which he is exposed most often that should be diminished. 
Well-known tobacco toxins include nicotine, carbon monoxide, formaldehyde, hydrogen cyanide, sulfur 
dioxide, nitrogen oxide, ammonia, polycyclic aromatic hydrocarbons, and the nitrosamines. These 
substances produce both irritant and immunologic effects on the respiratory tract. It is important to note 
that smoking cessation is difficult, and most pediatricians have little training in the process. However, 
there is evidence that a brief discussion of smoking cessation in the context of medical visits is 
effective. 
Wood smoke is another significant irritant to the respiratory tract; in addition to chemical irritants, 
wood smoke contains particulate matter. Assuming that the family's restaurant kitchen meets current 
standards for air filtration/exhaust, the child's exposure should be minimal. However, outdoor barbecue 
pits that discharge smoke directly into the environment should be avoided. 
Exposure to cooking spray/oils at the restaurant and cosmetic chemicals such as hairspray at the 
salon also should be avoided, although the use of volatile organic compounds, including 
chlorofluorocarbons in hairspray, permanent wave solution, cleaning solutions, and cooking sprays, has 
decreased sharply since passage of the Clean Air Act and other Environmental Protection Agency 
measures in the 1990s. Material Safety Data Sheets for most hairsprays, as well as other cosmetics 
and cleaning fluids, reveal little inhalation toxicity, except in the case of prolonged exposure under 
improper ventilation. 
Although dust mite exposure may contribute to asthma exacerbation, there is no evidence that 
environmental control can reduce the concentration of mites or exacerbations related to them. Finally, 
serial respiratory virus infections themselves may contribute significantly to this child's history of 
wheezing and cough. 
The child in the vignette may benefit from therapeutic advances in asthma management such as 
those contained in the National Heart, Lung and Blood Institute Guidelines for asthma, including 
attention to number of symptomatic days, use of inhaled corticosteroids, and use of a metered dose 
inhaler with spacer for proper delivery of medication, which may be more effective than nebulization in 
older infants and children. 
References: 
Brunnhuber K, Cummings KM, Feit S, Sherman S, Woodcock J.Putting evidence into practice: smoking 
cessation. BMJ Clinical Evidence. 2007. Available for subscription at: 
http://clinicalevidence.bmj.com/ceweb/resources/index.jsp 
Kum-Nji P, Meloy L, Herrod HG. Environmental tobacco smoke exposure: prevalence and mechanisms 
of causation of infections in children. Pediatrics. 2006;117:5:1745-1754. Available at: 
http://pediatrics.aappublications.org/cgi/content/full/117/5/1745 
Roseby R, Waters E, Polnay A, Campbell R, Webster P, Spencer N. Family and carer smoking control 
programmes for reducing children's exposure to environmental tobacco smoke. Cochrane Database 
Syst Rev. 2003;3:CD001746. Available at: http://www.cochrane.org/reviews/en/ab001746.html 
Stein RT, Holberg CJ, Sherrill D, et al. Influence of parental smoking on respiratory symptoms during 
the first decade of life: The Tucson Children's Respiratory Study. Am J Epidemiol. 1999;149:1030-1037. 
Available at: http://aje.oxfordjournals.org/cgi/reprint/149/11/1030 
page 68
2009 PREP SA on CD-ROM 
U.S. Environmental Protection Agency. Health effects of wood smoke. Available at: 
http://www.epa.gov/woodstoves/healtheffects.html 
U.S. Environmental Protection Agency. National volatile organic compound emission standards for 
consumer products. Available at: 
http://www.epa.gov/fedrgstr/EPA-AIR/1998/September/Day-11/a22660.htm 
page 69
2009 PREP SA on CD-ROM 
Question: 21 
You are evaluating a newborn 6 hours after his birth. Labor and delivery were uncomplicated, but 
amniocentesis performed during the pregnancy revealed trisomy 21. Fetal echocardiography at 20 
weeks’ gestation showed normal findings. The infant currently is sleeping and is well-perfused, with a 
heart rate of 140 beats/min and no audible murmurs. His physical features are consistent with Down 
syndrome. 
Of the following, the MOST appropriate diagnostic study to perform is 
page 70 
A. barium swallow 
B. cervical spine radiography 
C. echocardiography 
D. head ultrasonography 
E. radiography of the abdomen
2009 PREP SA on CD-ROM 
Critique: 21 Preferred Response: C 
Congenital heart defects (CHDs) are the most common of the congenital anomalies, occurring with an 
incidence of approximately 5 to 8 per 1,000 live births (0.5% to 0.8%). The incidence of CHD is greater 
in stillbirths and there is an increased incidence of CHD in those who have aneuploidy. It is believed by 
some that the abnormal chromosomal composition rather than the cardiac abnormality is responsible 
for the fetal demise of those who have CHDs. The strong association between chromosomal 
abnormality and CHD has been shown in a number of studies that focus on identification of fetal cardiac 
abnormality. A number of well-defined chromosomal anomalies are associated with CHD (Item C21). 
page 71 
Early and accurate diagnosis of CHD is important in counseling parents of children in whom a 
chromosomal abnormality or syndrome is suspected. Such diagnoses may have significant effects on 
the health and well-being of the newborns because some CHDs may require ductal patency for 
perfusion of either the systemic or pulmonary circulation. Therefore, echocardiography now is 
considered an important component of the routine health supervision of infants who have Down 
syndrome and other syndromes predisposing to structural heart defects. 
Although results of fetal echocardiography for the infant in the vignette were interpreted as normal, 
not all CHDs can be diagnosed routinely with fetal echocardiography because of the shunting pathways 
of the fetal circulation. For example, persistent patency of the ductus arteriosus and secundum atrial 
septal defects are diagnosed postnatally. Small ventricular septal defects frequently are not seen during 
fetal echocardiography because the pressure in the right and left ventricles are equal due to the ductus 
arteriosus, resulting in minimal flow across the defect prenatally. 
Given the strong association between Down syndrome and CHD, echocardiography should be 
performed in the newborn described in the vignette. Although children born with Down syndrome have 
an increased incidence of duodenal atresia and other types of gastrointestinal obstruction, a barium 
swallow is not an appropriate initial test for an asymptomatic newborn. Cervical radiography, beginning 
at age 3 years, is important because of the risk of atlantoaxial (C1-C2) subluxation. There is no 
indication for routine head ultrasonography or abdominal radiography in an otherwise asymptomatic 
newborn who has Down syndrome. 
References: 
Committee on Genetics. Health supervision for children with Down syndrome. Pediatrics. 
2001;107:442-449. Available at: http://pediatrics.aappublications.org/cgi/content/full/107/2/442 
Silberbach M, Hannon D. Presentation of congenital heart disease in the neonate and young infant. 
Pediatr Rev. 2007;28:123-131. Available at: 
http://pedsinreview.aappublications.org/cgi/content/full/28/4/123 
Tennstedt C, Chaoui R, Körner H, Dietel M. Spectrum of congenital heart defects and extracardiac 
malformations associated with chromosomal abnormalities: results of a seven year necropsy study. 
Heart. 1999;82:34-39. Abstract available at: http://www.ncbi.nlm.nih.gov/pubmed/10377306
2009 PREP SA on CD-ROM 
page 72 
Critique: 21
2009 PREP SA on CD-ROM 
Question: 22 
A 4-year-old boy presents with headache and difficulty walking. On physical examination, he is afebrile, 
all growth parameters are within normal limits, and his mentation appears normal. The optic discs are 
clearly visible and appear normal. He has normal eye position in primary gaze but cannot abduct his 
right eye fully. He has normal tone, strength, and reflexes in his upper limbs, but has bilateral 
hyperreflexia at the knees and ankle clonus. On gait examination, he toe-walks. 
Of the following, the MOST important next step is to obtain 
page 73 
A. computed tomography scan of the head 
B. electromyography/nerve conduction studies of the legs 
C. lumbar puncture 
D. magnetic resonance imaging of the thoracolumbar spine 
E. visual evoked potentials
2009 PREP SA on CD-ROM 
Critique: 22 Preferred Response: A 
The boy described in the vignette presents with a very concerning constellation of symptoms and signs 
requiring urgent evaluation of the central nervous system. The first step in the diagnostic process is to 
localize the problem to the proper level of the nervous system: brain, brainstem/cerebellum, spinal cord, 
nerve, junction, or muscle. Such a determination not only allows for urgent appropriate diagnosis and 
treatment but also reduces unnecessary discomfort, risks, and costs of inappropriate diagnostic testing. 
Consultation with a neurologist can be useful before ordering testing. Headache, gait disturbance 
(toe-walking), and acquired ocular misalignment (right eye cannot move to the right) localizes the 
problem to the central nervous system and raises concerns for both hydrocephalus and a brainstem 
lesion. 
page 74 
Computed tomography (CT) scan of the head is the preferred neuroimaging technique for this 
patient. Although brain magnetic resonance imaging (MRI) can document the brainstem/posterior fossa 
better, it may not be readily available. Moreover, a head CT scan in the emergency department is 
adequate to rule out hydrocephalus that requires emergent neurosurgical consultation. Unfortunately, a 
common cause for hydrocephalus in a child of this age is a brainstem or cerebellar neoplasm such as 
an astrocytoma, glioma, medulloblastoma, or ependymoma. When these tumors enlarge in the 
posterior fossa, they can obstruct the flow of cerebrospinal fluid and cause acute hydrocephalus, which 
is a neurosurgical emergency. 
Primary headache disorders such as migraine do not often present before the age of 6 years and 
are not accompanied by the neurologic findings described in the vignette. Toe-walking is a nonspecific 
finding that can occur in the context of relatively benign delayed development, although if this 
represents a change for a child, an upper motor neuron (brain/spinal cord) lesion must be ruled out. The 
ankle clonus described for the boy in the vignette mandates this approach. Similarly, inability to abduct 
one eye can be congenital, but if this is a new finding, a brain or brainstem lesion must be ruled out. In 
this case, headache and the oculomotor findings make a brain or brainstem lesion likely. 
Gait impairment and hyperreflexia only in the legs can result from hydrocephalus or can localize to 
the mid- to lower spinal cord. A spinal cord lesion would not explain the headache, and bilateral motor 
findings without sensory, bowel, or bladder involvement are uncommon for a spinal cord lesion. 
Therefore, MRI of the spine is not needed. Similarly, the hyperreflexia is not evidence of a nerve or 
muscle problem, obviating the need for electromyography and nerve conduction studies. Lumbar 
puncture is therapeutic for headache due to pseudotumor cerebri, which could present with this 
constellation of symptoms, but this degree of gait abnormality would be uncommon in pseudotumor, 
and the presence of true hydrocephalus or an intracranial mass must be excluded with a head CT or 
brain MRI prior to lumbar puncture. Visual evoked potentials occasionally are used to determine 
whether the axons from eye to occipital cortex function normally, but such a test generally is performed 
when demyelinating disorders are suspected. 
References: 
Avellino AM. Hydrocephalus. In: Singer HS, Kossoff EH, Hartman AL, Crawford TO, eds. Treatment of 
Pediatric Neurologic Disorders. Boca Raton, Fla: Taylor & Francis; 2005:25-36 
Garton HJ, Piatt JH Jr. Hydrocephalus. Pediatr Clin North Am. 2004;51:305-325. Abstract available at: 
http://www.ncbi.nlm.nih.gov/pubmed/15062673 
Kestle JR. Pediatric hydrocephalus: current management. Neurol Clin. 2003;21:883-895. Abstract 
available at: http://www.ncbi.nlm.nih.gov/pubmed/14743654
2009 PREP SA on CD-ROM 
Kuttesch J Jr, Ater JL. Brain tumors in childhood. In: Behrman RE, Kliegman RM, Jenson HB, Stanton 
BF, eds. Nelson Textbook of Pediatrics. 18th ed. Philadelphia, Pa: Saunders Elsevier; 2007:2128-2136 
Piatt JH Jr. Recognizing neurosurgical conditions in the pediatrician's office. Pediatr Clin North Am. 
2004;51:237-270. Abstract available at: http://www.ncbi.nlm.nih.gov/pubmed/15062671 
page 75
2009 PREP SA on CD-ROM 
Question: 23 
Parents who are new to your area bring in their 3-year-old daughter for evaluation because they are 
concerned about her delayed speech. They say that she uses about 50 single words. The girl has had 
tetralogy of Fallot repaired surgically and recurrent upper respiratory tract infections with otitis media, 
for which tympanostomy tubes have been placed. Findings on physical examination include 
microcephaly, underfolded pinnae, a broad nasal bridge, cleft uvula, and a small chin. In addition, the 
child’s speech has a hypernasal quality. The family history is negative for birth defects and 
developmental delays. 
Of the following, the contiguous gene deletion syndrome that BEST fits this child’s features is 
page 76 
A. Angelman 
B. Beckwith-Wiedemann 
C. 4p- 
D. Prader-Willi 
E. 22q11
AAP PREP 2009
AAP PREP 2009
AAP PREP 2009
AAP PREP 2009
AAP PREP 2009
AAP PREP 2009
AAP PREP 2009
AAP PREP 2009
AAP PREP 2009
AAP PREP 2009
AAP PREP 2009
AAP PREP 2009
AAP PREP 2009
AAP PREP 2009
AAP PREP 2009
AAP PREP 2009
AAP PREP 2009
AAP PREP 2009
AAP PREP 2009
AAP PREP 2009
AAP PREP 2009
AAP PREP 2009
AAP PREP 2009
AAP PREP 2009
AAP PREP 2009
AAP PREP 2009
AAP PREP 2009
AAP PREP 2009
AAP PREP 2009
AAP PREP 2009
AAP PREP 2009
AAP PREP 2009
AAP PREP 2009
AAP PREP 2009
AAP PREP 2009
AAP PREP 2009
AAP PREP 2009
AAP PREP 2009
AAP PREP 2009
AAP PREP 2009
AAP PREP 2009
AAP PREP 2009
AAP PREP 2009
AAP PREP 2009
AAP PREP 2009
AAP PREP 2009
AAP PREP 2009
AAP PREP 2009
AAP PREP 2009
AAP PREP 2009
AAP PREP 2009
AAP PREP 2009
AAP PREP 2009
AAP PREP 2009
AAP PREP 2009
AAP PREP 2009
AAP PREP 2009
AAP PREP 2009
AAP PREP 2009
AAP PREP 2009
AAP PREP 2009
AAP PREP 2009
AAP PREP 2009
AAP PREP 2009
AAP PREP 2009
AAP PREP 2009
AAP PREP 2009
AAP PREP 2009
AAP PREP 2009
AAP PREP 2009
AAP PREP 2009
AAP PREP 2009
AAP PREP 2009
AAP PREP 2009
AAP PREP 2009
AAP PREP 2009
AAP PREP 2009
AAP PREP 2009
AAP PREP 2009
AAP PREP 2009
AAP PREP 2009
AAP PREP 2009
AAP PREP 2009
AAP PREP 2009
AAP PREP 2009
AAP PREP 2009
AAP PREP 2009
AAP PREP 2009
AAP PREP 2009
AAP PREP 2009
AAP PREP 2009
AAP PREP 2009
AAP PREP 2009
AAP PREP 2009
AAP PREP 2009
AAP PREP 2009
AAP PREP 2009
AAP PREP 2009
AAP PREP 2009
AAP PREP 2009
AAP PREP 2009
AAP PREP 2009
AAP PREP 2009
AAP PREP 2009
AAP PREP 2009
AAP PREP 2009
AAP PREP 2009
AAP PREP 2009
AAP PREP 2009
AAP PREP 2009
AAP PREP 2009
AAP PREP 2009
AAP PREP 2009
AAP PREP 2009
AAP PREP 2009
AAP PREP 2009
AAP PREP 2009
AAP PREP 2009
AAP PREP 2009
AAP PREP 2009
AAP PREP 2009
AAP PREP 2009
AAP PREP 2009
AAP PREP 2009
AAP PREP 2009
AAP PREP 2009
AAP PREP 2009
AAP PREP 2009
AAP PREP 2009
AAP PREP 2009
AAP PREP 2009
AAP PREP 2009
AAP PREP 2009
AAP PREP 2009
AAP PREP 2009
AAP PREP 2009
AAP PREP 2009
AAP PREP 2009
AAP PREP 2009
AAP PREP 2009
AAP PREP 2009
AAP PREP 2009
AAP PREP 2009
AAP PREP 2009
AAP PREP 2009
AAP PREP 2009
AAP PREP 2009
AAP PREP 2009
AAP PREP 2009
AAP PREP 2009
AAP PREP 2009
AAP PREP 2009
AAP PREP 2009
AAP PREP 2009
AAP PREP 2009
AAP PREP 2009
AAP PREP 2009
AAP PREP 2009
AAP PREP 2009
AAP PREP 2009
AAP PREP 2009
AAP PREP 2009
AAP PREP 2009
AAP PREP 2009
AAP PREP 2009
AAP PREP 2009
AAP PREP 2009
AAP PREP 2009
AAP PREP 2009
AAP PREP 2009
AAP PREP 2009
AAP PREP 2009
AAP PREP 2009
AAP PREP 2009
AAP PREP 2009
AAP PREP 2009
AAP PREP 2009
AAP PREP 2009
AAP PREP 2009
AAP PREP 2009
AAP PREP 2009
AAP PREP 2009
AAP PREP 2009
AAP PREP 2009
AAP PREP 2009
AAP PREP 2009
AAP PREP 2009
AAP PREP 2009
AAP PREP 2009
AAP PREP 2009
AAP PREP 2009
AAP PREP 2009
AAP PREP 2009
AAP PREP 2009
AAP PREP 2009
AAP PREP 2009
AAP PREP 2009
AAP PREP 2009
AAP PREP 2009
AAP PREP 2009
AAP PREP 2009
AAP PREP 2009
AAP PREP 2009
AAP PREP 2009
AAP PREP 2009
AAP PREP 2009
AAP PREP 2009
AAP PREP 2009
AAP PREP 2009
AAP PREP 2009
AAP PREP 2009
AAP PREP 2009
AAP PREP 2009
AAP PREP 2009
AAP PREP 2009
AAP PREP 2009
AAP PREP 2009
AAP PREP 2009
AAP PREP 2009
AAP PREP 2009
AAP PREP 2009
AAP PREP 2009
AAP PREP 2009
AAP PREP 2009
AAP PREP 2009
AAP PREP 2009
AAP PREP 2009
AAP PREP 2009
AAP PREP 2009
AAP PREP 2009
AAP PREP 2009
AAP PREP 2009
AAP PREP 2009
AAP PREP 2009
AAP PREP 2009
AAP PREP 2009
AAP PREP 2009
AAP PREP 2009
AAP PREP 2009
AAP PREP 2009
AAP PREP 2009
AAP PREP 2009
AAP PREP 2009
AAP PREP 2009
AAP PREP 2009
AAP PREP 2009
AAP PREP 2009
AAP PREP 2009
AAP PREP 2009
AAP PREP 2009
AAP PREP 2009
AAP PREP 2009
AAP PREP 2009
AAP PREP 2009
AAP PREP 2009
AAP PREP 2009
AAP PREP 2009
AAP PREP 2009
AAP PREP 2009
AAP PREP 2009
AAP PREP 2009
AAP PREP 2009
AAP PREP 2009
AAP PREP 2009
AAP PREP 2009
AAP PREP 2009
AAP PREP 2009
AAP PREP 2009
AAP PREP 2009
AAP PREP 2009
AAP PREP 2009
AAP PREP 2009
AAP PREP 2009
AAP PREP 2009
AAP PREP 2009
AAP PREP 2009
AAP PREP 2009
AAP PREP 2009
AAP PREP 2009
AAP PREP 2009
AAP PREP 2009
AAP PREP 2009
AAP PREP 2009
AAP PREP 2009
AAP PREP 2009
AAP PREP 2009
AAP PREP 2009
AAP PREP 2009
AAP PREP 2009
AAP PREP 2009
AAP PREP 2009
AAP PREP 2009
AAP PREP 2009
AAP PREP 2009
AAP PREP 2009
AAP PREP 2009
AAP PREP 2009
AAP PREP 2009
AAP PREP 2009
AAP PREP 2009
AAP PREP 2009
AAP PREP 2009
AAP PREP 2009
AAP PREP 2009
AAP PREP 2009
AAP PREP 2009
AAP PREP 2009
AAP PREP 2009
AAP PREP 2009
AAP PREP 2009
AAP PREP 2009
AAP PREP 2009
AAP PREP 2009
AAP PREP 2009
AAP PREP 2009
AAP PREP 2009
AAP PREP 2009
AAP PREP 2009
AAP PREP 2009
AAP PREP 2009
AAP PREP 2009
AAP PREP 2009
AAP PREP 2009
AAP PREP 2009
AAP PREP 2009
AAP PREP 2009
AAP PREP 2009
AAP PREP 2009
AAP PREP 2009
AAP PREP 2009
AAP PREP 2009
AAP PREP 2009
AAP PREP 2009
AAP PREP 2009
AAP PREP 2009
AAP PREP 2009
AAP PREP 2009
AAP PREP 2009
AAP PREP 2009
AAP PREP 2009
AAP PREP 2009
AAP PREP 2009
AAP PREP 2009
AAP PREP 2009
AAP PREP 2009
AAP PREP 2009
AAP PREP 2009
AAP PREP 2009
AAP PREP 2009
AAP PREP 2009
AAP PREP 2009
AAP PREP 2009
AAP PREP 2009
AAP PREP 2009
AAP PREP 2009
AAP PREP 2009
AAP PREP 2009
AAP PREP 2009
AAP PREP 2009
AAP PREP 2009
AAP PREP 2009
AAP PREP 2009
AAP PREP 2009
AAP PREP 2009
AAP PREP 2009
AAP PREP 2009
AAP PREP 2009
AAP PREP 2009
AAP PREP 2009
AAP PREP 2009
AAP PREP 2009
AAP PREP 2009
AAP PREP 2009
AAP PREP 2009
AAP PREP 2009
AAP PREP 2009
AAP PREP 2009
AAP PREP 2009
AAP PREP 2009
AAP PREP 2009
AAP PREP 2009
AAP PREP 2009
AAP PREP 2009
AAP PREP 2009
AAP PREP 2009
AAP PREP 2009
AAP PREP 2009
AAP PREP 2009
AAP PREP 2009
AAP PREP 2009
AAP PREP 2009
AAP PREP 2009
AAP PREP 2009
AAP PREP 2009
AAP PREP 2009
AAP PREP 2009
AAP PREP 2009
AAP PREP 2009
AAP PREP 2009
AAP PREP 2009
AAP PREP 2009
AAP PREP 2009
AAP PREP 2009
AAP PREP 2009
AAP PREP 2009
AAP PREP 2009
AAP PREP 2009
AAP PREP 2009
AAP PREP 2009
AAP PREP 2009
AAP PREP 2009
AAP PREP 2009
AAP PREP 2009
AAP PREP 2009
AAP PREP 2009
AAP PREP 2009
AAP PREP 2009
AAP PREP 2009
AAP PREP 2009
AAP PREP 2009
AAP PREP 2009
AAP PREP 2009
AAP PREP 2009
AAP PREP 2009
AAP PREP 2009
AAP PREP 2009
AAP PREP 2009
AAP PREP 2009
AAP PREP 2009
AAP PREP 2009
AAP PREP 2009
AAP PREP 2009
AAP PREP 2009
AAP PREP 2009
AAP PREP 2009
AAP PREP 2009
AAP PREP 2009
AAP PREP 2009
AAP PREP 2009
AAP PREP 2009
AAP PREP 2009
AAP PREP 2009
AAP PREP 2009
AAP PREP 2009
AAP PREP 2009
AAP PREP 2009
AAP PREP 2009
AAP PREP 2009
AAP PREP 2009
AAP PREP 2009
AAP PREP 2009
AAP PREP 2009
AAP PREP 2009
AAP PREP 2009
AAP PREP 2009
AAP PREP 2009
AAP PREP 2009
AAP PREP 2009
AAP PREP 2009
AAP PREP 2009
AAP PREP 2009
AAP PREP 2009
AAP PREP 2009
AAP PREP 2009
AAP PREP 2009
AAP PREP 2009
AAP PREP 2009
AAP PREP 2009
AAP PREP 2009
AAP PREP 2009
AAP PREP 2009
AAP PREP 2009
AAP PREP 2009
AAP PREP 2009
AAP PREP 2009
AAP PREP 2009
AAP PREP 2009
AAP PREP 2009
AAP PREP 2009
AAP PREP 2009
AAP PREP 2009
AAP PREP 2009
AAP PREP 2009
AAP PREP 2009
AAP PREP 2009
AAP PREP 2009
AAP PREP 2009
AAP PREP 2009
AAP PREP 2009
AAP PREP 2009
AAP PREP 2009
AAP PREP 2009
AAP PREP 2009
AAP PREP 2009
AAP PREP 2009
AAP PREP 2009
AAP PREP 2009
AAP PREP 2009
AAP PREP 2009
AAP PREP 2009
AAP PREP 2009
AAP PREP 2009
AAP PREP 2009
AAP PREP 2009
AAP PREP 2009
AAP PREP 2009
AAP PREP 2009
AAP PREP 2009
AAP PREP 2009
AAP PREP 2009
AAP PREP 2009
AAP PREP 2009
AAP PREP 2009
AAP PREP 2009
AAP PREP 2009
AAP PREP 2009
AAP PREP 2009
AAP PREP 2009
AAP PREP 2009
AAP PREP 2009
AAP PREP 2009
AAP PREP 2009
AAP PREP 2009
AAP PREP 2009
AAP PREP 2009
AAP PREP 2009
AAP PREP 2009
AAP PREP 2009
AAP PREP 2009
AAP PREP 2009
AAP PREP 2009
AAP PREP 2009
AAP PREP 2009
AAP PREP 2009
AAP PREP 2009
AAP PREP 2009
AAP PREP 2009
AAP PREP 2009
AAP PREP 2009
AAP PREP 2009
AAP PREP 2009
AAP PREP 2009
AAP PREP 2009
AAP PREP 2009
AAP PREP 2009
AAP PREP 2009
AAP PREP 2009
AAP PREP 2009
AAP PREP 2009
AAP PREP 2009
AAP PREP 2009
AAP PREP 2009
AAP PREP 2009
AAP PREP 2009
AAP PREP 2009
AAP PREP 2009
AAP PREP 2009
AAP PREP 2009
AAP PREP 2009
AAP PREP 2009
AAP PREP 2009
AAP PREP 2009
AAP PREP 2009
AAP PREP 2009
AAP PREP 2009
AAP PREP 2009
AAP PREP 2009
AAP PREP 2009
AAP PREP 2009
AAP PREP 2009
AAP PREP 2009
AAP PREP 2009
AAP PREP 2009
AAP PREP 2009
AAP PREP 2009
AAP PREP 2009
AAP PREP 2009
AAP PREP 2009
AAP PREP 2009
AAP PREP 2009
AAP PREP 2009
AAP PREP 2009
AAP PREP 2009
AAP PREP 2009
AAP PREP 2009
AAP PREP 2009
AAP PREP 2009
AAP PREP 2009
AAP PREP 2009
AAP PREP 2009
AAP PREP 2009
AAP PREP 2009
AAP PREP 2009
AAP PREP 2009
AAP PREP 2009
AAP PREP 2009
AAP PREP 2009
AAP PREP 2009
AAP PREP 2009
AAP PREP 2009
AAP PREP 2009
AAP PREP 2009
AAP PREP 2009
AAP PREP 2009
AAP PREP 2009
AAP PREP 2009
AAP PREP 2009
AAP PREP 2009
AAP PREP 2009
AAP PREP 2009
AAP PREP 2009
AAP PREP 2009
AAP PREP 2009
AAP PREP 2009
AAP PREP 2009
AAP PREP 2009
AAP PREP 2009
AAP PREP 2009
AAP PREP 2009
AAP PREP 2009
AAP PREP 2009
AAP PREP 2009
AAP PREP 2009
AAP PREP 2009
AAP PREP 2009
AAP PREP 2009
AAP PREP 2009
AAP PREP 2009
AAP PREP 2009
AAP PREP 2009
AAP PREP 2009
AAP PREP 2009
AAP PREP 2009
AAP PREP 2009
AAP PREP 2009
AAP PREP 2009
AAP PREP 2009
AAP PREP 2009
AAP PREP 2009
AAP PREP 2009
AAP PREP 2009
AAP PREP 2009
AAP PREP 2009
AAP PREP 2009
AAP PREP 2009
AAP PREP 2009
AAP PREP 2009
AAP PREP 2009
AAP PREP 2009
AAP PREP 2009
AAP PREP 2009
AAP PREP 2009
AAP PREP 2009
AAP PREP 2009
AAP PREP 2009
AAP PREP 2009
AAP PREP 2009
AAP PREP 2009
AAP PREP 2009
AAP PREP 2009
AAP PREP 2009
AAP PREP 2009
AAP PREP 2009
AAP PREP 2009
AAP PREP 2009
AAP PREP 2009
AAP PREP 2009
AAP PREP 2009
AAP PREP 2009
AAP PREP 2009
AAP PREP 2009
AAP PREP 2009
AAP PREP 2009
AAP PREP 2009
AAP PREP 2009
AAP PREP 2009
AAP PREP 2009
AAP PREP 2009
AAP PREP 2009
AAP PREP 2009
AAP PREP 2009
AAP PREP 2009
AAP PREP 2009
AAP PREP 2009
AAP PREP 2009
AAP PREP 2009
AAP PREP 2009
AAP PREP 2009
AAP PREP 2009
AAP PREP 2009
AAP PREP 2009
AAP PREP 2009
AAP PREP 2009
AAP PREP 2009
AAP PREP 2009
AAP PREP 2009
AAP PREP 2009
AAP PREP 2009
AAP PREP 2009
AAP PREP 2009
AAP PREP 2009
AAP PREP 2009
AAP PREP 2009
AAP PREP 2009
AAP PREP 2009
AAP PREP 2009
AAP PREP 2009
AAP PREP 2009
AAP PREP 2009
AAP PREP 2009
AAP PREP 2009
AAP PREP 2009
AAP PREP 2009
AAP PREP 2009
AAP PREP 2009
AAP PREP 2009
AAP PREP 2009
AAP PREP 2009
AAP PREP 2009
AAP PREP 2009
AAP PREP 2009
AAP PREP 2009
AAP PREP 2009
AAP PREP 2009
AAP PREP 2009
AAP PREP 2009
AAP PREP 2009
AAP PREP 2009
AAP PREP 2009
AAP PREP 2009
AAP PREP 2009
AAP PREP 2009
AAP PREP 2009
AAP PREP 2009
AAP PREP 2009
AAP PREP 2009
AAP PREP 2009
AAP PREP 2009
AAP PREP 2009
AAP PREP 2009
AAP PREP 2009
AAP PREP 2009
AAP PREP 2009
AAP PREP 2009
AAP PREP 2009
AAP PREP 2009
AAP PREP 2009
AAP PREP 2009
AAP PREP 2009
AAP PREP 2009
AAP PREP 2009
AAP PREP 2009
AAP PREP 2009
AAP PREP 2009
AAP PREP 2009
AAP PREP 2009
AAP PREP 2009
AAP PREP 2009
AAP PREP 2009
AAP PREP 2009
AAP PREP 2009
AAP PREP 2009
AAP PREP 2009
AAP PREP 2009
AAP PREP 2009
AAP PREP 2009
AAP PREP 2009
AAP PREP 2009
AAP PREP 2009
AAP PREP 2009
AAP PREP 2009

Contenu connexe

Tendances

Extreme preterm newborns – survivorship and controversies
Extreme preterm newborns – survivorship and controversiesExtreme preterm newborns – survivorship and controversies
Extreme preterm newborns – survivorship and controversies
Varsha Shah
 
Outpatient Follow Up Of Premature Infants, by Dr. Khaled El-Atawi A/Consultan...
Outpatient Follow Up Of Premature Infants, by Dr. Khaled El-Atawi A/Consultan...Outpatient Follow Up Of Premature Infants, by Dr. Khaled El-Atawi A/Consultan...
Outpatient Follow Up Of Premature Infants, by Dr. Khaled El-Atawi A/Consultan...
mohamed osama hussein
 

Tendances (20)

Necrotizing Enterocolitis: Why Such Enigma?
Necrotizing Enterocolitis: Why Such Enigma?Necrotizing Enterocolitis: Why Such Enigma?
Necrotizing Enterocolitis: Why Such Enigma?
 
Extreme preterm newborns – survivorship and controversies
Extreme preterm newborns – survivorship and controversiesExtreme preterm newborns – survivorship and controversies
Extreme preterm newborns – survivorship and controversies
 
Hypothermia and Beyond: Fine-Tuning Therapeutic Hypothermia and Evidence for ...
Hypothermia and Beyond: Fine-Tuning Therapeutic Hypothermia and Evidence for ...Hypothermia and Beyond: Fine-Tuning Therapeutic Hypothermia and Evidence for ...
Hypothermia and Beyond: Fine-Tuning Therapeutic Hypothermia and Evidence for ...
 
Problems in late preterm babies, iap bps,bangalore,webinar, 20-5-20 - Dr Kart...
Problems in late preterm babies, iap bps,bangalore,webinar, 20-5-20 - Dr Kart...Problems in late preterm babies, iap bps,bangalore,webinar, 20-5-20 - Dr Kart...
Problems in late preterm babies, iap bps,bangalore,webinar, 20-5-20 - Dr Kart...
 
Maternal BMI and risk of cerebral palsy
Maternal BMI and risk of cerebral palsyMaternal BMI and risk of cerebral palsy
Maternal BMI and risk of cerebral palsy
 
Fetal Programming of Adult Disease
Fetal Programming of Adult DiseaseFetal Programming of Adult Disease
Fetal Programming of Adult Disease
 
Mcq in neonatology for medical students
Mcq in neonatology for medical studentsMcq in neonatology for medical students
Mcq in neonatology for medical students
 
Energy nutrient-dense formula in critically ill
Energy nutrient-dense formula in critically illEnergy nutrient-dense formula in critically ill
Energy nutrient-dense formula in critically ill
 
Nelson pediatrics review (mcqs) 19ed
Nelson pediatrics review (mcqs) 19edNelson pediatrics review (mcqs) 19ed
Nelson pediatrics review (mcqs) 19ed
 
Newborn Screening
Newborn ScreeningNewborn Screening
Newborn Screening
 
Problems of late preterms lsd
Problems of late preterms lsdProblems of late preterms lsd
Problems of late preterms lsd
 
Implications for clinicians of the Barker hypothesis
Implications for clinicians of the Barker hypothesisImplications for clinicians of the Barker hypothesis
Implications for clinicians of the Barker hypothesis
 
Foetal programming in blood pressure regulation
Foetal programming in blood pressure regulationFoetal programming in blood pressure regulation
Foetal programming in blood pressure regulation
 
Thaimine
ThaimineThaimine
Thaimine
 
8 follow up care of high risk new born
8 follow up care of high risk new born8 follow up care of high risk new born
8 follow up care of high risk new born
 
Outpatient Follow Up Of Premature Infants, by Dr. Khaled El-Atawi A/Consultan...
Outpatient Follow Up Of Premature Infants, by Dr. Khaled El-Atawi A/Consultan...Outpatient Follow Up Of Premature Infants, by Dr. Khaled El-Atawi A/Consultan...
Outpatient Follow Up Of Premature Infants, by Dr. Khaled El-Atawi A/Consultan...
 
What Does the Future Hold for Autism Families?
What Does the Future Hold for Autism Families?What Does the Future Hold for Autism Families?
What Does the Future Hold for Autism Families?
 
Feeding Difficulties: Controversies, Advances, and Practice Implications
Feeding Difficulties: Controversies, Advances, and Practice ImplicationsFeeding Difficulties: Controversies, Advances, and Practice Implications
Feeding Difficulties: Controversies, Advances, and Practice Implications
 
Food Protein-induced Allergic Proctocolitis to Multiple Foods
Food Protein-induced Allergic Proctocolitis to Multiple FoodsFood Protein-induced Allergic Proctocolitis to Multiple Foods
Food Protein-induced Allergic Proctocolitis to Multiple Foods
 
Can we prevent allergies in children 2019 khaled saad
Can we prevent allergies in children 2019 khaled saadCan we prevent allergies in children 2019 khaled saad
Can we prevent allergies in children 2019 khaled saad
 

Similaire à AAP PREP 2009

Module 4 Paediatric Nutrition.pptx
Module 4 Paediatric Nutrition.pptxModule 4 Paediatric Nutrition.pptx
Module 4 Paediatric Nutrition.pptx
ShafaatHussain20
 
Newborn nutrition and growth
Newborn nutrition and growthNewborn nutrition and growth
Newborn nutrition and growth
Varsha Shah
 
Protective Gut And Nutritional Stratigies
Protective Gut And Nutritional StratigiesProtective Gut And Nutritional Stratigies
Protective Gut And Nutritional Stratigies
Perwin Waly
 
L1,2,3,4. nutrition.pdf
L1,2,3,4. nutrition.pdfL1,2,3,4. nutrition.pdf
L1,2,3,4. nutrition.pdf
Saad49687
 
Transitioning The Breastfeeding Breastmilk Fed Premature Infant From The Neon...
Transitioning The Breastfeeding Breastmilk Fed Premature Infant From The Neon...Transitioning The Breastfeeding Breastmilk Fed Premature Infant From The Neon...
Transitioning The Breastfeeding Breastmilk Fed Premature Infant From The Neon...
Biblioteca Virtual
 
Nutrition and nutritional disorders
Nutrition and nutritional disordersNutrition and nutritional disorders
Nutrition and nutritional disorders
Emily Riegel
 
Flash Heat Method for Breastfeeding Moms with HIV
Flash Heat Method for Breastfeeding Moms with HIVFlash Heat Method for Breastfeeding Moms with HIV
Flash Heat Method for Breastfeeding Moms with HIV
Jenny
 

Similaire à AAP PREP 2009 (20)

Prematurity Pediatrics
Prematurity Pediatrics Prematurity Pediatrics
Prematurity Pediatrics
 
Module 4 Paediatric Nutrition.pptx
Module 4 Paediatric Nutrition.pptxModule 4 Paediatric Nutrition.pptx
Module 4 Paediatric Nutrition.pptx
 
2011. hyperbilirubinemia in the newborn. pediatrics in review
2011. hyperbilirubinemia in the newborn. pediatrics in review2011. hyperbilirubinemia in the newborn. pediatrics in review
2011. hyperbilirubinemia in the newborn. pediatrics in review
 
Preterm : ABCDE; approach to nutritional assessment in preterm infants.pptx
Preterm : ABCDE; approach to nutritional assessment in preterm infants.pptxPreterm : ABCDE; approach to nutritional assessment in preterm infants.pptx
Preterm : ABCDE; approach to nutritional assessment in preterm infants.pptx
 
FEEDING IN AEDF NEWBORN.pptx
FEEDING IN AEDF NEWBORN.pptxFEEDING IN AEDF NEWBORN.pptx
FEEDING IN AEDF NEWBORN.pptx
 
Newborn nutrition and growth
Newborn nutrition and growthNewborn nutrition and growth
Newborn nutrition and growth
 
Protective Gut And Nutritional Stratigies
Protective Gut And Nutritional StratigiesProtective Gut And Nutritional Stratigies
Protective Gut And Nutritional Stratigies
 
Managing diarrhea- Current nutrition perspectives.pptx
Managing diarrhea- Current nutrition perspectives.pptxManaging diarrhea- Current nutrition perspectives.pptx
Managing diarrhea- Current nutrition perspectives.pptx
 
Introduction to global strategy of iycf
Introduction to global strategy of iycfIntroduction to global strategy of iycf
Introduction to global strategy of iycf
 
DIARRHEA-GROUP-5-1.pdf
DIARRHEA-GROUP-5-1.pdfDIARRHEA-GROUP-5-1.pdf
DIARRHEA-GROUP-5-1.pdf
 
Guidelines-of-enteral-feeding in neonates.pdf
Guidelines-of-enteral-feeding in neonates.pdfGuidelines-of-enteral-feeding in neonates.pdf
Guidelines-of-enteral-feeding in neonates.pdf
 
What is colic?
What is colic? What is colic?
What is colic?
 
Breastfeeding
BreastfeedingBreastfeeding
Breastfeeding
 
L1,2,3,4. nutrition.pdf
L1,2,3,4. nutrition.pdfL1,2,3,4. nutrition.pdf
L1,2,3,4. nutrition.pdf
 
Feeding in the low birth weight infant
Feeding in the low birth weight infantFeeding in the low birth weight infant
Feeding in the low birth weight infant
 
Nutritional management of diarrhea
Nutritional management of diarrhea  Nutritional management of diarrhea
Nutritional management of diarrhea
 
Transitioning The Breastfeeding Breastmilk Fed Premature Infant From The Neon...
Transitioning The Breastfeeding Breastmilk Fed Premature Infant From The Neon...Transitioning The Breastfeeding Breastmilk Fed Premature Infant From The Neon...
Transitioning The Breastfeeding Breastmilk Fed Premature Infant From The Neon...
 
Nutrition and nutritional disorders
Nutrition and nutritional disordersNutrition and nutritional disorders
Nutrition and nutritional disorders
 
Flash Heat Method for Breastfeeding Moms with HIV
Flash Heat Method for Breastfeeding Moms with HIVFlash Heat Method for Breastfeeding Moms with HIV
Flash Heat Method for Breastfeeding Moms with HIV
 
total parental nutrition in neonate guidline
total parental nutrition in neonate guidlinetotal parental nutrition in neonate guidline
total parental nutrition in neonate guidline
 

Plus de Imad Zoukar

Pediatric neurology
Pediatric neurologyPediatric neurology
Pediatric neurology
Imad Zoukar
 
Evaluation and management of intestinal obstruction
Evaluation and management of intestinal obstructionEvaluation and management of intestinal obstruction
Evaluation and management of intestinal obstruction
Imad Zoukar
 
Green book-chapter-23-v2 0
Green book-chapter-23-v2 0Green book-chapter-23-v2 0
Green book-chapter-23-v2 0
Imad Zoukar
 
Fever in young children
Fever in young childrenFever in young children
Fever in young children
Imad Zoukar
 
Guillain barré syndrome
Guillain barré syndromeGuillain barré syndrome
Guillain barré syndrome
Imad Zoukar
 
Cat scratch disease
Cat scratch diseaseCat scratch disease
Cat scratch disease
Imad Zoukar
 
emergency pediatrics
emergency pediatricsemergency pediatrics
emergency pediatrics
Imad Zoukar
 
Infectious diseases in children nurse
Infectious diseases in children nurseInfectious diseases in children nurse
Infectious diseases in children nurse
Imad Zoukar
 
Pediatrics in review 1997
Pediatrics in review 1997Pediatrics in review 1997
Pediatrics in review 1997
Imad Zoukar
 
Childhood febrile seizures
Childhood febrile seizuresChildhood febrile seizures
Childhood febrile seizures
Imad Zoukar
 

Plus de Imad Zoukar (11)

Pediatric neurology
Pediatric neurologyPediatric neurology
Pediatric neurology
 
Paediatrics.a.clinical.guide.for.nurses.practitioners ublog.tk
Paediatrics.a.clinical.guide.for.nurses.practitioners ublog.tkPaediatrics.a.clinical.guide.for.nurses.practitioners ublog.tk
Paediatrics.a.clinical.guide.for.nurses.practitioners ublog.tk
 
Evaluation and management of intestinal obstruction
Evaluation and management of intestinal obstructionEvaluation and management of intestinal obstruction
Evaluation and management of intestinal obstruction
 
Green book-chapter-23-v2 0
Green book-chapter-23-v2 0Green book-chapter-23-v2 0
Green book-chapter-23-v2 0
 
Fever in young children
Fever in young childrenFever in young children
Fever in young children
 
Guillain barré syndrome
Guillain barré syndromeGuillain barré syndrome
Guillain barré syndrome
 
Cat scratch disease
Cat scratch diseaseCat scratch disease
Cat scratch disease
 
emergency pediatrics
emergency pediatricsemergency pediatrics
emergency pediatrics
 
Infectious diseases in children nurse
Infectious diseases in children nurseInfectious diseases in children nurse
Infectious diseases in children nurse
 
Pediatrics in review 1997
Pediatrics in review 1997Pediatrics in review 1997
Pediatrics in review 1997
 
Childhood febrile seizures
Childhood febrile seizuresChildhood febrile seizures
Childhood febrile seizures
 

Dernier

Call Girls Aurangabad Just Call 8250077686 Top Class Call Girl Service Available
Call Girls Aurangabad Just Call 8250077686 Top Class Call Girl Service AvailableCall Girls Aurangabad Just Call 8250077686 Top Class Call Girl Service Available
Call Girls Aurangabad Just Call 8250077686 Top Class Call Girl Service Available
Dipal Arora
 
Call Girls Bhubaneswar Just Call 9907093804 Top Class Call Girl Service Avail...
Call Girls Bhubaneswar Just Call 9907093804 Top Class Call Girl Service Avail...Call Girls Bhubaneswar Just Call 9907093804 Top Class Call Girl Service Avail...
Call Girls Bhubaneswar Just Call 9907093804 Top Class Call Girl Service Avail...
Dipal Arora
 
College Call Girls in Haridwar 9667172968 Short 4000 Night 10000 Best call gi...
College Call Girls in Haridwar 9667172968 Short 4000 Night 10000 Best call gi...College Call Girls in Haridwar 9667172968 Short 4000 Night 10000 Best call gi...
College Call Girls in Haridwar 9667172968 Short 4000 Night 10000 Best call gi...
perfect solution
 

Dernier (20)

Call Girls Dehradun Just Call 9907093804 Top Class Call Girl Service Available
Call Girls Dehradun Just Call 9907093804 Top Class Call Girl Service AvailableCall Girls Dehradun Just Call 9907093804 Top Class Call Girl Service Available
Call Girls Dehradun Just Call 9907093804 Top Class Call Girl Service Available
 
Call Girls Ooty Just Call 8250077686 Top Class Call Girl Service Available
Call Girls Ooty Just Call 8250077686 Top Class Call Girl Service AvailableCall Girls Ooty Just Call 8250077686 Top Class Call Girl Service Available
Call Girls Ooty Just Call 8250077686 Top Class Call Girl Service Available
 
(Rocky) Jaipur Call Girl - 09521753030 Escorts Service 50% Off with Cash ON D...
(Rocky) Jaipur Call Girl - 09521753030 Escorts Service 50% Off with Cash ON D...(Rocky) Jaipur Call Girl - 09521753030 Escorts Service 50% Off with Cash ON D...
(Rocky) Jaipur Call Girl - 09521753030 Escorts Service 50% Off with Cash ON D...
 
Top Rated Hyderabad Call Girls Erragadda ⟟ 6297143586 ⟟ Call Me For Genuine ...
Top Rated  Hyderabad Call Girls Erragadda ⟟ 6297143586 ⟟ Call Me For Genuine ...Top Rated  Hyderabad Call Girls Erragadda ⟟ 6297143586 ⟟ Call Me For Genuine ...
Top Rated Hyderabad Call Girls Erragadda ⟟ 6297143586 ⟟ Call Me For Genuine ...
 
Call Girls Jabalpur Just Call 8250077686 Top Class Call Girl Service Available
Call Girls Jabalpur Just Call 8250077686 Top Class Call Girl Service AvailableCall Girls Jabalpur Just Call 8250077686 Top Class Call Girl Service Available
Call Girls Jabalpur Just Call 8250077686 Top Class Call Girl Service Available
 
Call Girls Ludhiana Just Call 9907093804 Top Class Call Girl Service Available
Call Girls Ludhiana Just Call 9907093804 Top Class Call Girl Service AvailableCall Girls Ludhiana Just Call 9907093804 Top Class Call Girl Service Available
Call Girls Ludhiana Just Call 9907093804 Top Class Call Girl Service Available
 
Call Girls Haridwar Just Call 8250077686 Top Class Call Girl Service Available
Call Girls Haridwar Just Call 8250077686 Top Class Call Girl Service AvailableCall Girls Haridwar Just Call 8250077686 Top Class Call Girl Service Available
Call Girls Haridwar Just Call 8250077686 Top Class Call Girl Service Available
 
Call Girls Aurangabad Just Call 8250077686 Top Class Call Girl Service Available
Call Girls Aurangabad Just Call 8250077686 Top Class Call Girl Service AvailableCall Girls Aurangabad Just Call 8250077686 Top Class Call Girl Service Available
Call Girls Aurangabad Just Call 8250077686 Top Class Call Girl Service Available
 
Top Quality Call Girl Service Kalyanpur 6378878445 Available Call Girls Any Time
Top Quality Call Girl Service Kalyanpur 6378878445 Available Call Girls Any TimeTop Quality Call Girl Service Kalyanpur 6378878445 Available Call Girls Any Time
Top Quality Call Girl Service Kalyanpur 6378878445 Available Call Girls Any Time
 
Call Girls Faridabad Just Call 9907093804 Top Class Call Girl Service Available
Call Girls Faridabad Just Call 9907093804 Top Class Call Girl Service AvailableCall Girls Faridabad Just Call 9907093804 Top Class Call Girl Service Available
Call Girls Faridabad Just Call 9907093804 Top Class Call Girl Service Available
 
VIP Hyderabad Call Girls Bahadurpally 7877925207 ₹5000 To 25K With AC Room 💚😋
VIP Hyderabad Call Girls Bahadurpally 7877925207 ₹5000 To 25K With AC Room 💚😋VIP Hyderabad Call Girls Bahadurpally 7877925207 ₹5000 To 25K With AC Room 💚😋
VIP Hyderabad Call Girls Bahadurpally 7877925207 ₹5000 To 25K With AC Room 💚😋
 
Call Girls Bareilly Just Call 8250077686 Top Class Call Girl Service Available
Call Girls Bareilly Just Call 8250077686 Top Class Call Girl Service AvailableCall Girls Bareilly Just Call 8250077686 Top Class Call Girl Service Available
Call Girls Bareilly Just Call 8250077686 Top Class Call Girl Service Available
 
VIP Call Girls Indore Kirti 💚😋 9256729539 🚀 Indore Escorts
VIP Call Girls Indore Kirti 💚😋  9256729539 🚀 Indore EscortsVIP Call Girls Indore Kirti 💚😋  9256729539 🚀 Indore Escorts
VIP Call Girls Indore Kirti 💚😋 9256729539 🚀 Indore Escorts
 
Top Rated Bangalore Call Girls Mg Road ⟟ 9332606886 ⟟ Call Me For Genuine S...
Top Rated Bangalore Call Girls Mg Road ⟟   9332606886 ⟟ Call Me For Genuine S...Top Rated Bangalore Call Girls Mg Road ⟟   9332606886 ⟟ Call Me For Genuine S...
Top Rated Bangalore Call Girls Mg Road ⟟ 9332606886 ⟟ Call Me For Genuine S...
 
Call Girls Bhubaneswar Just Call 9907093804 Top Class Call Girl Service Avail...
Call Girls Bhubaneswar Just Call 9907093804 Top Class Call Girl Service Avail...Call Girls Bhubaneswar Just Call 9907093804 Top Class Call Girl Service Avail...
Call Girls Bhubaneswar Just Call 9907093804 Top Class Call Girl Service Avail...
 
Pondicherry Call Girls Book Now 9630942363 Top Class Pondicherry Escort Servi...
Pondicherry Call Girls Book Now 9630942363 Top Class Pondicherry Escort Servi...Pondicherry Call Girls Book Now 9630942363 Top Class Pondicherry Escort Servi...
Pondicherry Call Girls Book Now 9630942363 Top Class Pondicherry Escort Servi...
 
College Call Girls in Haridwar 9667172968 Short 4000 Night 10000 Best call gi...
College Call Girls in Haridwar 9667172968 Short 4000 Night 10000 Best call gi...College Call Girls in Haridwar 9667172968 Short 4000 Night 10000 Best call gi...
College Call Girls in Haridwar 9667172968 Short 4000 Night 10000 Best call gi...
 
Best Rate (Hyderabad) Call Girls Jahanuma ⟟ 8250192130 ⟟ High Class Call Girl...
Best Rate (Hyderabad) Call Girls Jahanuma ⟟ 8250192130 ⟟ High Class Call Girl...Best Rate (Hyderabad) Call Girls Jahanuma ⟟ 8250192130 ⟟ High Class Call Girl...
Best Rate (Hyderabad) Call Girls Jahanuma ⟟ 8250192130 ⟟ High Class Call Girl...
 
Manyata Tech Park ( Call Girls ) Bangalore ✔ 6297143586 ✔ Hot Model With Sexy...
Manyata Tech Park ( Call Girls ) Bangalore ✔ 6297143586 ✔ Hot Model With Sexy...Manyata Tech Park ( Call Girls ) Bangalore ✔ 6297143586 ✔ Hot Model With Sexy...
Manyata Tech Park ( Call Girls ) Bangalore ✔ 6297143586 ✔ Hot Model With Sexy...
 
Book Paid Powai Call Girls Mumbai 𖠋 9930245274 𖠋Low Budget Full Independent H...
Book Paid Powai Call Girls Mumbai 𖠋 9930245274 𖠋Low Budget Full Independent H...Book Paid Powai Call Girls Mumbai 𖠋 9930245274 𖠋Low Budget Full Independent H...
Book Paid Powai Call Girls Mumbai 𖠋 9930245274 𖠋Low Budget Full Independent H...
 

AAP PREP 2009

  • 1. 2009 PREP SA on CD-ROM Question: 1 You are evaluating a 6-month-old child who has a ventricular septal defect and is scheduled for cardiac surgery. The child’s weight is 6 kg (3rd percentile), length is at the 30th percentile, and head circumference is at the 50th percentile. His mother states she prepares the formula by adding 1 scoop of powder to 2 oz of water. She estimates that he drinks 24 oz of formula per day. You estimate the baby’s intake is approximately 500 kcal per day of cow milk formula, which is the recommended dietary allowance (RDA) for his age. According to his mother, he spits up three times a day and passes two soft stools daily. On physical examination, you hear a 3/6 holosystolic murmur and palpate the liver 1 cm below the right costal margin. Of the following, the BEST explanation for the child’s malnutrition is page 1 A. caloric requirements exceeding the RDA B. cow milk protein intolerance C. incorrect preparation of the formula D. pathologic gastroesophageal reflux E. undiagnosed pancreatic insufficiency
  • 2. 2009 PREP SA on CD-ROM Critique: 1 Preferred Response: A Children who have large ventricular septal defects, such as described for the child in the vignette, have increased pulmonary blood flow and may have ventricular hypertrophy and heart failure. Because their hearts have to work harder, their caloric needs often are increased above the recommended dietary allowance (RDA) for healthy children. The absence of significant vomiting, diarrhea, or rectal bleeding suggests that the patient does not have significant gastroesophageal reflux, cow milk protein intolerance, or pancreatic insufficiency. Incorrect preparation of the formula always should be considered, but the mother's reported preparation is correct. The RDA is defined as "a nutrient intake level that is ... sufficient to meet the nutrient requirements of 97% of healthy individuals" in a given group, categorized by sex and age. RDAs are developed by the Food and Nutrition Board of the Institute of Medicine in collaboration with Health Canada. A full list of RDAs may be found in many reference textbooks and at the Food and Nutrition Information Center on the United States Department of Agriculture web site (http://fnic.nal.usda.gov). RDAs estimate energy requirements of healthy children and do not adjust for chronic illness states that increase caloric needs, such as cystic fibrosis or congenital heart disease. Children who have chronic illnesses often require more calories than the RDA to grow and develop. The estimated daily caloric requirement of a patient is the aggregate of the patient's basal metabolic rate and physical activity. Various mathematic equations can be used to estimate the daily caloric intake of children and adults of different ages. Such equations usually take into account the individual's age, sex, physical activity level, and either the weight and height or, preferably, the body surface area. For children who have chronic illnesses, more accurate estimates of daily caloric requirements can be obtained by using a laboratory-based technique, such as indirect calorimetry. References: McDaniel NL. Ventricular and atrial septal defects. Pediatr Rev. 2001;22:265-270. Available at: http://pedsinreview.aappublications.org/cgi/content/full/22/8/265 Sonneville K. Nutritional requirements: dietary reference intakes. In: Hendricks KM, Duggan C. Manual of Pediatric Nutrition. 4th ed. Hamilton, Ontario, Canada: BC Decker; 2005:83-100 page 2
  • 3. 2009 PREP SA on CD-ROM Question: 2 You are admitting a 750-g female infant to the neonatal intensive care unit (NICU) for treatment of respiratory distress and presumed sepsis. The pregnancy was complicated by chorioamnionitis and preterm labor. The infant’s trachea was intubated, a single dose of exogenous surfactant administered, and both an umbilical venous catheter and umbilical arterial catheter were placed successfully in the delivery room. In the NICU, the infant is placed on a radiant warmer. The nurse caring for the infant asks if the infant will need to be transferred to an isolette incubator. Of the following, the MOST likely reason for this infant to be relocated into an isolette incubator is page 3 A. avoidance of light damage to the eyes B. bronzing of the skin under the radiant warmer C. inability to maintain core temperature on a radiant warmer D. increased risk for infection under the radiant warmer E. increased transcutaneous water loss under the radiant warmer
  • 4. 2009 PREP SA on CD-ROM Critique: 2 Preferred Response: E The extremely low gestational-age newborn (ELGAN) who has a birthweight of less than 1,000 g is at great risk for transcutaneous evaporative water loss in an open air, nonhumidified environment such as a radiant warmer. This is most concerning in the first 24 to 72 hours of postnatal life, diminishing thereafter as the infant's skin becomes cornified. The use of a contained, convectively heated, and humidified environment can reduce transcutaneous evaporative water loss best in the first few days of the ELGAN's postnatal life. Ambient light is not a contributing factor to retinopathy of prematurity, for which the ELGAN is also at risk. Although excessive thermal warming can occur under the radiant warmer, bronzing of the skin is related to the use of phototherapy lights in newborns who have conjugated hyperbilirubinemia. The ELGAN's core temperature can be maintained using a radiant warmer, but evaporative and convective heat losses remain a concern over lengthy periods of time, and differences in extremity, head, and core temperatures may affect perfusion and acid-base status. There is no increased risk for infection on the radiant warmer compared with the isolette incubator. References: Dollberg S, Hoath SB. Temperature regulation in preterm infants: role of the skin-environment interface. NeoReviews. 2001;2:e282-e291. Available for subscription at: http://neoreviews.aappublications.org/cgi/content/full/2/12/e282 Korones SB. An encapsulated history of thermoregulation in the neonate. NeoReviews. 2004;5:e78-e85. Available for subscription at: http://neoreviews.aappublications.org/cgi/content/full/5/3/e78 Sedin G. The thermal environment of the newborn infant. In: Martin RJ, Fanaroff AA, Walsh MC, eds. Fanaroff and Martin's Neonatal-Perinatal Medicine. 8th ed. Philadelphia, Pa: Mosby Elsevier; 2006:585-596 page 4
  • 5. 2009 PREP SA on CD-ROM Question: 3 You are seeing a 1-month-old girl for follow-up after a hospitalization for acute gastroenteritis caused by rotavirus. Her diarrhea had decreased in the hospital while taking oral rehydration solution, but when her mother resumed her usual cow milk formula, the girl began to have an increased number of very watery stools. She appears well hydrated, and findings on her abdominal examination are normal. Of the following, the MOST appropriate approach to managing this infant’s diarrhea is to page 5 A. change to a lactose-free formula for the next few days B. dilute the cow milk formula with oral rehydration solution for the next few days C. give her only oral rehydration solution until the diarrhea resolves D. readmit her to the hospital for administration of intravenous fluids E. repeat her stool studies to confirm the diagnosis of rotavirus infection
  • 6. 2009 PREP SA on CD-ROM Critique: 3 Preferred Response: A The infant described in the vignette most likely has lactase deficiency due to rotavirus infection. Lactase is an enzyme found in the most superficial villous portion of the intestinal brush border, which hydrolyzes lactose to glucose and galactose. Lactase deficiency may have several causes in children and adults. Primary lactase deficiency, the most common type, is a genetically determined condition that affects children and adults at different ages but is unusual before 5 years. Symptoms include abdominal distention, bloating, flatulence, or nausea after the ingestion of lactose, with the amount of lactose needed to cause such symptoms varying from person to person. The diagnosis is made by breath hydrogen testing, and management consists of removing some or all lactose from the diet. Congenital lactase deficiency is extremely rare. Secondary lactase deficiency may develop after an infectious gastroenteritis, such as rotavirus, giardiasis, or cryptosporidiosis. Other causes include celiac disease and enteropathy related to immunodeficiency. Secondary lactase deficiency is suggested when a child who has a recent diarrheal illness experiences worsening diarrhea or bloating after the reintroduction of lactose into the diet, as described for the girl in the vignette. Most children who have gastroenteritis do not develop lactase deficiency. For this reason, most infants can tolerate and should continue taking human milk or standard lactose-containing formula throughout a diarrheal illness. For very young infants (eg, <3 months old), such as the one described in the vignette, or those who have significant fluid losses, a lactose-free formula may be attempted until the diarrhea resolves. Infants who are breastfed should be encouraged to continue breastfeeding, even if secondary lactase deficiency is suspected. Giving full-strength formula or human milk is recommended to supply the child with sufficient calories during the recovery phase of a diarrheal illness; therefore, diluting the formula or providing only oral rehydration solution is inappropriate. If the child is not vomiting, oral hydration is optimal, and intravenous hydration is not necessary. There is no need to confirm the diagnosis of rotavirus infection; doing so would not alter management plans. References: Dalby-Payne J, Elliott E. Gastroenteritis in children. BMJ Clinical Evidence. 2007. Available for subscription at: http://clinicalevidence.bmj.com/ceweb/conditions/chd/0314/0314.jsp Heyman MB; Committee on Nutrition. Lactose intolerance in infants, children, and adolescents. Pediatrics. 2006;118:1279-1286. Available at: http://pediatrics.aappublications.org/cgi/content/full/118/3/1279 King CK, Glass R, Bresee, Duggan C. Managing acute gastroenteritis among children: oral rehydration, maintenance, and nutritional therapy. MMWR Morbid Mortal Wkly Rep Recomm Rep. 2003;52(RR-16):1-16. Available at: http://www.cdc.gov/mmwr/preview/mmwrhtml/rr5216a1.htm page 6
  • 7. 2009 PREP SA on CD-ROM Question: 4 During a prenatal visit with expectant parents, they report that they are strict vegans. They ask you to advise them on a healthy diet and any required supplements. The mother plans to breastfeed the newborn exclusively for the first 6 months. Of the following, you are MOST likely to tell them that their newborn may require supplemental A. calcium B. folate C. iron D. vitamin B6 E. vitamin B12 page 7
  • 8. 2009 PREP SA on CD-ROM Critique: 4 Preferred Response: E A vegan diet, by definition, excludes all foods derived from animal products. A lacto-ovo-vegetarian diet may include milk and eggs. Although a vegan diet may be healthy, there is a risk for vitamin B12 deficiency because vitamin B12 is only found in foods of animal origin. Breastfeeding vegan mothers may produce milk that is deficient in this vitamin and require supplementation that generally is achieved by continuing the consumption of prenatal vitamins containing vitamin B12. The recommended supplementation for breastfed vegan infants to prevent vitamin B12 deficiency is 0.4 mcg/day during the first 6 postnatal months and 0.5 mcg/day from 6 months to 1 year of age. Vegan infants who are not breastfed should receive iron-fortified soy infant formula until 1 year of age to avoid deficiencies in iron. Vegan infants require no other mineral or vitamin supplementation. Vegan diets in older children and adolescents may be low in calcium (similar to the typical American "teenage diet" that contains less than the recommended intake of dairy products), and the zinc consumption may be relatively low due to the absence of phytate, which renders zinc more bioavailable. Children who follow vegan diets may have relatively diminished overall energy intake because such diets commonly are low in fat and high in fiber. Review of nutrient intake and energy intake in conjunction with growth curves of children eating vegan diets in both the United States and the United Kingdom demonstrate no significant health issues. Height and weight measured in vegan populations may be slightly lower than average but not in the range of failure to thrive or short stature. Adolescents eating vegan diets are more likely than adolescents eating a typical American diet to meet nutritional goals, including recommended intake of fruits and vegetables. Vegan adolescents are less likely to be obese because they consume fewer foods high in fat. However, they remain at risk for vitamin B12 deficiency and should consume at least a daily multivitamin. They are less likely to have anemia but just as likely to have low calcium intake as their non-vegan peers. Adolescents who follow a lacto-ovo-vegetarian diet are less likely to have deficiencies in vitamin B12, calcium, and iron. Folate and vitamin B6 are not likely to be deficient in persons who consume vegan diets because those nutrients are found in many legumes, fruits, and vegetables that are the mainstays of the diet. References: Kleinman RE. Nutritional aspects of vegetarian diets. In: Pediatric Nutrition Handbook. 5th ed. Elk Grove Village, Ill: American Academy of Pediatrics; 2003:191-208 Mangels AR, Messina V. Considerations in planning vegan diets: infants. J Am Diet Assoc. 2001:101:670-677. Abstract available at: http://www.ncbi.nlm.nih.gov/pubmed/11424546 Messina V, Mangels AR. Considerations in planning vegan diets: children. J Am Diet Assoc. 2001:101:661-669. Abstract available at: http://www.ncbi.nlm.nih.gov/pubmed/11424545 Moilanen BC. Vegan diets in infants, children and adolescents. Pediatr Rev. 2004:25:174-176. Available at: http://pedsinreview.aappublications.org/cgi/content/full/25/5/174 Perry CL, McGuire MT, Neumark-Sztainer D, Story M. Adolescent vegetarians: how well do their dietary patterns meet the Healthy People 2010 objectives? Arch Pediatr Adolesc Med. 2002; 156:431-437. Available at: http://archpedi.ama-assn.org/cgi/content/full/156/5/431 page 8
  • 9. 2009 PREP SA on CD-ROM Question: 5 You are treating a child who has suffered a splenic injury and is being transfused with large volumes of packed red blood cells for severe anemia. He weighs 10 kg and has received 4 units thus far. Of the following, the finding on electrocardiography that is MOST likely to represent a serious complication of his therapy is page 9 A. atrial flutter B. delta waves C. prominent U waves D. supraventricular tachycardia E. tall-peaked T waves
  • 10. 2009 PREP SA on CD-ROM Critique: 5 Preferred Response: E Administration of fluids and blood products can be essential for resuscitation of the trauma patient but may lead to potentially dangerous electrolyte imbalances. Recognition of these abnormalities, which may be subtle, can be important and even lifesaving. When large volumes of blood are transfused, as reported for the child in the vignette, hyperkalemia may occur, which is believed to be the result of extravasation of potassium from the red blood cells that have been irradiated and stored over time. Hyperkalemia can be associated with paresthesias, weakness, and tingling, although cardiac toxicity typically precedes such symptoms. Severe cardiac rhythym changes may begin abruptly. The classic electrocardiographic sign of hyperkalemia is tall, peaked T waves (Item C5A), particularly as the serum potassium concentration approaches or exceeds 5.0 to 6.0 mEq/L (5.0 to 6.0 mmol/L). As hyperkalemia progresses, other changes in the ECG, such as widening of the QRS complex, may be noted. The rhythm changes that occur as a result of hyperkalemia, including ventricular arrhythmias, may develop abruptly, leading to sudden changes in the patient's clinical condition. Another potential electrolyte complication of fluid and blood administration is hypocalcemia, which results from the citrate-containing anticoagulants that bind free calcium. Hypocalcemia may manifest on ECG as a prolonged QT interval, which tends to widen as the ionized calcium concentrations decrease. Atrial flutter, a primary disease of the atrial tissue, would not be expected in the patient described in the vignette. The delta wave refers to the ECG finding of pre-excitation, seen in conditions such as the Wolff-Parkinson-White syndrome that have an associated "bypass" tract allowing for excitement of the His-Purkinje system without passage through the atrioventricular node (Item C5B). The U wave may be seen in hypokalemia (Item C5C), particularly as concentrations decrease below 2.7 mEq/L (2.7 mmol/L), or hypercalcemia (calcium concentrations exceeding 12.0 mg/dL [3.0 mmol/L]), which would not be expected in the patient described in the vignette. Hypercalcemia also may lead to a diminished QT interval, and with more severe hypercalcemia, second- or third-degree heart block (Item C5D) may develop. Supraventricular tachycardia (Item C5E) would not be expected to occur as a result of a large-volume transfusion process. References: Galel SA, Naiman JL. Use of blood and blood products. In: Rudolph CD, Rudolph AM, eds. Rudolph's Pediatrics. 21st ed. New York, NY: McGraw-Hill Medical Publishing Division; 2003:1576-1581 Vetter V. Arrhythmias. In: Moller JH, Hoffman JIE, eds. Pediatric Cardiovascular Medicine. Philadelphia, Pa: Churchill Livingstone; 2000:833-884 page 10
  • 11. 2009 PREP SA on CD-ROM page 11 Critique: 5 The electrocardiogram in hyperkalemia shows peaked T waves. (Courtesy of A. Friedman)
  • 12. 2009 PREP SA on CD-ROM page 12 Critique: 5 In Wolff-Parkinson-White syndrome, delta waves (arrows) are present that represent pre-excitation depolarization of the QRS complex. (Courtesy of A. Friedman)
  • 13. 2009 PREP SA on CD-ROM page 13 Critique: 5 The U wave (arrow) may be observed in hypokalemia (shown here in which there is also ST depression and flattening of the T wave) or hypercalcemia. (Courtesy of A. Friedman)
  • 14. 2009 PREP SA on CD-ROM page 14 Critique: 5 In third-degree heart block that may result from severe hypercalcemia, P waves are completely dissociated from QRS complexes. (Courtesy of A. Friedman)
  • 15. 2009 PREP SA on CD-ROM page 15 Critique: 5 Narrow complex tachycardia that is consistent with supraventricular tachycardia or paroxysmal atrial tachycardia. (Courtesy of A. Friedman)
  • 16. 2009 PREP SA on CD-ROM Question: 6 A 15-year-old girl presents to the emergency department with a 4-week history of nasal drainage and face pain and a 2-week history of frontal headaches and fatigue. Her mother complains that her daughter has an "attitude" and has not been respectful or seemed to care about anything for the past 2 weeks. The daughter awoke this morning with a headache and vomited. On physical examination, the adolescent is afebrile and has normal vital signs. She responds slowly to questions and is not oriented to the date. She complains of pain to palpation of her cheeks and forehead. She has no nuchal rigidity and no focal weakness. The remainder of the physical examination findings are normal. Of the following, the BEST initial diagnostic procedure is page 16 A. computed tomography scan of the head with intravenous contrast B. emergent electroencephalography to rule out nonconvulsive status epilepticus C. lumbar puncture to rule out meningitis D. nasal swab for bacterial culture E. urine drug screen for barbiturates, amphetamines, and cocaine
  • 17. 2009 PREP SA on CD-ROM Critique: 6 Preferred Response: A The subacute onset of mental status changes described for the adolescent in the vignette warrants an emergency evaluation. In most cases, neuroimaging is indicated, along with appropriate laboratory testing. The relatively nonspecific pain and what her mother perceives as common emotional problems (apathy in a teenager) probably represent early frontal lobe symptoms. The headache on awakening and vomiting are concerning for increased intracranial pressure (ICP). Confusion and psychomotor retardation on the mental status examination indicate involvement of the central nervous system. A focal, ischemic, ictal, infectious/inflammatory, or toxic/metabolic process must be identified urgently. A brain abscess is suggested by the prominent facial pain in this setting; the sinuses are a common source of brain abscesses (Item C6). Brain abscesses often present only with nonspecific pain and not with fever. Head computed tomography (CT) scan is preferred for this patient because the constellation of pain, confusion, and morning vomiting makes a focal intracranial mass a possibility. Increased ICP is associated with morning vomiting because ICP is highest in the morning. Contrast is recommended because of the insidious onset, which could indicate either a neoplasm or infectious process. Intravenous contrast is not needed for all neuroimaging procedures. However, it increases the diagnostic yield of imaging studies where either neoplasm or infection is suspected because both typically involve some degradation of the blood-brain barrier or hypervascularity, resulting in contrast enhancement at the site of the lesion. Magnetic resonance imaging (MRI) with contrast also is a good choice. The advantage of MRI is higher spatial and soft-tissue resolution. Disadvantages of MRI compared with CT include: 1) less availability for emergency department studies; 2) need for pharmacologic sedation in agitated patients because sedation affects mental status, thereby masking disease-related mental status; 3) longer time in the scanner, which could delay treatment decisions; and 4) cost. Thus, in most cases, a head CT scan with contrast is preferred as the initial study in the emergency department. page 17 Electroencephalography (EEG) is an important test for assessment of a patient who has encephalopathy of unclear cause to rule out nonconvulsive status epilepticus (NCSE), particularly if the patient is known to have epilepsy. If an EEG cannot be obtained rapidly, intravenous administration of 0.1 mg/kg lorazepam can treat NCSE immediately, although this would not clear confusion about other causes. For this adolescent, the facial pain makes the diagnosis of NCSE less likely than a brain abscess. A lumbar puncture may be needed to rule out meningitis or encephalitis, but the pain and morning vomiting more strongly suggest the possibility of an intracranial mass. Lumbar puncture prior to head CT is not advised in this case because it could reduce pressure below the foramen magnum and result in herniation from the supratentorial mass. Toxicology screening and nasal swabs are reasonable but not the preferred initial diagnostic tests because they will not affect emergency management of the increased intracranial pressure. References: Goodkin HP, Harper MB, Pomeroy SL. Intracerebral abscess in children: historical trends at Children's Hospital Boston. Pediatrics. 2004;113:1765-1770. Available at: http://pediatrics.aappublications.org/cgi/content/full/113/6/1765 Haslam RHA. Brain abscess. In: Kliegman RM, Behrman RE, Jenson HB, Stanton BF, eds. Nelson Textbook of Pediatrics. 18th ed. Philadelphia, Pa: Saunders Elsevier; 2007:2524-2525
  • 18. 2009 PREP SA on CD-ROM Kan L, Nagelberg J, Maytal J. Headaches in a pediatric emergency department: etiology, imaging, and treatment. Headache. 2000;40:25-29. Abstract available at: http://www.ncbi.nlm.nih.gov/pubmed/10759899 page 18
  • 19. 2009 PREP SA on CD-ROM page 19 Critique: 6 Brain abscess: Contrast-enhanced computed tomography scan of the head reveals a rim-enhancing lesion extending to an area of bony destruction in an opacified left frontal sinus. Vasogenic edema surrounding the abscess creates a mass effect and shift of the midline to the right. (Courtesy of D. Krowchuk)
  • 20. 2009 PREP SA on CD-ROM Question: 7 A 5-year-old girl who is new to your practice presents to the clinic for a prekindergarten physical examination. Her primary caretaker, the maternal grandmother, reports that the child’s mother used multiple street drugs throughout her pregnancy as well as medications prescribed for seizure and bipolar disorders. The grandmother is concerned that this child’s speech development is delayed. On physical examination, you note that the girl has wide-spaced eyes, a short nose, and midface hypoplasia. Of the following, the substance that is MOST likely to be associated with this child’s dysmorphic features is page 20 A. lithium B. lysergic acid diethylamide (LSD) C. marijuana D. methamphetamine E. phenobarbital
  • 21. 2009 PREP SA on CD-ROM Critique: 7 Preferred Response: E The features described for the child in the vignette are most consistent with fetal anticonvulsant syndrome, which can occur following exposure to numerous medications, including phenytoin, carbamazepine, valproate, and phenobarbital. Multiple authors have observed a 10% to 20% incidence of birth defects in infants exposed to phenobarbital in utero. Anomalies include midface hypoplasia, ocular hypertelorism (Item C7A), nail hypoplasia (Item C7B), cleft lip+/-cleft palate, and heart defects as well as developmental delay and pre- and postnatal growth failure. page 21 Phenobarbital-exposed newborns may exhibit a withdrawal syndrome that is evidenced by tremulousness and increased activity. Of interest, such infants are likely to have lower serum bilirubin concentrations than nonexposed neonates. Prenatal exposure to lithium is associated with an increased risk of cardiac malformations (eg, Ebstein anomaly) in the fetus. If the mother takes lithium near term, the exposed neonate may have cyanosis, hypotonia, abnormalities of cardiac rhythm, goiter, hypothyroidism, and nephrogenic diabetes insipidus. Lithium exposure is not associated with dysmorphic features or developmental delays. Despite popular belief, lysergic acid diethylamide (LSD) generally is not associated with birth defects or withdrawal symptoms in prenatally exposed infants. Although there are isolated case reports of birth defects in exposed neonates, an increased risk for anomalies is not borne out by epidemiologic studies. It is important to note, however, that LSD users often abuse other substances, underscoring the importance of taking an in-depth drug/substance abuse history in pregnant women. Marijuana use during pregnancy is not known to be associated with an increased incidence of birth defects, dysmorphic features, or developmental delay in exposed offspring, although further study is needed in this regard. Some investigations have shown reduced fetal growth in exposed pregnancies, but this is not confirmed. Prenatally exposed newborns may have tremulousness, increased irritability, and abnormal visual response to light stimulus. Although there does not appear to be an increase in congenital anomalies associated with methamphetamine use during pregnancy, further study is necessary. There are reports of decreased birthweight in exposed neonates. A neonatal withdrawal syndrome consisting of abnormal sleep patterns, tremulousness, poor feeding, and increased tone has been observed commonly. Once again, it is important to consider polydrug abuse in these instances. As with all teratogens, the timing of exposure is critical, with the most vulnerable period of embryonic development occurring between 18 and 60 days after conception, during organogenesis. Dosage of the offending agent also is important, as are route of administration, modifying environmental factors, and genetic background of the mother and fetus. References: Gallagher RC, Kingham K, Hoyme HE. Fetal anticonvulsant syndrome. In: Cassidy SB, Allanson JE, eds. Management of Genetic Syndromes. 2nd ed. Hoboken, NJ: Wiley-Liss; 2005:239-250 Phenobarbital, lithium, LSD, marijuana, methamphetamine. Reprotox. Available for subscription at: http://www.reprotox.org Phenobarbital, lithium, LSD, marijuana, methamphetamine. Teris. Available for subscription at: http://depts.washington.edu/terisweb/teris
  • 22. 2009 PREP SA on CD-ROM page 22 Critique: 7 Wide-spaced eyes and a short upturned nose are facial features of the fetal anticonvulsant syndrome. This child was exposed to phenytoin. (Courtesy of M. Rimsza)
  • 23. 2009 PREP SA on CD-ROM page 23 Critique: 7 Nail hypoplasia is observed in a child who has fetal anticonvulsant syndrome. (Courtesy of the Media Lab at Doernbecher)
  • 24. 2009 PREP SA on CD-ROM Question: 8 A 15-year-old girl comes to the urgent care clinic complaining of lower abdominal pain for 48 hours. She is nauseated but has had no fever, vomiting, or diarrhea. She is afebrile and denies abdominal trauma. She localizes the pain to the left lower quadrant and describes it as intermittent, stabbing pain episodes separated by intervals of more continuous dull pain. She has never been sexually active. Her last menstrual period was 1 week ago. She has had no vaginal discharge or itching. On physical examination, she has left lower quadrant guarding and rebound tenderness. Her pelvic examination shows no vaginal discharge or uterine tenderness, although there is an exquisitely tender mass in the left adnexal area. Of the following, the MOST likely diagnosis is page 24 A. appendicitis B. endometritis C. ovarian torsion D. sacroiliitis E. splenic rupture
  • 25. 2009 PREP SA on CD-ROM Critique: 8 Preferred Response: C Torsion of any adnexal structure, including the ovary, can result in an acute pelvic mass (Item C8). The most common causes of adnexal torsion in young women are cysts and neoplasms, with about 6% of torsions in one series occurring in the setting of normal-appearing ovaries. Histopathology was benign in more than 90% of cases in this series. Torsion occurs when masses cause the ovary to swing on its vascular pedicle, and larger masses generally are associated with a greater potential for torsion until the size of the mass impedes movement. The onset of pain associated with torsion can be abrupt, sharp, and very severe. However, with intermittent or partial torsion, intense periods of intermittent pain may be separated by generalized aching during detorsion, as described for the teenager in the vignette. Pain can occur for several days or weeks prior to a complete torsion. Nausea or vomiting can accompany severe pain. If adnexal torsion is diagnosed, prompt intervention (untwisting the adnexa usually via laparoscopy) is indicated to preserve ovarian function. Appendicitis is in the differential diagnosis for lower abdominal pain, but is less likely when pain occurs in the left lower quadrant. Splenic rupture is a surgical emergency, like ovarian torsion, but the finding of an adnexal mass on examination is unlikely. Endometritis is not very likely in a young woman who is not sexually active and more commonly presents with midline uterine pain. An orthopedic cause for this patient's pain (eg, sacroiliitis) is unlikely, but it remains in the differential diagnosis of pelvic pain. Sacroiliitis usually presents with low back or hip pain. References: Adams Hillard PJ. Pelvic masses. In: Neinstein LS, eds. Adolescent Health Care: A Practical Guide. 5th ed. Philadelphia, Pa: Lippincott Williams & Wilkins; 2008:706-713 Growdon WB, Laufer MR. Ovarian torsion. UpToDate Online 15.3. 2008. Available for subscription at: http://www.utdol.com/utd/content/topic.do?topicKey=gyn_surg/5273 Laufer MR, Goldstein DP. Gynecologic pain: dysmenorrhea, acute and chronic pelvic pain, endometriosis, and premenstrual syndrome. In: Emans SJH, Laufer MR, Goldstein DP, eds. Pediatric and Adolescent Gynecology. 5th ed. Philadelphia, Pa: Lippincott, Williams & Wilkins; 2005:417-476 Varras M, Tsikini A, Polyzos D, Samara Ch, Hadjopoulos G, Akrivis Ch. Uterine adnexal torsion: pathologic and gray-scale ultrasonographic findings. Clin Exp Obstet Gynecol. 2004;31:34-38. Abstract available at: http://www.ncbi.nlm.nih.gov/pubmed/14998184 page 25
  • 26. 2009 PREP SA on CD-ROM page 26 Critique: 8 Ovarian torsion: Color flow Doppler ultrasonography of the right ovary shows abundant flow in the adjacent pelvic tissue (blue, red, and orange color seen inferiorly) but none in the ovary (the area within the dashed line).
  • 27. 2009 PREP SA on CD-ROM Question: 9 A 2-year-old boy who has chronic renal failure is brought to the emergency department for evaluation of nausea, fatigue, and muscle weakness. On physical examination, the boy has a heart rate of 140 beats/min, decreased perfusion, and palpable pulses. You obtain electrocardiography (Item Q9). Electrolyte measurements include a potassium concentration of 7.5 mEq/L (7.5 mmol/L) and a glucose value of 72.0 mg/dL (4.0 mmol/L). Of the following, the MOST appropriate initial treatment is administration of page 27 A. calcium chloride B. insulin C. normal saline 20 mL/kg D. sodium polystyrene sulfonate E. verapamil
  • 28. 2009 PREP SA on CD-ROM page 28 Question: 9 In hyperkalemia, electrocardiography demonstrates peaked T waves. (Courtesy of A. Friedman)
  • 29. 2009 PREP SA on CD-ROM Critique: 9 Preferred Response: A Hyperkalemia can be asymptomatic or produce symptoms such as nausea, fatigue, and muscle weakness, as reported for the boy in the vignette. The most concerning effect of hyperkalemia is on cardiac membrane polarization. Electrocardiographic changes typically consist of peaked T waves and progress to widening of the QRS complexes with dampening of P waves as the hyperkalemia increases. Without treatment, the patient eventually develops ventricular fibrillation or asystole. The electrocardiographic tracing of the boy described in the vignette demonstrates peaked T waves (Item C9), and he has decreased perfusion. The immediate priority is to stabilize his cardiac membrane potential and decrease his risk of arrhythmias, which is accomplished best by administration of intravenous calcium chloride, an agent that has a rapid onset of action. Insulin causes potassium to shift to the intracellular spaces, thereby decreasing serum concentrations, but it always should be administered in conjunction with glucose to avoid hypoglycemia. Sodium bicarbonate administered intravenously also produces intracellular shifts of potassium and may be useful in patients who have metabolic acidosis. page 29 Treatments to enhance the elimination of potassium often are indicated in the treatment of hyperkalemia, but they have a slower onset of action and, therefore, should not be the first line of treatment in life-threatening hyperkalemia. Sodium polystyrene sulfonate, which can be administered orally or rectally, exchanges sodium for potassium, which subsequently is eliminated from the body. Loop diuretics also can enhance potassium elimination. Dialysis may be indicated when ongoing elevated potassium concentrations are expected, such as in renal failure, or with very high acute concentrations, as seen with tumor lysis syndrome or rhabdomyolysis. Administration of verapamil is not indicated in the treatment of hyperkalemia; it has been reported to be associated with the development of complete heart block in this setting. Normal saline has a minimal effect on hyperkalemia. References: Greenbaum LA. Electrolyte and acid-base disorders: potassium. In: Kliegman RM, Behrman RE, Jenson HB, Stanton BF, eds. Nelson Textbook of Pediatrics. 18th ed. Philadelphia, Pa: Saunders Elsevier; 2007:279-284 Hauser GJ, Kulick AF. Electrolyte disorders in the pediatric intensive care unit. In: Wheeler DS, Wong HR, Shanley TP, eds. Pediatric Critical Care Medicine: Basic Science and Clinical Evidence. New York, NY: Springer-Verlag; 2007:1156-1175
  • 30. 2009 PREP SA on CD-ROM page 30 Critique: 9 In hyperkalemia, electrocardiography demonstrates peaked T waves. (Courtesy of A. Friedman)
  • 31. 2009 PREP SA on CD-ROM Question: 10 A 13-year-old girl who has just moved to the United States from Brazil comes to your office because her mother is worried that she is not "developing yet." On physical examination, her height is 50 inches, and she has a triangular face, a low hairline, high-arched palate, and a shield-shaped chest (Item Q10). Breast tissue is not visible or palpable, but there is Sexual Maturity Rating 3 pubic hair. You obtain bone age radiography and a karyotype and measure serum luteinizing hormone and follicle-stimulating hormone. Of the following, the MOST appropriate additional laboratory measurement is page 31 A. adrenocorticotropic hormone B. prolactin C. 17-hydroxyprogesterone D. testosterone E. thyroid-stimulating hormone
  • 32. 2009 PREP SA on CD-ROM page 32 Question: 10 Shield-shaped chest and lack of breast development, as described for the girl in the vignette. (Courtesy of M. Rimsza)
  • 33. 2009 PREP SA on CD-ROM Critique: 10 Preferred Response: E The clinical findings described for the girl in the vignette are characteristic of Turner syndrome (gonadal dysgenesis) associated with an abnormality of one X chromosome. Girls who have this disorder usually are short (mean adult height, approximately 55 inches without growth hormone treatment); have poorly developed ovaries; and often have dysmorphisms, including a triangular facies, low hairline, high-arched palate, hypoplastic nipples, and an increased carrying angle. They may have left heart disorders such as coarctation of the aorta as well as horseshoe kidney or other renal malformations. Initial screening studies to diagnose Turner syndrome include a karyotype and measurement of luteinizing hormone (LH) and follicle-stimulating hormone (FSH). Most girls who have Turner syndrome do not initiate normal puberty. Concentrations of LH and FSH rise as they reach pubertal age range because they have ovarian failure. Although concentrations of estradiol and other estrogens are low, clinical estradiol assays are not designed to provide accurate values in the low-normal range expected in early puberty. Therefore, physical findings such as breast development are a better marker of estrogen effect than measurements of estrogen. page 33 Adolescents who have Turner syndrome are at higher risk of developing chronic lymphocytic thyroiditis and hypothyroidism than the general population. Approximately 20% of affected adolescent girls have antibody-positive autoimmune chronic lymphocytic thyroiditis, and 5% to 10% develop overt hypothyroidism. Accordingly, measurement of thyroid-stimulating hormone is an appropriate laboratory test for patients such as the girl described in the vignette. An elevated value indicates primary hypothyroidism and the need for confirmatory assessment of free thyroxine and antithyroid antibodies (thyroperoxidase, antimicrosomal, or antithyroglobulin). Abnormalities of the hypothalamic-pituitary-adrenal axis are unusual in patients who have Turner syndrome. Therefore, measurement of adrenocorticotropic hormone is not useful. Measurement of prolactin would be useful if the girl had a pituitary or hypothalamic problem, but her clinical findings strongly point to Turner syndrome. A 17-hydroxyprogesterone value would be elevated in the presence of an adrenal biosynthetic defect leading to the development of the most common form of congenital adrenal hyperplasia (cyp21 or 21-hydroxylase deficiency) as well as some of the less common disorders of adrenal biosynthesis. Measuring testosterone would be reasonable if there were evidence of inappropriate masculinization, such as clitoromegaly and a growth spurt. Some girls who have Turner syndrome have functioning Y chromosomal DNA and could have androgenization, but this is unusual. The presence of Y chromosomal DNA does increase the risk of gonadal malignancy, and girls who have significant Y chromosomal DNA on testing often require prophylactic gonadectomy. References: Chiovato L, Larizza D, Bendinelli G, et al. Autoimmune hypothyroidism and hyperthyroidism in patients with Turner's syndrome. Eur J Endocrinol. 1996;134:568-575. Abstract available at: http://www.ncbi.nlm.nih.gov/pubmed/8664977 Doswell BH, Visootsak J, Brady AN, Graham JM Jr. Turner syndrome: an update and review for the primary pediatrician. Clin Pediatr. 2006;45:301-313. Abstract available at: http://www.ncbi.nlm.nih.gov/pubmed/16703153 Frias JL, Davenport ML, Committee on Genetics and Section on Endocrinology. Health supervision for children with Turner syndrome. Pediatrics. 2003;111:692-702. Available at: http://pediatrics.aappublications.org/cgi/content/full/111/3/692 Matura LA, Ho VB, Rosing DR, Bondy CA. Aortic dilatation and dissection in Turner syndrome.
  • 34. 2009 PREP SA on CD-ROM Circulation. 2007;116:1663-1670. Abstract available at: http://www.ncbi.nlm.nih.gov/pubmed/17875973 Sybert VP, McCauley E. Turner's syndrome. N Engl J Med. 2004;351:1227-1238. Extract available at: http://content.nejm.org/cgi/content/extract/351/12/1227 Turner Syndrome Society Website. Available at: http://www.turnersyndrome.org page 34
  • 35. 2009 PREP SA on CD-ROM page 35 Question: 11 A 2-month-old infant has lost the vision in both of his eyes due to bilateral retinoblastoma. His distressed parents ask how the infant’s blindness will affect his behavior and development. Of the following, the child MOST likely will A. begin saying single words at 16 to 20 months B. begin walking between 18 and 22 months C. display behaviors of an autism spectrum disorder D. have a language-based learning disorder E. have significant cognitive impairments
  • 36. 2009 PREP SA on CD-ROM Critique: 11 Preferred Response: B Legal blindness is defined as central visual acuity with corrective lenses of 20/200 or less in the strongest eye or a limited visual field that extends to an angle of 20 degrees. Congenital blindness occurs in 30 per 100,000 births. More than 50% of children who have visual impairment also have developmental disabilities, such as cognitive-adaptive disability, seizures, hearing impairments, and learning disorders. In many of these cases, the disabilities result from central nervous system pathology. Postnatal blindness, which accounts for approximately 8% to 11% of all childhood blindness, can be caused by infections, trauma, or tumors. Retinoblastoma is the most common primary malignant intraocular tumor of childhood. The initial finding in most cases is a white pupillary reflex (leukokoria) (Item C11). Advanced tumors may be treated with enucleation. Children who have congenital or acquired (eg, due to retinoblastoma) blindness without associated neurologic abnormalities should not be at increased risk for motor or cognitive impairment. They are not at increased risk for language-based learning disabilities or autism spectrum disorders. However, children who have significant visual impairment may begin to walk at an older age (18 to 22 months) than sighted children due to different exposure to motor exploration. They typically develop language skills at the same time (12 months) as sighted children. Children who have visual impairments should be provided with much physical contact that includes hugging and comforting. They should be encouraged to partake in self-help skills and exploration of their environment. References: Davidson PW, Burns CM. Visual impairment and blindness. In: Levine MD, Carey WB, Crocker AC, eds. Developmental- Behavioral Pediatrics. 3rd ed. Philadelphia, Pa: WB Saunders Company; 1999:571-578 Msall ME. Visual impairment. In: Parker S, Zukerman B, Augustyn M. Developmental and Behavioral Pediatrics: A Handbook for Primary Care. 2nd ed. Philadelphia, Pa: Lippincott Williams & Wilkins; 2005:366-369 Olitsky SE, Hug D, Smith LP. Disorders of vision. In: Kleigman RM, Behrman RE, Jenson HB, Stanton BF, eds. Textbook of Pediatrics. 18th ed. Philadelphia, Pa: Saunders Elsevier; 2007:2573-2576 page 36
  • 37. 2009 PREP SA on CD-ROM page 37 Critique: 11 Leukokoria may be observed in patients who have retinoblastoma. (Courtesy of R.G. Weaver, Jr)
  • 38. 2009 PREP SA on CD-ROM Question: 12 A 15-year-old boy presents to the clinic because of a persistent cough. According to his mother, his cough has been present for approximately 2 weeks, but it seems to be getting worse. He does not cough all the time, but the coughing episodes tend to come in bursts. This morning she became very worried because he passed out during a coughing spell. Physical examination reveals a healthy-appearing male in no apparent distress. He is afebrile, and his vital signs are normal. He has petechiae on his face but no other skin lesions. His lungs are clear. Of the following, the MOST appropriate antimicrobial agent to prescribe for this patient is page 38 A. azithromycin B. clarithromycin C. doxycycline D. erythromycin E. trimethoprim-sulfamethoxazole
  • 39. 2009 PREP SA on CD-ROM Critique: 12 Preferred Response: A Any patient who has episodic coughing episodes that conclude in syncope or vomiting, such as the boy described in the vignette, should be considered to have pertussis. Other complications from pertussis described in adolescents include urinary incontinence, sleep interruption, rib fractures, and pneumonia. Despite universal immunization of children against pertussis, a marked increase in disease incidence has been demonstrated among adolescents of 11 to 18 years of age. In an attempt to address this problem, the American Academy of Pediatrics recommends that adolescents in this age group receive a single dose of tetanus toxoid, reduced diphtheria toxoid, and acellular pertussis (Tdap) for booster immunization. For those who received only a tetanus toxoid and reduced diphtheria toxoid (Td) booster, a booster with Tdap is suggested if the interval since the Td administration is at least 2 years or if the adolescent is living in a setting of increased disease, has a risk of a complicated course if he or she acquires the disease, or possibly can transmit infection to a vulnerable contact. Treatment of pertussis with antimicrobial agents once the cough has started does not affect the course of the illness but is recommended to limit the spread of disease to others. In the past, the treatment of choice was erythromycin, but recent data have demonstrated that the effectiveness of other macrolides such as azithromycin or clarithromycin is similar to that of erythromycin, and they have fewer adverse effects. The most common complaints of patients who take erythromycin preparations are gastrointestinal irritation, including epigastric distress, abdominal cramping, nausea, vomiting, and diarrhea. The occurrence of these adverse effects can result in poor adherence to treatment regimens. In addition, erythromycin cannot be used in children younger than 1 month of age due to its association with the development of hypertrophic pyloric stenosis. Azithromycin and clarithromycin attain higher tissue concentrations than erythromycin, have longer half-lives, and can be administered less often and for a shorter total duration. With the advent of these newer macrolides, erythromycin rarely is indicated as a first-line agent for any illness. Azithromycin has become the agent of choice for treatment of pertussis because of the ease of administration (once daily for 5 days) and the fact that it does not inhibit cytochrome P-450, as erythromycin and clarithromycin do. Therefore, it does not interact with other medications that are metabolized by this system (eg, digoxin, carbamazepine). Trimethoprim-sulfamethoxazole also is effective against pertussis and is considered an alternative for patients who cannot tolerate a macrolide or have a macrolide-resistant isolate if they are older than 2 months of age. Doxycycline is not recommended for the treatment of pertussis. References: American Academy of Pediatrics. Pertussis (whooping cough). In: Pickering LK, Baker CJ, Long SS, McMillan JA, eds. Red Book: 2006 Report of the Committee on Infectious Diseases. 27th ed. Elk Grove Village, Ill: American Academy of Pediatrics; 2006:498-520 Committee on Infectious Diseases. Prevention of pertussis among adolescents: recommendations for use of tetanus toxoid, reduced diphtheria toxoid, and acellular pertussis (Tdap) vaccine. Pediatrics. 2006;117:965-978. Available at: http://pediatrics.aappublications.org/cgi/content/full/117/3/965 Tiwari T, Murphy TV, Moran J. Recommended antimicrobial agents for the treatment and postexposure prophylaxis of pertussis: 2005 CDC guidelines. MMWR Recomm Rep. 2005;54(RR14):1-16. Available at: http://www.cdc.gov/mmwr/preview/mmwrhtml/rr5414a1.htm page 39
  • 40. 2009 PREP SA on CD-ROM Question: 13 A 15-year-old male presents for evaluation of a progressively enlarging lesion on his left forearm that began 5 days ago. He explains that the lesion initially looked like a "spider bite" with a blister, but over the last several days, a black scab has developed in the center of the lesion, and there is a large area of redness around the scab. The lesion has been pruritic but not painful. Except for low-grade fevers for the last 2 days, he has had no other systemic symptoms. He returned 1 week ago from a school trip to Morocco, where he visited a leather tannery, went shopping in the large outdoor marketplace, visited some historic sites, and took a camel ride in the desert. He states that the students stayed in a hostel in Morocco, but there were no screens on the windows, and spiders, ants, and other insects were visible in the rooms. On physical examination, the boy is afebrile, and his left forearm is edematous, with a 3x3-cm black eschar surrounded by a 5-cm area of erythema and induration (Item Q13). The lesion is not tender to palpation, and there is no drainage. There are several 1.5-cm tender lymph nodes in his left axilla. Findings on the remainder of his examination are within normal limits. Of the following, the MOST likely cause of this patient’s lesion is page 40 A. Bacillus anthracis B. Francisella tularensis C. Loxosceles laeta D. methicillin-resistant Staphylococcus aureus E. Yersinia pestis
  • 41. 2009 PREP SA on CD-ROM page 41 Question: 13 Eschar, as desribed for the patient in the vignette. (Courtesy of the Centers for Disease Control and Prevention, Public Health Image Library, James H. Steele)
  • 42. 2009 PREP SA on CD-ROM Critique: 13 Preferred Response: A Anthrax is a zoonotic disease caused by Bacillus anthracis, which is a gram-positive, encapsulated, spore-forming rod that occurs in many areas of the world. B anthracis spores can remain viable in the soil for decades (Item C13A), representing a major reservoir of infection for herbivorous livestock through ingestion. Human infection occurs through contact with infected animals or contaminated animal products, including carcasses, hides, hair, wool, meat, bone meal, and other contaminated foodstuffs. page 42 Depending on the route of infection, anthrax disease may manifest in three different forms: cutaneous, inhalational, and gastrointestinal. Approximately 95% of all human anthrax cases are cutaneous anthrax. The incubation period of cutaneous anthrax is 1 to 12 days. The initial skin lesion is a pruritic papule that resembles an insect or spider bite, as described for the boy in the vignette. The papule progresses to the development of a central vesicular or bullous lesion that becomes necrotic and hemorrhagic and forms a central black painless eschar, which is the classic lesion of anthrax (Item C13B). There is marked surrounding edema, swelling, induration, and erythema of the involved area but no associated tenderness. Regional lymphadenopathy, fever, malaise, and headache also may be present. In most cases, the eschar falls off in 1 to 2 weeks, and total resolution occurs in 6 weeks. Infections caused by methicillin-resistant Staphylococcus aureus (MRSA), the bite of Loxosceles laeta, Francisella tularensis, and Yersinia pestis are associated with pain, rapid progression, and in most cases, systemic symptoms. Skin and soft-tissue infections due to MRSA have been described as resembling a "spider bite" that is very tender, with progressive worsening of the erythema, warmth, and tenderness (Item C13C). It is not associated with the development of a bullous, necrotic lesion with eschar. The bite of Loxosceles laeta (the brown recluse spider) may resemble the cutaneous lesion of anthrax, but it is very painful (Item C13D). Also, symptoms develop shortly after the bite and progress rapidly within 24 hours. Francisella tularensis is the agent that causes tularemia. Signs and symptoms develop within 3 to 5 days of exposure and include the abrupt onset of fever, chills, headache, malaise, and fatigue as well as the development of a progressively enlarging, tender, localized lymphadenopathy and a red, painful papule (Item C13E) in a region draining into the involved lymph nodes. The signs and symptoms of Yersinia pestis (plague) develop abruptly and include fever; chills; weakness; headache; and extremely tender, rapidly enlarging swelling of the lymph nodes of the groin, axilla, or neck. References: American Academy of Pediatrics. Anthrax. In: Pickering LK, Baker CJ, Long SS, McMillan JA, eds. Red Book: 2006 Report of the Committee on Infectious Diseases. 27th ed. Elk Grove Village, Ill: American Academy of Pediatrics; 2006:208-211 Butler T, Dennis DT. Yersinia species, including plague. In: Mandell GL, Bennett JE, Dolin R, eds. Mandell, Douglas and Bennett's Principles and Practice of Infectious Diseases. 6th ed. New York, NY: Elsevier Churchill Livingstone; 2005:2691-2700 Inglesby TV, Henderson DA, Bartlett JG, et al. Anthrax as a biological weapon: medical and public health management. JAMA. 1999;281:1735-1745. Available at: http://jama.ama-assn.org/cgi/content/full/281/18/1735 King MD, Humphrey BJ, Wang YF, Kourbatova EV, Ray SM, Blumberg HM. Emergence of community-acquired methicillin-resistant Staphylococcus aureus USA 300 clone as the predominant cause of skin and soft-tissue infections. Ann Intern Med. 2006;144:309-317. Abstract available at: http://www.ncbi.nlm.nih.gov/pubmed/16520471
  • 43. 2009 PREP SA on CD-ROM Penn RL. Francisella tularensis (tularemia). In: Mandell GL, Bennett JE, Dolin R, eds. Mandell, Douglas and Bennett's Principles and Practice of Infectious Diseases. 6th ed. New York, NY: Elsevier Churchill Livingstone; 2005:2674-2685 Swanson DL, Vetter RS. Bites of brown recluse spiders and suspected necrotic arachnidism. N Engl J Med. 2005;352:700-707. Extract available at: http://content.nejm.org/cgi/content/extract/352/7/700 page 43
  • 44. 2009 PREP SA on CD-ROM page 44 Critique: 13 Scanning electron micrograph at magnification 6408X demonstrates spores of Bacillus anthracis. The spores may surive in soil for decades. (Courtesy of the Centers for Disease Control and Prevention, Public Health Image Library, Laura Rose)
  • 45. 2009 PREP SA on CD-ROM page 45 Critique: 13 The classic lesion of cutaneous anthrax is a black eschar with surrounding swelling, erythema, and induration. The affected area is not painful. (Courtesy of the Centers for Disease Control and Prevention, Public Health Image Library)
  • 46. 2009 PREP SA on CD-ROM page 46 Critique: 13 Initial lesions of staphylococcal skin infection may be small tender papules that often are attributed to spider bites (yellow arrows). Frequently, lesions enlarge and develop an overlying pustule (black arrow). (Courtesy of Bernard Cohen, MD, DermAtlas; www.dermatlas.org)
  • 47. 2009 PREP SA on CD-ROM page 47 Critique: 13 An eschar with surrounding painful erythema and swelling is observed following the bite of a brown recluse spider. (Courtesy of M. Smith)
  • 48. 2009 PREP SA on CD-ROM page 48 Critique: 13 An ulcerated papule may appear at the site of inoculation of Francisella tularensis. (Courtesy of the Centers for Disease Control and Prevention, Public Health Image Library, Dr. Thomas F. Sellers)
  • 49. 2009 PREP SA on CD-ROM Question: 14 A 3-month-old infant who has a history of renal dysplasia associated with obstructive uropathy has marked polyuria. He is breastfeeding and receiving supplemental cow milk-based formula. In an effort to reduce the high urine output, you consider reducing the renal solute load by changing feedings from the milk-based formula currently being used. Of the following, the MOST appropriate change is to page 49 A. a hydrolyzed formula containing medium-chain triglycerides B. a more concentrated (24-kcal) milk-based formula C. human milk exclusively D. soy milk-based formula E. whole cow milk
  • 50. 2009 PREP SA on CD-ROM Critique: 14 Preferred Response: C The infant described in the vignette has polyuria caused by a urinary concentrating defect. The concentrating defect is the result of tubular damage due to the obstructive uropathy. The inability to concentrate the urine causes the kidneys to create an "excessive" volume of urine to excrete the solute load presented to them. One strategy to reduce polyuria is to reduce the solute burden placed on the kidneys. Potential renal solute load is affected by intake of protein, sodium, potassium, chloride, and phosphorus. The protein and phosphorus content are the most important variables when comparing infant feeding regimens. page 50 Human milk possesses a lower potential renal solute load than cow milk or cow milk-based formulas. Accordingly, the most appropriate change in feeding for the infant in the vignette is to recommend that the mother stop cow milk formula supplementation and exclusively breastfeed. If human milk is not available, a "low-solute" cow milk-based formula can be used. A low calcium-phosphorus formula has the next lowest potential renal solute load compared with human milk. Cow milk, soy milk-based formula, hydrolyzed formula with medium-chain triglycerides, and 24-kcal milk-based formula all have greater renal solute loads than human milk. Renal solute load should also be considered in nephrogenic diabetes insipidus. References: Fiorino KN, Cox J. Nutrition and growth. In: Robertson J, Shilkofski N, eds. Harriet Lane Handbook: A Manual for Pediatric House Officers. 17th ed. Philadelphia, Pa: Elsevier Mosby; 2005:525-608 Hall RT, Carroll RE. Infant feeding. Pediatr Rev. 2000;21:191-200. Available at: http://pedsinreview.aappublications.org/cgi/content/full/21/6/191 Linshaw MA. Congenital nephrogenic diabetes insipidus. Pediatr Rev. 2007;28:372-380. Available at: http://pedsinreview.aappublications.org/cgi/content/full/28/10/372 Ziegler EE, Fomon SJ. Potential renal solute load of infant formulas. J Nutr. 1989;119 (12 suppl):1785-1788. Available at: http://jn.nutrition.org/cgi/reprint/119/12_Suppl/1785
  • 51. 2009 PREP SA on CD-ROM Question: 15 A mother brings in her 13-month-old daughter for evaluation because her girl developed a perioral rash and "hives" on two occasions last week. One episode occurred while eating yogurt and another happened immediately after eating a bagel with cream cheese. She states that her daughter has eaten other foods such as eggs and bread without problems but is breastfeeding and never has been given milk-based formulas or cow milk. The infant has been given rice milk, but she became fussy and seems to prefer breastfeeding. The mother is concerned that her daughter may be allergic to milk but would like to stop breastfeeding. Of the following, the BEST advice is to recommend page 51 A. a cow milk food challenge in the clinic B. avoidance of milk, egg, soy, and wheat products C. breastfeeding until the child is 3 years old D. switching to an amino acid-based formula E. switching to soy milk
  • 52. 2009 PREP SA on CD-ROM Critique: 15 Preferred Response: E Milk protein allergy is an immunoglobulin (Ig) E-mediated food reaction that affects 2% to 3% of infants within the first postnatal year. Typical symptoms include urticaria, angioedema, atopic dermatitis, and anaphylaxis. With IgE-mediated reactions, the quantity of milk required to result in a reaction often is minimal (eg, milk touching the face, a taste of ice cream). Taking a detailed history about the specific food(s) involved, timing of the onset of symptoms, and type of symptoms is important to distinguish IgE-mediated reactions, as described for the child in the vignette, from other adverse milk reactions, such as milk protein enterocolitis and lactose intolerance. Once an IgE-mediated food allergy is suspected, the clinician should consider allergy skin testing or serum IgE testing for the suspected food. page 52 While awaiting results from either blood testing or allergy consultation for skin testing, the first reasonable action is to switch to a soy-based formula. Approximately 10% to 15% of infants and children who have IgE-mediated milk protein allergies may not tolerate soy formula, but this risk applies to infants younger than 6 months of age. For infants older than 6 months, the risk is closer to 5%. Nonetheless, the initial soy formula trial should be performed in the clinic. Other acceptable formula options in this scenario include an extensively hydrolyzed or an amino acid-based formula, although the unpleasant taste and significantly higher cost can be limiting for many families. Also, because almost all affected infants can be fed successfully with a soy or extensively hydrolyzed formula, switching initially to an amino acid-based formula is not required. Food challenges often are used to assess adverse food reactions, but they generally are reserved for foods that are unlikely allergens or if the clinical history is inconsistent or vague (eg, a patient who eats a particular food and does not always have a reaction). Food challenges may result in anaphylaxis and generally are avoided when the history and testing results support an IgE-mediated reaction. Infants who have a specific food allergy sometimes are placed incorrectly on restricted diets that avoid multiple foods. Without a specific history of other adverse food reactions, avoidance of other foods such as egg or wheat is not recommended. However, parents should be counseled that children can develop other food allergies and should monitor their children during ingestion of other common food allergens. Breastfeeding until age 3 years old is an option that is not preferred by the mother in the vignette. Although most IgE-mediated cow milk allergies resolve by 3 years of age, milk protein allergy can persist past 5 years of age in up to 20% of affected children. References: American Academy of Pediatrics Committee on Nutrition. Soy protein-based formulas: recommendations for use in infant feeding. Pediatrics. 1998;101:148-153. Available at: http://pediatrics.aappublications.org/cgi/content/full/101/1/148 Bhatia J, Greer F, and the Committee on Nutrition. The use of soy protein-based formulas in infant feeding. Pediatrics. 2008;121:1062-1068. Available at: http://pediatrics.aappublications.org/cgi/content/full/121/5/1062 Klemola T, Vanto T, Juntunen-Backman K, Kalimo K, Korpela R, Varjonen E. Allergy to soy formula and to extensively hydrolyzed whey formula in infants with cow's milk allergy: a prospective, randomized study with a follow-up to the age of 2 years. J Pediatr. 2002;140:219-224. Abstract available at: http://www.ncbi.nlm.nih.gov/ pubmed/11865274 Saarinen KM, Pelkonen AS, Mäkelä MJ, Savilahti E. Clinical course and prognosis of cow's milk allergy are dependent on milk-specific IgE status. J Allergy Clin Immunol. 2005;116:869-875. Abstract
  • 53. 2009 PREP SA on CD-ROM available at: http://www.ncbi.nlm.nih.gov/pubmed/16210063 Sampson HA, Leung DYM. Adverse reactions to foods. In: Kleigman RM, Behrman RE, Jenson HB, Stanton BF, eds. Nelson Textbook of Pediatrics. 18th ed. Philadelphia, Pa: Saunders Elsevier; 2007:986-989 page 53
  • 54. 2009 PREP SA on CD-ROM Question: 16 A 16-year-old girl is brought to the emergency department after being found unresponsive in her bedroom. Her parents report finding a note in which she wrote of "wanting to end the pain." In addition, they found several empty, unlabeled pill vials on her dresser. On physical examination, the girl is responsive only to painful stimuli. Her heart rate is 60 beats/min, respiratory rate is 16 breaths/min, blood pressure is 90/60 mm Hg, and oxygen saturation is 92%. Her pupils are 3 mm, equal in size, and sluggishly reactive. The remainder of findings on her physical examination are normal. Of the following, the MOST important diagnostic test to obtain when evaluating this patient is a page 54 A. carboxyhemoglobin concentration B. complete blood count C. serum acetaminophen concentration D. serum ammonia concentration E. serum osmolality
  • 55. 2009 PREP SA on CD-ROM Critique: 16 Preferred Response: C Treatment of a patient who has ingested an unknown substance or substances, such as the one described in the vignette, should focus initially on stabilization of vital functions; treatment of the patient's symptoms; and identification of agents that are potentially fatal, have delayed clinical toxicity, or for which antidotal therapy is indicated. The evaluation should include a screening history to identify circumstances surrounding the event, potential environmental exposures, available medications or toxins, and previous medical or psychiatric history. The physical examination should focus on vital sign abnormalities, pupillary size and reactivity, skin findings, and mental status. These components are likely to yield the most useful clues to toxins that have identifiable symptom complexes (Item C16A). Laboratory testing should assess the patient's acid/base status, oxygenation and ventilation, glucose concentration, and anion gap. Qualitative urine drug testing may be obtained but has significant limitations, including the small number of drugs tested (typically drugs of abuse) and screening thresholds that may produce false-negative results. Therefore, the results of urine toxicologic screening rarely affect treatment plans. Quantitative drug concentrations should be measured based on the information gleaned from the initial history, physical examination, and screening laboratory testing, although acetaminophen and salicylate concentrations should be measured for every patient who has an unknown, mixed, or intentional ingestion. Acetaminophen is of particular concern because it causes few initial symptoms, is a common agent in adolescent and adult intentional ingestions, and may require antidotal therapy to prevent potential fatal liver damage. Abdominal radiographs may be considered to examine the patient for radio-opaque substances (Item C16B). A complete blood count is not likely to provide information leading to the identification of a toxin. Measurement of serum ammonia may be indicated if acetaminophen is identified as the ingested agent because toxicity can lead to hepatic damage. Carboxyhemoglobin should be measured in the patient whose history is suggestive of carbon monoxide exposure. Increased serum osmolality may provide indirect evidence of alcohol ingestion, although a quantitative serum test for alcohols is more useful in this setting. References: Erikson TB, Thompson TM, Lu JJ. The approach to the patient with an unknown overdose. Emerg Med Clin North Am. 2007;25:249-281. Abstract available at: http://www.ncbi.nlm.nih.gov/pubmed/17482020 Lavallee M, Olsson J Jr, Cheng TL. In brief: unknown poison. Pediatr Rev. 2004;25:370-371. Available at: http://pedsinreview.aappublications.org/cgi/content/full/25/10/370 McKay CA Jr. Can the laboratory help me? Toxicology laboratory testing in the possibly poisoned pediatric patient. Clin Pediatr Emerg Med. 2005;6:116-122 Valez LI, Shepherd JG, Goto CS. Approach to the child with occult toxic exposure. UpToDate Online 15.3. 2008. Available for subscription at: http://www.utdol.com/utd/content/topic.do?topicKey=ped_tox/3023&selectedTitle=4~150&source=searc h_result page 55
  • 56. 2009 PREP SA on CD-ROM page 56 Critique: 16
  • 57. 2009 PREP SA on CD-ROM page 57 Critique: 16
  • 58. 2009 PREP SA on CD-ROM Question: 17 You are following a 3-month-old infant who was born at 30 weeks’ gestation, underwent a distal ileal resection for necrotizing enterocolitis at 2 weeks of age, and subsequently was placed on parenteral nutrition for 2 months. The baby has residual cholestasis from the parenteral nutrition (total bilirubin, 5.0 mg/dL [85.5 mcmol/L]; direct bilirubin, 3.0 mg/dL [51.3 mcmol/L]). Currently, she is receiving a cow milk protein hydrolysate formula concentrated to 24 kcal/oz (0.8 kcal/mL). You are considering adding a dietary supplement to increase the caloric density of the formula. Of the following, the supplement that is the MOST likely to be tolerated and cause less diarrhea in this infant is page 58 A. flaxseed oil B. medium-chain triglyceride oil C. olive oil D. omega-3 polyunsaturated fatty acid (fish oil) E. soybean oil
  • 59. 2009 PREP SA on CD-ROM Critique: 17 Preferred Response: B Infants who have chronic illnesses may have specialized nutritional requirements and often do not tolerate the standard 20-kcal/oz formula given to healthy term infants. For example, children who have some forms of congenital heart disease or renal disease may require a more concentrated formula because the standard formula may lead to volume overload. Children who have intestinal disease or malabsorption, such as the child described in the vignette, also may require a more concentrated formula to decrease the likelihood of feeding intolerance or diarrhea. page 59 Formula may be concentrated by increasing the concentration of protein, carbohydrate, or fat. Perhaps the easiest method of increasing the caloric density of a formula is to mix more powder with the same amount of water. For example, four scoops of most commercially available formulas mixed with 8 oz of water yields standard 0.67-kcal/mL (20-kcal/oz) formula, but mixing five scoops in 8 oz results in 0.83-kcal/mL (25-kcal/oz) formula. However, exceeding 25-kcal/oz formula by increasing the amount of powder may yield too high a concentration of protein, which could result in an excessive renal solute load. For this reason, carbohydrate and fat supplements are available to concentrate infant and toddler feedings further. The most common adverse effect of carbohydrate supplements is diarrhea, and lipid supplements may cause either diarrhea or delayed gastric emptying. For these reasons, caution is recommended when increasing the caloric density of a formula, especially when the caloric density is increased to greater than 1 kcal/mL (30 kcal/oz). Because the patient in the vignette has had an ileal resection and cholestasis, the best fat supplement for him is oil composed of medium-chain triglycerides (MCT oil). MCT oil can be absorbed directly across the enterocyte and does not require intraluminal digestion by bile acids. In contrast, soy, olive, flaxseed, and fish oils are long-chain fatty acids that require bile acids for digestion and might cause diarrhea in a child who has cholestasis and ileal resection. References: Courtney E, Grunko A, McCarthy T. Enteral nutrition. In: Hendricks KM, Duggan C. Manual of Pediatric Nutrition. 4th ed. Hamilton, Ontario, Canada: BC Decker; 2005:252-316 Suchy FJ. Neonatal cholestasis. Pediatr Rev. 2004;25:388-396. Available at: http://pedsinreview.aappublications.org/cgi/content/full/25/11/388
  • 60. 2009 PREP SA on CD-ROM Question: 18 You have admitted a 750-g male infant to the neonatal intensive care unit (NICU) for treatment of respiratory distress and presumed sepsis. The Apgar scores were 1, 5, and 7 at 1, 5, and 10 minutes, respectively. The infant received one dose of exogenous surfactant in the delivery room. In the NICU, the infant is being cared for on a radiant warmer. At 4 hours after birth, physical examination reveals a temperature of 97.0°F (36.1°C), heart rate of 180 beats/min, respiratory rate of 40 breaths/min (assisted breaths on the ventilator), blood pressure of 45/27 mm Hg, mean arterial blood pressure of 30 mm Hg, and pulse oximetry of 92%. The infant is receiving synchronized intermittent mechanical ventilation with a peak inflation pressure of 18 cm H2O over a positive end-expiratory pressure of 4 cm H2O at a rate of 40 breaths/min and an FiO2 of 0.40. Umbilical catheters are present in the umbilical artery and vein. On physical examination, you note a soft, flat anterior fontanelle. You auscultate equal mechanical breath sounds bilaterally over the chest and note minimal subcostal retractions. The skin is thin and somewhat moist, and many veins are visible through it. The ears are flattened against the cranium and lack any cartilage or recoil. There is a small phallus and an empty scrotum. The infant is laying on the warmer with legs and arms extended. The neuromotor tone is decreased, and the infant does not appear to be very active, but he does respond to tactile stimuli with movement of the arms and legs in seemingly random and purposeless activity. Of the following, the MOST important next step is to page 60 A. administer another dose of exogenous surfactant B. consult the urologist for cryptorchidism C. increase the inspired oxygen concentration D. move the infant into an isolette incubator E. obtain head ultrasonography to evaluate for any hemorrhage
  • 61. 2009 PREP SA on CD-ROM Critique: 18 Preferred Response: D The extremely low gestational-age newborn (ELGAN) whose birthweight is less than 1,000 g may present with physical examination findings indicative of marked physical and neurodevelopmental immaturity. These findings have been studied and correlated with gestational age in scoring systems such as those published by Dubowitz in 1980 and Ballard in 1979 and (revised) 1991. Even the best efforts to estimate the gestational age of very low-birthweight and immature infants at dates less than 28 completed weeks of gestation by maternal last menstrual period and obstetric ultrasonography may be imprecise and tend to overestimate maturity (gestational age) by 2 weeks. Nevertheless, the thin, moist, transparent skin; flattened thin ears without cartilage or recoil; and small phallus and empty scrotum described for the infant in the vignette indicate a degree of physical immaturity consistent with 28 weeks' gestation or less. Similarly, the low neuromotor tone, absence of flexed posture, relative inactivity, and random purposeless flailing of extremities with tactile stimulation indicate significant neurologic immaturity. Moving the infant into an isolette incubator is an important next step in the care of this ELGAN. The absolute benefit of using a radiant warmer bed (access to the infant to conduct early examinations, admission nursing care, and procedures such as placement of umbilical catheters) is eclipsed by its associated increased ambient convection, evaporative heat loss, and transcutaneous water loss compared with the contained, convectively heated, and humidified environment of the isolette. The isolette environment still allows access, visibility, and treatment with phototherapy, but with less insensible water loss or variability in patient temperature. This ELGAN is being treated for respiratory distress syndrome and has had a good response to exogenous surfactant administration, oxygen, and assisted ventilation. At 4 hours of postnatal age, it is too early to administer a second dose of surfactant. Developmentally cryptorchid testes may yet descend into the scrotal sac in coming weeks. An oxygen saturation of 92% is good for this infant, and increasing the inspired oxygen concentration may be harmful, contributing to oxidative injury of pulmonary tissues or retinopathy of prematurity. Cranial ultrasonography is indicated in this sick and preterm infant, but without clinical indications of hemorrhage, shock, and seizure or encephalopathy, such a study is performed best at 3 to 7 days of postnatal life. References: Donovan EF, Tyson JE, Ehrenkranz RA, et al. Inaccuracy of Ballard scores before 28 weeks' gestation. National Institute of Child Health and Human Development Neonatal Research Network. J Pediatr. 1999;135:147-152. Abstract available at: http://www.ncbi.nlm.nih.gov/pubmed/10431107 Marín GMA, Martín Moreiras J, Llitera Fleixas G, et al. Assessment of the new Ballard score to estimate gestational age [in Spanish]. An Pediatr (Barc). 2006;64:140-145. English abstract available at: http://www.ncbi.nlm.nih.gov/pubmed/16527066 Sedin G. The thermal environment of the newborn infant. In: Martin RJ, Fanaroff AA, Walsh MC, eds. Fanaroff and Martin's Neonatal-Perinatal Medicine. 8th ed. Philadelphia, Pa: Mosby Elsevier; 2006:585-596 Thilo EH, Rosenberg AA. The newborn infant. In: Hay WW Jr, Levin MJ, Sondheimer JM, Deterding RR, eds. Current Pediatric Diagnosis & Treatment. 18th ed. New York, NY: The McGraw-Hill Companies, Inc; 2007:chap 1 page 61
  • 62. 2009 PREP SA on CD-ROM Question: 19 You are called to the newborn nursery to evaluate a 1-day-old girl whose hands and feet are blue. She was born at term via a cesarean section, and there were no complications. Apgar scores were 9 at both 1 and 5 minutes. Her respiratory rate is 40 breaths/min, heart rate is 140 beats/min, and blood pressure is normal. Pulse oximetry is 98% on room air. Her lungs are clear, and there is no murmur. Her lips are pink, but her hands and feet are cyanotic (Item Q19), and capillary refill is less than 2 seconds. Of the following, the MOST likely cause of her peripheral cyanosis is page 62 A. cold environment B. polycythemia C. retained fetal lung fluid D. sepsis E. transposition of the great arteries
  • 63. 2009 PREP SA on CD-ROM page 63 Question: 19 Cyanosis of the feet, as exhibited by the infant in the vignette. (Courtesy of the Media Lab at Doernbecher)
  • 64. 2009 PREP SA on CD-ROM Critique: 19 Preferred Response: A Cyanosis is a common finding in newborns. The first step in the evaluation of an infant who exhibits cyanosis is to determine whether it is central or peripheral (acrocyanosis). The infant described in the vignette has clinical features suggestive of acrocyanosis, a bluish discoloration of the hands and feet (Item C19A) in response to vasomotor instability or cold environment. In this condition, the lips and mucous membranes are spared, although the perioral area may be affected. The extremities may be cool to touch. Acrocyanosis is believed to be due to vasoconstriction of small arterioles and resolves in the first few postnatal months. Infants who have acrocyanosis require no further evaluation; parental reassurance is all that is needed. Central cyanosis is the bluish discoloration of the tissues best observed in the lips, tongue, mucous membranes, and occasionally nail beds (Item C19B). It occurs when the concentration of deoxygenated (reduced) hemoglobin exceeds 4.0 to 6.0 g/dL (40.0 to 60.0 g/L) within the capillary bed. In an individual who has normal hemoglobin, cyanosis usually becomes apparent when the oxygen saturation drops below 75%-80%. Causes of central cyanosis in the neonatal period typically are related to cyanotic heart disease, such as transposition of the great arteries, or respiratory disease, such as pneumonia and retained fetal lung fluid (transient tachypnea of the newborn). The absence of central cyanosis, a cardiac murmur, and respiratory distress for the infant in the vignette makes these diagnoses less likely than acrocyanosis of infancy. Polycythemia, defined as a venous hematocrit of at least 65% (0.65), causes a deep reddish-purple discoloration of the skin and mucous membranes. Infants may be otherwise asymptomatic or may exhibit lethargy, anorexia, or respiratory distress. Polycythemia does not cause either peripheral or central cyanosis per se, but it does affect the percent saturation at which cyanosis becomes clinically apparent. Cyanosis in infants who have polycythemia occurs at higher oxygen saturations than in those who have anemia. For example, if an infant has a hemoglobin of 24.0 g/dL (240.0 g/L), he is likely to appear cyanotic when his oxygen saturation reaches 87%. In an infant who has a hemoglobin of 12.0 g/dL (120.0 g/L), cyanosis may not be seen until the oxygen saturation falls to 75%. Newborns who have sepsis may develop cyanosis as a component of multisystem involvement, but other signs, such as respiratory distress, poor peripheral perfusion, lethargy, or poor feeding, also are evident. References: Bernstein D. Evaluation of the cardiovascular system: history and physical examination. In: Kliegman RM, Behrman RE, Jenson HB, Stanton BF, eds. Nelson Textbook of Pediatrics. 18th ed. Philadelphia, Pa: Saunders Elsevier; 2007:1857-1863 Sasidharan P. An approach to diagnosis and management of cyanosis and tachypnea in term infants. Pediatr Clin North Am. 2004;51:999-1021. Abstract available at: http://www.ncbi.nlm.nih.gov/pubmed/15275985 page 64
  • 65. 2009 PREP SA on CD-ROM page 65 Critique: 19 Acrocyanosis is a blue discoloration of the hands and feet. (Courtesy of the Media Lab at Doernbecher)
  • 66. 2009 PREP SA on CD-ROM page 66 Critique: 19 Central cyanosis, as exhibited by this infant who has tetralogy of Fallot, is observed best in highly vascularized areas, such as the lips, tongue, mucous membranes, and nail beds. (Courtesy of M. Rimsza)
  • 67. 2009 PREP SA on CD-ROM Question: 20 A frustrated mother requests referral of her 15-month-old child to an allergy and asthma specialist because the boy never seems to have stopped coughing and wheezing over the 6 months of the past respiratory virus season. During the history taking, the mother states that the baby only occasionally is exposed to wood smoke at the family’s barbecue restaurant and to cosmetic chemicals used at the grandmother’s hair salon. Both the father and grandfather smoke cigarettes in the home. Of the following, the environmental exposure that is MOST likely to be causing the child’s respiratory symptoms is exposure to page 67 A. cigarette smoke B. cleaning fluids C. dust mites D. hairspray E. wood smoke
  • 68. 2009 PREP SA on CD-ROM Critique: 20 Preferred Response: A The child described in the vignette may have multiple triggers for his wheezing exacerbations, although tobacco smoke probably is the one to which he is exposed most often that should be diminished. Well-known tobacco toxins include nicotine, carbon monoxide, formaldehyde, hydrogen cyanide, sulfur dioxide, nitrogen oxide, ammonia, polycyclic aromatic hydrocarbons, and the nitrosamines. These substances produce both irritant and immunologic effects on the respiratory tract. It is important to note that smoking cessation is difficult, and most pediatricians have little training in the process. However, there is evidence that a brief discussion of smoking cessation in the context of medical visits is effective. Wood smoke is another significant irritant to the respiratory tract; in addition to chemical irritants, wood smoke contains particulate matter. Assuming that the family's restaurant kitchen meets current standards for air filtration/exhaust, the child's exposure should be minimal. However, outdoor barbecue pits that discharge smoke directly into the environment should be avoided. Exposure to cooking spray/oils at the restaurant and cosmetic chemicals such as hairspray at the salon also should be avoided, although the use of volatile organic compounds, including chlorofluorocarbons in hairspray, permanent wave solution, cleaning solutions, and cooking sprays, has decreased sharply since passage of the Clean Air Act and other Environmental Protection Agency measures in the 1990s. Material Safety Data Sheets for most hairsprays, as well as other cosmetics and cleaning fluids, reveal little inhalation toxicity, except in the case of prolonged exposure under improper ventilation. Although dust mite exposure may contribute to asthma exacerbation, there is no evidence that environmental control can reduce the concentration of mites or exacerbations related to them. Finally, serial respiratory virus infections themselves may contribute significantly to this child's history of wheezing and cough. The child in the vignette may benefit from therapeutic advances in asthma management such as those contained in the National Heart, Lung and Blood Institute Guidelines for asthma, including attention to number of symptomatic days, use of inhaled corticosteroids, and use of a metered dose inhaler with spacer for proper delivery of medication, which may be more effective than nebulization in older infants and children. References: Brunnhuber K, Cummings KM, Feit S, Sherman S, Woodcock J.Putting evidence into practice: smoking cessation. BMJ Clinical Evidence. 2007. Available for subscription at: http://clinicalevidence.bmj.com/ceweb/resources/index.jsp Kum-Nji P, Meloy L, Herrod HG. Environmental tobacco smoke exposure: prevalence and mechanisms of causation of infections in children. Pediatrics. 2006;117:5:1745-1754. Available at: http://pediatrics.aappublications.org/cgi/content/full/117/5/1745 Roseby R, Waters E, Polnay A, Campbell R, Webster P, Spencer N. Family and carer smoking control programmes for reducing children's exposure to environmental tobacco smoke. Cochrane Database Syst Rev. 2003;3:CD001746. Available at: http://www.cochrane.org/reviews/en/ab001746.html Stein RT, Holberg CJ, Sherrill D, et al. Influence of parental smoking on respiratory symptoms during the first decade of life: The Tucson Children's Respiratory Study. Am J Epidemiol. 1999;149:1030-1037. Available at: http://aje.oxfordjournals.org/cgi/reprint/149/11/1030 page 68
  • 69. 2009 PREP SA on CD-ROM U.S. Environmental Protection Agency. Health effects of wood smoke. Available at: http://www.epa.gov/woodstoves/healtheffects.html U.S. Environmental Protection Agency. National volatile organic compound emission standards for consumer products. Available at: http://www.epa.gov/fedrgstr/EPA-AIR/1998/September/Day-11/a22660.htm page 69
  • 70. 2009 PREP SA on CD-ROM Question: 21 You are evaluating a newborn 6 hours after his birth. Labor and delivery were uncomplicated, but amniocentesis performed during the pregnancy revealed trisomy 21. Fetal echocardiography at 20 weeks’ gestation showed normal findings. The infant currently is sleeping and is well-perfused, with a heart rate of 140 beats/min and no audible murmurs. His physical features are consistent with Down syndrome. Of the following, the MOST appropriate diagnostic study to perform is page 70 A. barium swallow B. cervical spine radiography C. echocardiography D. head ultrasonography E. radiography of the abdomen
  • 71. 2009 PREP SA on CD-ROM Critique: 21 Preferred Response: C Congenital heart defects (CHDs) are the most common of the congenital anomalies, occurring with an incidence of approximately 5 to 8 per 1,000 live births (0.5% to 0.8%). The incidence of CHD is greater in stillbirths and there is an increased incidence of CHD in those who have aneuploidy. It is believed by some that the abnormal chromosomal composition rather than the cardiac abnormality is responsible for the fetal demise of those who have CHDs. The strong association between chromosomal abnormality and CHD has been shown in a number of studies that focus on identification of fetal cardiac abnormality. A number of well-defined chromosomal anomalies are associated with CHD (Item C21). page 71 Early and accurate diagnosis of CHD is important in counseling parents of children in whom a chromosomal abnormality or syndrome is suspected. Such diagnoses may have significant effects on the health and well-being of the newborns because some CHDs may require ductal patency for perfusion of either the systemic or pulmonary circulation. Therefore, echocardiography now is considered an important component of the routine health supervision of infants who have Down syndrome and other syndromes predisposing to structural heart defects. Although results of fetal echocardiography for the infant in the vignette were interpreted as normal, not all CHDs can be diagnosed routinely with fetal echocardiography because of the shunting pathways of the fetal circulation. For example, persistent patency of the ductus arteriosus and secundum atrial septal defects are diagnosed postnatally. Small ventricular septal defects frequently are not seen during fetal echocardiography because the pressure in the right and left ventricles are equal due to the ductus arteriosus, resulting in minimal flow across the defect prenatally. Given the strong association between Down syndrome and CHD, echocardiography should be performed in the newborn described in the vignette. Although children born with Down syndrome have an increased incidence of duodenal atresia and other types of gastrointestinal obstruction, a barium swallow is not an appropriate initial test for an asymptomatic newborn. Cervical radiography, beginning at age 3 years, is important because of the risk of atlantoaxial (C1-C2) subluxation. There is no indication for routine head ultrasonography or abdominal radiography in an otherwise asymptomatic newborn who has Down syndrome. References: Committee on Genetics. Health supervision for children with Down syndrome. Pediatrics. 2001;107:442-449. Available at: http://pediatrics.aappublications.org/cgi/content/full/107/2/442 Silberbach M, Hannon D. Presentation of congenital heart disease in the neonate and young infant. Pediatr Rev. 2007;28:123-131. Available at: http://pedsinreview.aappublications.org/cgi/content/full/28/4/123 Tennstedt C, Chaoui R, Körner H, Dietel M. Spectrum of congenital heart defects and extracardiac malformations associated with chromosomal abnormalities: results of a seven year necropsy study. Heart. 1999;82:34-39. Abstract available at: http://www.ncbi.nlm.nih.gov/pubmed/10377306
  • 72. 2009 PREP SA on CD-ROM page 72 Critique: 21
  • 73. 2009 PREP SA on CD-ROM Question: 22 A 4-year-old boy presents with headache and difficulty walking. On physical examination, he is afebrile, all growth parameters are within normal limits, and his mentation appears normal. The optic discs are clearly visible and appear normal. He has normal eye position in primary gaze but cannot abduct his right eye fully. He has normal tone, strength, and reflexes in his upper limbs, but has bilateral hyperreflexia at the knees and ankle clonus. On gait examination, he toe-walks. Of the following, the MOST important next step is to obtain page 73 A. computed tomography scan of the head B. electromyography/nerve conduction studies of the legs C. lumbar puncture D. magnetic resonance imaging of the thoracolumbar spine E. visual evoked potentials
  • 74. 2009 PREP SA on CD-ROM Critique: 22 Preferred Response: A The boy described in the vignette presents with a very concerning constellation of symptoms and signs requiring urgent evaluation of the central nervous system. The first step in the diagnostic process is to localize the problem to the proper level of the nervous system: brain, brainstem/cerebellum, spinal cord, nerve, junction, or muscle. Such a determination not only allows for urgent appropriate diagnosis and treatment but also reduces unnecessary discomfort, risks, and costs of inappropriate diagnostic testing. Consultation with a neurologist can be useful before ordering testing. Headache, gait disturbance (toe-walking), and acquired ocular misalignment (right eye cannot move to the right) localizes the problem to the central nervous system and raises concerns for both hydrocephalus and a brainstem lesion. page 74 Computed tomography (CT) scan of the head is the preferred neuroimaging technique for this patient. Although brain magnetic resonance imaging (MRI) can document the brainstem/posterior fossa better, it may not be readily available. Moreover, a head CT scan in the emergency department is adequate to rule out hydrocephalus that requires emergent neurosurgical consultation. Unfortunately, a common cause for hydrocephalus in a child of this age is a brainstem or cerebellar neoplasm such as an astrocytoma, glioma, medulloblastoma, or ependymoma. When these tumors enlarge in the posterior fossa, they can obstruct the flow of cerebrospinal fluid and cause acute hydrocephalus, which is a neurosurgical emergency. Primary headache disorders such as migraine do not often present before the age of 6 years and are not accompanied by the neurologic findings described in the vignette. Toe-walking is a nonspecific finding that can occur in the context of relatively benign delayed development, although if this represents a change for a child, an upper motor neuron (brain/spinal cord) lesion must be ruled out. The ankle clonus described for the boy in the vignette mandates this approach. Similarly, inability to abduct one eye can be congenital, but if this is a new finding, a brain or brainstem lesion must be ruled out. In this case, headache and the oculomotor findings make a brain or brainstem lesion likely. Gait impairment and hyperreflexia only in the legs can result from hydrocephalus or can localize to the mid- to lower spinal cord. A spinal cord lesion would not explain the headache, and bilateral motor findings without sensory, bowel, or bladder involvement are uncommon for a spinal cord lesion. Therefore, MRI of the spine is not needed. Similarly, the hyperreflexia is not evidence of a nerve or muscle problem, obviating the need for electromyography and nerve conduction studies. Lumbar puncture is therapeutic for headache due to pseudotumor cerebri, which could present with this constellation of symptoms, but this degree of gait abnormality would be uncommon in pseudotumor, and the presence of true hydrocephalus or an intracranial mass must be excluded with a head CT or brain MRI prior to lumbar puncture. Visual evoked potentials occasionally are used to determine whether the axons from eye to occipital cortex function normally, but such a test generally is performed when demyelinating disorders are suspected. References: Avellino AM. Hydrocephalus. In: Singer HS, Kossoff EH, Hartman AL, Crawford TO, eds. Treatment of Pediatric Neurologic Disorders. Boca Raton, Fla: Taylor & Francis; 2005:25-36 Garton HJ, Piatt JH Jr. Hydrocephalus. Pediatr Clin North Am. 2004;51:305-325. Abstract available at: http://www.ncbi.nlm.nih.gov/pubmed/15062673 Kestle JR. Pediatric hydrocephalus: current management. Neurol Clin. 2003;21:883-895. Abstract available at: http://www.ncbi.nlm.nih.gov/pubmed/14743654
  • 75. 2009 PREP SA on CD-ROM Kuttesch J Jr, Ater JL. Brain tumors in childhood. In: Behrman RE, Kliegman RM, Jenson HB, Stanton BF, eds. Nelson Textbook of Pediatrics. 18th ed. Philadelphia, Pa: Saunders Elsevier; 2007:2128-2136 Piatt JH Jr. Recognizing neurosurgical conditions in the pediatrician's office. Pediatr Clin North Am. 2004;51:237-270. Abstract available at: http://www.ncbi.nlm.nih.gov/pubmed/15062671 page 75
  • 76. 2009 PREP SA on CD-ROM Question: 23 Parents who are new to your area bring in their 3-year-old daughter for evaluation because they are concerned about her delayed speech. They say that she uses about 50 single words. The girl has had tetralogy of Fallot repaired surgically and recurrent upper respiratory tract infections with otitis media, for which tympanostomy tubes have been placed. Findings on physical examination include microcephaly, underfolded pinnae, a broad nasal bridge, cleft uvula, and a small chin. In addition, the child’s speech has a hypernasal quality. The family history is negative for birth defects and developmental delays. Of the following, the contiguous gene deletion syndrome that BEST fits this child’s features is page 76 A. Angelman B. Beckwith-Wiedemann C. 4p- D. Prader-Willi E. 22q11